You are on page 1of 100

Chapter 20 Rationale: A HbA1c level of 13%

indicates poor glucose control. This, in


1. The nurse is teaching a pregnant conjunction with the woman being in the
woman with type 1 diabetes about her first trimester, increases the risk for
diet during pregnancy. Which client congenital anomalies in the fetus.
statement indicates that the nurse's Elevated glucose levels are not
teaching was successful? associated with incompetent cervix,
A. "I'll basically follow the same diet that placenta previa, or placental
I was following before I became abruption (abruptio placentae).
pregnant."
B. "Because I need extra protein, I'll 3. A nurse is conducting a review class
have to increase my intake of milk and for a group of perinatal nurses working
meat." at the local clinic.
C. "Pregnancy affects insulin production, The clinic sees a high population of
so I'll need to make adjustments in my women who are HIV positive. After
diet." discussing the
D. "I'll adjust my diet and insulin based recommendations for antiretroviral
on the results of my urine tests for therapy with the group, the nurse
glucose." determines that the teaching was
successful when the group identifies
Rationale: In pregnancy, placental which rationale as the underlying
hormones cause insulin resistance at a principle for the
level that tends to therapy?
parallel growth of the fetoplacental unit. A. reduction in viral loads in the blood
Nutritional management focuses on B. treatment of opportunistic infections
maintaining C. adjunct therapy to radiation and
balanced glucose levels. Thus, the chemotherapy
woman will probably need to make D. can cure acute HIV/AIDS infections
adjustments in her diet.
Protein needs increase during Rationale: Drug therapy is the mainstay
pregnancy, but this is unrelated to of treatment and is important in reducing
diabetes. Blood glucose the viral load
monitoring results typically guide as much as possible. Antiretroviral
therapy. agents do not treat opportunistic
infections and are not
2. A pregnant woman with diabetes at adjunctive therapy. There is no cure for
10 weeks' gestation has a glycosylated HIV/AIDS.
hemoglobin (HbA1c) level of 13%. At
this time the nurse should be most 4. Assessment of a pregnant woman
concerned about which possible and her fetus reveals tachycardia and
fetal outcome? hypertension. There is
A. congenital anomalies also evidence suggesting
B. incompetent cervix vasoconstriction. The nurse would
C. placenta previa question the woman about use of
D. placental abruption (abruptio which substance?
placentae) A. marijuana
B. alcohol
C. heroin
D. cocaine D. "Antiretroviral medications are
available to help reduce the risk of
Rationale: Cocaine use produces transmission."
vasoconstriction, tachycardia, and
hypertension in both the Rationale: Drug therapy is the mainstay
mother and fetus. The effects of of treatment for pregnant women
marijuana are not yet fully understood. infected with HIV. The
Alcohol ingestion would goal of therapy is to reduce the viral
lead to cognitive and behavioral load as much as possible; this reduces
problems in the newborn. Heroin is a the risk of
central nervous system transmission to the fetus. Decisions
depressant. about the method of birth should be
based on the woman's
5. When teaching a class of pregnant viral load, duration of ruptured
women about the effects of substance membranes, progress of labor, and
use during pregnancy, the nurse would other pertinent clinical factors.
include which effect? The newborn is at risk for HIV because
A. low-birthweight infants of potential perinatal transmission.
B. excessive weight gain Waiting until after
C. higher pain tolerance the infant is born may be too late.
D. longer gestational periods
7. When preparing a schedule of
Rationale: Substance use during follow-up visits for a pregnant woman
pregnancy is associated with low birth with chronic hypertension, which
weight infants, preterm schedule would be most appropriate?
labor, abortion, intrauterine growth A. monthly visits until 32 weeks, then
restriction, abruptio placentae, bi-monthly visits
neurobehavioral B. bi-monthly visits until 28 weeks, then
abnormalities, and long-term childhood weekly visits
developmental consequences. C. monthly visits until 20 weeks, then
Excessive weight gain, bi-monthly visits
higher pain tolerance, and longer D. bi-monthly visits until 36 weeks, then
gestational periods are not associated weekly visits
with substance use.
Rationale: For the woman with chronic
6. A client who is HIV-positive is in her hypertension, antepartum visits typically
second trimester and remains occur every 2
asymptomatic. She voices concern weeks until 28 weeks' gestation and
about her newborn's risk for the then weekly to allow for frequent
infection. Which statement by the nurse maternal and fetal
would be most surveillance.
appropriate?
A. "You'll probably have a cesarean birth 8. A woman with a history of asthma
to prevent exposing your newborn." comes to the clinic for evaluation for
B. "Antibodies cross the placenta and pregnancy. The
provide immunity to the newborn." woman's pregnancy test is positive.
C. "Wait until after the infant is born, and When reviewing the woman's
then something can be done." medication therapy regimen
for asthma, which medication would the cannot tolerate the gastrointestinal
nurse identify as problematic for the discomfort it causes. In such cases, the
woman now that woman should take it with meals. Iron
she is pregnant? typically causes the stool to become
A. ipratropium black and tarry; there is no need for the
B. albuterol woman to notify her primary care
C. salmeterol provider.
D. Prednisone
10. The nurse is assessing a newborn of
Rationale: Oral corticosteroids such as a woman who is suspected of abusing
prednisone are not preferred for the alcohol. Which
long-term treatment newborn finding would provide
of asthma during pregnancy. Inhaled additional evidence to support this
steroids are the choice for maintenance suspicion?
medications to A. wide, large eyes
reduce inflammation that leads to B. thin upper lip
bronchospasm. Common ones C. protruding jaw
prescribed include D. elongated nose
beclomethasone and salmeterol.
Rescue agents such as albuterol or Rationale: Newborn characteristics
ipratropium provide suggesting fetal alcohol spectrum
immediate symptomatic relief by disorder include thin upper
reducing acute bronchospasm. lip, small head circumference, small
eyes, receding jaw, and short nose.
9. A pregnant woman is diagnosed with Other features include a
iron-deficiency anemia and is prescribed low nasal bridge, short palpebral
an iron fissures, flat midface, epicanthal folds,
supplement. After teaching her about and minor ear
her prescribed iron supplement, which abnormalities.
statement indicates
successful teaching? 11. After teaching a group of nurses
A. "I should take my iron with milk." working at the women's health clinic
B. "I should avoid drinking orange juice." about the impact of
C. "I need to eat foods high in fiber." pregnancy on the older woman, which
D. "I'll call the primary care provider if statement by the group indicates that
my stool is black and tarry." the teaching was
successful?
Rationale: Iron supplements can lead to A. "The majority of women who become
constipation, so the woman needs to pregnant over age 35 experience
increase her intake complications."
of fluids and high-fiber foods. Milk B. "Women over the age of 35 who
inhibits absorption and should be become pregnant require a specialized
discouraged. Vitamin C- type of assessment."
containing fluids such as orange juice C. "Women over age 35 and are
are encouraged because they promote pregnant have an increased risk for
absorption. Ideally the woman should spontaneous abortions."
take the iron on an empty stomach to D. "Women over age 35 are more likely
improve absorption, but many women to have a substance use disorder."
Rationale: Infection with gonorrhea
Rationale: Whether childbearing is during pregnancy can cause ophthalmia
delayed by choice or by chance, women neonatorum in the
starting a family at newborn from birth through an infected
age 35 or older are not doing so without birth canal. Infection with syphilis can
risk. Women in this age group may cause congenital
already have chronic syphilis in the neonate. Infection with
health conditions that can put the chlamydia can lead to conjunctivitis or
pregnancy at risk. In addition, numerous pneumonia in the
studies have shown newborn. Exposure to HPV during birth
that increasing maternal age is a risk is associated with laryngeal papillomas.
factor for infertility and spontaneous
abortions, gestational 13. A nurse is preparing a presentation
diabetes, chronic hypertension, for a group of young adult pregnant
postpartum hemorrhage, preeclampsia, women about
preterm labor and birth, multiple common infections and their effect on
pregnancy, genetic disorders and pregnancy. When describing the
chromosomal abnormalities, placenta infections, which
previa, fetal growth restriction, low infection would the nurse include as the
Apgar scores, and surgical births (Dillion most common congenital and perinatal
et al. 2019). However, even viral infection in
though increased age implies increased the world?
complications, most women today who A. rubella
become pregnant B. hepatitis B
after age 34, have healthy pregnancies C. cytomegalovirus
and healthy newborns. Nursing D. parvovirus B19
assessment of the Rationale: Although rubella, hepatitis B,
pregnant woman over age 35 is the and parovirus B19 can affect pregnant
same as that for any pregnant woman. women and their
Women of this age fetuses, cytomegalovirus (CMV) is the
have the same risk for a substance use most common congenital and perinatal
disorder as any other age group. viral infection in
the world. CMV is the leading cause of
12. A nurse is conducting an in-service congenital infection, with morbidity and
presentation to a group of perinatal mortality at birth and sequelae. Each
nurses about sexually year approximately 1% to 7% of
transmitted infections and their effect on pregnant women acquire a primary
pregnancy. The nurse determines that CMV infection. Of these, about 30% to
the teaching was 40% transmits infection to their fetuses.
successful when the group identifies
which infection as being responsible for 14. A pregnant woman asks the nurse,
ophthalmia "I'm a big coffee drinker. Will the caffeine
neonatorum? in my coffee hurt my baby?" Which
A. syphilis response by the nurse would be most
B. gonorrhea appropriate?
C. chlamydia A. "The caffeine in coffee has been
D. HPV linked to birth defects."
B. "Caffeine has been shown to restrict 16. A nurse has been invited to speak at
growth in the fetus." a local high school about adolescent
C. "Caffeine is a stimulant and needs to pregnancy. When
be avoided completely." developing the presentation, the nurse
D. "If you keep your intake to less than would incorporate information related to
200 mg/day, you should be okay." which aspects?
Select all that apply.
Rationale: The effect of caffeine intake A. peer pressure to become sexually
during pregnancy on fetal growth and active
development is still unclear. A recent B. rise in teen birth rates over the years.
study found that caffeine intake of no C. Asian Americans as having the
more than 200 mg/day during highest teen birth rate
pregnancy does not affect pregnancy D. loss of self-esteem as a major impact
duration and the condition of the E. about half occurring within a year of
newborn. Birth defects first sexual intercourse
have not been linked to caffeine
consumption, but maternal coffee Rationale: Adolescent pregnancy has
consumption decreases iron emerged as one of the most significant
absorption and may increase the risk of social problems
anemia during pregnancy. It is not facing our society. Early pregnancies
known if there is a correlation between among adolescents have major health
high caffeine intake and miscarriage due consequences for
to lack of sufficient studies. mothers and their infants. The latest
estimates show that approximately 1
15. A neonate born to a mother who million teenagers
was abusing heroin is exhibiting signs become pregnant each year in the
and symptoms of United States, accounting for 13% of all
withdrawal. Which signs would the U.S. births, but the
nurse assess? Select all that apply. rates have been declining in the last
A. low whimpering cry several years. Teen birth rates in the
B. hypertonicity United States have
C. lethargy declined but remain high,especially
D. excessive sneezing among African American and Hispanic
E. overly vigorous sucking teenagers and
F. tremors adolescents in southern states. The
most important impact lies in the
Rationale: Signs and symptoms of psychosocial area as it
withdrawal, or neonatal abstinence contributes to a loss of self-esteem, a
syndrome, include: destruction of life projects, and the
irritability, hypertonicity, excessive and maintenance of the
often high-pitched crying, vomiting, circle of poverty. Moreover, about half of
diarrhea, feeding all teen pregnancies occur within 6
disturbances, respiratory distress, months of first
disturbed sleeping, excessive sneezing having sexual intercourse. About one in
and yawning, nasal four teen mothers under age 18 have a
stuffiness, diaphoresis, fever, poor second baby
sucking, tremors, and seizures. within 2 years after the birth of the first
baby.
There is no immunization for CMV.
17. A nurse is counseling a pregnant Antibody titer levels would be useful for
woman with rheumatoid arthritis about identifying women
medications that can at risk for rubella.
be used during pregnancy. The nurse
would emphasize the need to avoid 19. A pregnant woman comes to the
which medication at clinic for her first evaluation. The woman
this time? is screened for
A. hydroxychloroquine hepatitis B (HBV) and tests positive. The
B. nonsteroidal anti-inflammatory drugs nurse would anticipate administering
C. glucocorticoid which agent?
D. methotrexate A. HBV immune globulin
B. HBV vaccine
Rationale: Methotrexate is C. acylcovir
contraindicated during pregnancy. For D. valacyclovir
rheumatoid arthritis,
medications are limited to Rationale: If a woman tests positive for
hydroxychloroquine, glucocorticoids, HBV, expect to administer HBV immune
and NSAIDS. globulin. The
newborn will also receive HBV vaccine
18. A nurse is preparing a teaching within 12 hours of birth. Acyclovir or
program for a group of pregnant women valacyclovir
about preventing would be used to treat herpes simplex
infections during pregnancy. When virus infection.
describing measures for preventing
cytomegalovirus 20. After teaching a pregnant woman
infection, which measure would the with iron deficiency anemia about
nurse include as a priority? nutrition, the nurse
A. frequent handwashing determines that the teaching was
B. immunization successful when the woman identifies
C. prenatal screening which foods as being
D. antibody titer screening good sources of iron in her diet? Select
all that apply.
Rationale: Most women are A. dried fruits
asymptomatic and do not know they B. peanut butter
have been exposed to CMV. C. meats
Prenatal screening for CMV infection is D. milk
not routinely performed. Since there is E. white bread
no therapy that
prevents or treats CMV infections, Rationale: Foods high in iron include
nurses are responsible for educating meats, green leafy vegetables, legumes,
and supporting dried fruits, whole
childbearing-age women at risk for CMV grains, peanut butter, bean dip,
infection. Stressing the importance of whole-wheat fortified breads, and
good handwashing and use of sound cereals.
hygiene practices can help to reduce
transmission of the virus.
21. A pregnant woman with gestational for long-term diabetic complications
diabetes comes to the clinic for a fasting such as retinopathy or nephropathy, as
blood glucose evidenced by
\level. When reviewing the results, the laboratory testing such as BUN levels, is
nurse determines that the woman is an important aspect of preconception
achieving good glucose control based care to ensure that
on which result? the mother enters the pregnancy in an
A. 88 mg/dL optimal state.
B. 100 mg/dL
C. 110 mg/dL 23. A nurse is providing care to several
D. 120 mg/dL pregnant women at different weeks of
gestation. The
Rationale: For a pregnant woman with nurse would expect to screen for group
diabetes, the ADA and ACOG B streptococcus infection in the client
recommend maintaining a who is at:
fasting blood glucose level below 95 A. 16 weeks' gestation.
mg/dL, with postprandial levels below B. 28 weeks' gestation.
140 mg/dL at 1 C. 32 weeks' gestation.
hour, below 120 mg/dL at 2 hours. D. 36 weeks' gestation.

22. A nurse is conducting a program for Rationale: Pregnant women between


pregnant women with gestational 36 and 37 weeks’ gestation should be
diabetes about universally screened
reducing complications. The nurse for GBS infection during a prenatal visit
determines that the teaching was and if positive, receive appropriate
successful when the group intrapartum
identifies which factor as being most antibiotic prophylaxis.
important in helping to reduce
complications associated 24. A woman with a history of systemic
with pregnancy and diabetes? lupus erythematosus comes to the clinic
A. stability of the woman's emotional for evaluation.
and psychological status The woman tells the nurse that she and
B. degree of blood glucose control her partner would like to have a baby
achieved during the pregnancy but that they are
C. reduction in retinopathy risk by afraid her lupus will be a problem.
frequent ophthalmologic evaluations Which response would be most
D. control of blood urea nitrogen (BUN) appropriate for the nurse to
levels for optimal kidney function make?
A. "It's probably not a good idea for you
Rationale: Therapeutic management to get pregnant since you have lupus."
for the woman with diabetes focuses on B. "Be sure that your lupus is stable or
tight glucose in remission for 6 months before getting
control, thereby minimizing the risks to pregnant."
the mother, fetus, and neonate. The C. "Your lupus will not have any effect
woman's emotional on your pregnancy whatsoever."
and psychological status is highly D. "If you get pregnant, we'll have to add
variable and may or may not affect the quite a few medications to your normal
pregnancy. Evaluating treatment plan.
B. cooking all meat to an internal
Rationale: The time at which the nurse temperature of 125° F (52° C)
comes in contact with the woman in her C. wearing gardening gloves when
childbearing life working in the soil
cycle will determine the focus of the D. avoiding contact with a cat's litter box
assessment. If the woman is
considering pregnancy, it is Rationale: Meats should be cooked to
recommended that she postpone an internal temperature of 160° F
conception until the disease has been (71°C). Other measures to prevent
stable or in remission for 6 toxoplasmosis include peeling or
months. Active disease at time of thoroughly washing all raw fruits and
conception and history of renal disease vegetables before eating them, wearing
increase the likelihood gardening gloves when in contact with
of a poor pregnancy outcome outdoor soil, and avoiding
(Cunningham et al., 2018). In particular, the emptying or cleaning of a cat's litter
if pregnancy is planned box.
during periods of inactive or stable
disease, the result is often giving birth to 26. A pregnant client with iron-deficiency
healthy full-term anemia is prescribed an iron
babies without increased risks of supplement. After
pregnancy complications. Nonetheless, teaching the woman about using the
pregnancies with most supplement, the nurse determines that
autoimmune diseases are still classified more teaching is
as high risk because of the potential for needed based on which client
major complications. Preconception statement?
counseling should include the medical A. "Taking the iron supplement with food
and obstetric risks of will help with the side effects."
spontaneous abortion, stillbirth, fetal B. "I will need to avoid coffee and tea
death, fetal growth restriction, when I take this supplement."
preeclampsia, preterm labor, C. "I will take the iron with milk instead
and neonatal death and the need for of orange or grapefruit juice."
more frequent visits for monitoring the D. "If I happen to miss a dose, I will take
condition. Treatment it as soon as I remember."
of SLE in pregnancy is generally limited
to NSAIDs (e.g., ibuprofen), prednisone, Rationale: The pregnant client should
and an antimalarial agent, take the iron supplement with vitamin
hydroxychloroquine. During pregnancy C-containing fluids
in the woman with SLE, the goal is to such as orange juice, which will promote
keep drug therapy to a minimum. absorption, rather than milk, which can
inhibit iron
25. A nurse is conducting a presentation absorption. Taking iron on an empty
for a group of pregnant women about stomach improves its absorption, but
measures to prevent toxoplasmosis. many women cannot
The nurse determines that additional tolerate the gastrointestinal discomfort it
teaching is needed when the group causes. In such cases, the woman is
identifies which measure as preventive? advised to take it
A. washing raw fruits and vegetables
before eating them
with meals. The woman also needs 11 g/dL or 110 g/L), low hematocrit (less
instruction about adverse effects, which than 35% or 0.35), low serum iron (less
are predominantly than 30 µg/dL
gastrointestinal and include gastric or 5.37 µmol/L), microcytic and
discomfort, nausea, vomiting, anorexia, hypochromic cells, and low serum
diarrhea, metallic ferritin (less than 100 ng/dL
taste, and constipation. Taking the iron or 100 µg/L). The client's hemoglobin,
supplement with meals and increasing hematocrit, and serum iron levels are
intake of fiber and borderline low
fluids helps overcome the most common normal, but the client's serum ferritin is
side effects. If the woman misses a below 100 ng/dL (100 µg/L), helping to
dose, she should support the
take a dose as soon as she remembers. diagnosis.

28. A young adult woman comes to the


clinic for a routine check-up. During the
visit, the woman
27. A client in her first trimester comes who works in a day care facility tells the
to the clinic for an evaluation. nurse that she and her partner are
Assessment reveals considering having a
reports of fatigue, anorexia, and baby. "We are concerned that I might be
frequent upper respiratory infections. exposed to common childhood
The client's skin is pale illnesses." The woman
and the client is slightly tachycardic. The undergoes testing and finds out that she
client also reports drinking about 6 cups is not immune from chickenpox. Based
of coffee on on this
average each day. A diagnosis of information, which information would the
iron-deficiency anemia is suspected. nurse give to the client?
The client is scheduled for A. "You will need to be vaccinated now
laboratory testing and the results are as and wait at least 1 month before getting
follows: pregnant."
• Hemoglobin 11.5 g/dL (115 g/L) B. "It is very likely that you will need to
• Hematocrit 35% (0.35) quit your job if you do get pregnant."
• Serum iron 32 µg/dL (5.73 µmol/L) C. "Because chickenpox is so rare
• Serum ferritin 90 ng/dL (90 µg/L nowadays, there is nothing to worry
about."
Which laboratory finding would the D. "You will need to take a leave of
nurse correlate with the suspected absence during winter and spring
diagnosis? months."
A. Hemoglobin
B. Hematocrit Rationale: Preconception counseling is
C. Serum iron level important for preventing chickenpox
D. Serum ferritin level (varicella). A major component of
counseling involves determining the
Rationale: Laboratory tests for woman's varicella immunity. Vaccination
iron-deficiency anemia usually reveal is the cornerstone of prevention. The
low hemoglobin (less than vaccine is administered if needed.
Varicella vaccine is a live
attenuated viral vaccine. It should be used to treat rheumatoid arthritis but are
administered to all adolescents and contraindicated for use in pregnancy
adults 13 years of age because of the
and older who do not have evidence of potential for fetal toxicity.
varicella immunity. Therefore, the
woman should be 30. The nurse reviews the medical
vaccinated now before she becomes record of a woman who has come to the
pregnant and then wait at least 1 month clinic for an
before getting evaluation. The client has a history of
pregnant. The varicella vaccine is mitral valve prolapse and is listed as risk
contraindicated for pregnant women class II. During the visit, the woman
because the effects of the states, "We want to have a baby, but I
vaccine on the fetus are unknown. know I am at higher risk. But what is
There is no need for the woman to quit my risk, really?" Which response by the
her job once she is nurse would be appropriate?
immunized nor does she need to take a A. "If you do get pregnant, you will need
leave of abscence during the winter and to be seen by a cardiologist every other
spring months month for
when the incidence is highest. monitoring."
Chickenpox does occur and is highly B. "Your risk during pregnancy is small,
contagious. Maternal but you should see your cardiologist first
varicella can be transmitted to the fetus before getting
through the placenta, leading to pregnant."
congenital varicella C. "Your heart disease would put too
syndrome if the mother is infected much strain on your heart if you were to
during the first half of pregnancy via an get pregnant."
ascending aorta. D. "Your pregnancy would be
uneventful, but you would need
29. A nurse is obtaining a medication specialized care for labor and
history from a pregnant client with a birth."
history of systemic
lupus erythematosus (SLE). Which
medication(s) would the nurse expect
the woman to report to
be currently using? Select all that apply. Rationale: Typically, a woman with
A. Ibuprofen class I or II cardiac disease can go
B. Hydroxychloroquine through a pregnancy
C. Methotrexate without major complications. For class I
D. Leflunomide disease, there is no detectable
E. Prednisone increased risk of maternal
mortality and no increase or a mild
Rationale: Treatment of SLE in increase in morbidity. For class II
pregnancy is generally limited to disease, there is a small
NSAIDs like ibuprofen, increased risk of maternal mortality or
prednisone, and an antimalarial agent, moderate increase in morbidity and
hydroxychloroquine. Methotrexate and cardiac consultation
leflunomide are
should occur every trimester. It is best to pressure and to assess for any signs of
have the woman see her cardiologist preeclampsia. At approximately 24
before becoming weeks’ gestation, the woman will be
pregnant. A woman with class III instructed to document fetal movement.
disease needs frequent visits with the At this same time, serial ultrasounds will
cardiac care team be prescribed to monitor fetal growth
throughout pregnancy. There is a and amniotic fluid volume. The woman
significantly increased risk of maternal should also have daily periods of
mortality or severe rest (1 hour) in the left lateral recumbent
morbidity and cardiologist consult position to maximize placental perfusion
should occur every other month with and use home blood pressure
prenatal care and monitoring devices frequently (daily
delivery occurring at an appropriate checks would be preferred), reporting
level hospital. A woman with class IV any elevations.
disease is typically
advised to avoid pregnancy. 32. A pregnant client with type 1
diabetes is in labor. The client's blood
31. A pregnant woman with chronic glucose levels are being monitored
hypertension is entering her second every hour and she has a prescription
trimester. The nurse is for an infusion of regular insulin as
providing anticipatory guidance to the needed based on the client's blood
woman about measures to promote a glucose levels. Her levels are as follows:
healthy outcome. 1300: 105 mg/dL (5.83 mmol/L)
The nurse determines that the teaching 1400: 100 mg/dL (5.55 mmol/L)
was successful based on which client 1500: 120 mg/dL (6.66 mmol/L)1600:
statement(s)? Select 106 mg/dl (5.88 mmol/L)
all that apply.
A. "I will need to schedule follow-up Based on the recorded blood glucose
appointments every 2 weeks until I levels, at which time would the nurse
reach 32 weeks' likely administer the regular insulin
gestation." infusion?
B. "I should try to lie down and rest on
my left side for about an hour each day."
C. "I will start doing daily counts of my Rationale: For the laboring woman with
baby's activity at about 24 weeks' diabetes, intravenous (IV) saline or
gestation." lactated Ringer's is given, and blood
D. "I will need to have an ultrasound at glucose levels are monitored every 1 to
each visit beginning at 28 weeks' 2 hours. Glucose levels are
gestation." maintained below 110 mg/dL (6.11
E. "I should take my blood pressure mmol/L) throughout labor to reduce the
frequently at home and report any high likelihood of neonatal hypoglycemia. If
readings." necessary, an infusion of regular insulin
may be given to maintain this level. The
Rationale: The woman with chronic insulin infusion would be given at 1500,
hypertension will be seen more based on the blood glucose level being
frequently (every 2 weeks higher than 110 mg/dL (6.11 mmol/L).
until 28 weeks' gestation and then
weekly until birth) to monitor her blood
33. A pregnant client with iron-deficiency for delivery before making any decisions
anemia is prescribed an iron on delivery. After determining the
supplement. After teaching the woman readiness of the fetus, then plans for
about using the supplement, the nurse delivery can be determined and
determines that more teaching is scheduled.
needed based on which client
statement? 35. The nurse is assessing a pregnant
"I will take the iron with milk instead of client with a known history of congestive
orange or grapefruit juice." heart failure who is in her third trimester.
Which assessment findings should the
Rationale: The pregnant client should nurse prioritize?
take the iron supplement with vitamin dyspnea, crackles, and irregular weak
C-containing fluids such as orange juice, pulse
which will promote absorption, rather
than milk, which can inhibit iron Rationale: The nurse should be alert for
absorption. Taking iron on an empty signs of cardiac decompensation due to
stomach improves its absorption, but congestive heart failure, which include
many women cannot tolerate the crackles in the lungs from fluid, difficulty
gastrointestinal discomfort it causes. In breathing, and weak pulse from heart
such cases, the woman is advised to exhaustion. The heart rate would not be
take it with meals. The woman also regular, and a cough would not be dry.
needs instruction about adverse effects, The heart rate would increase rather
which are predominantly gastrointestinal than decrease.
and include gastric discomfort, nausea,
vomiting, anorexia, diarrhea, metallic 36. A woman with known cardiac
taste, and constipation. Taking the iron disease from childhood presents at the
supplement with meals and increasing obstetrician's office 6 weeks' pregnant.
intake of fiber and fluids helps overcome What recommendations would the nurse
the most common side effects. If the make to the client to address the known
woman misses a dose, she should take cardiac problems for this pregnancy?
a dose as soon as she remembers. Select all that apply.
- Plan periods of rest into the workday.
34. A 17-year-old primigravida with type - Receive pneumococcal and influenza
1 diabetes is at 37 weeks' gestation vaccines.
comes to the clinic for an evaluation. - Let the physician know if you become
The nurse notes her blood sugar has short of breath or have a nighttime
been poorly controlled and the health cough.
care provider is suspecting the fetus has
macrosomia. The nurse predicts which Rationale: Women with known heart
step will be completed next? conditions need to be closely followed
Preparing for amniocentesis and fetal by both the obstetrician and a
lung maturity assessment cardiologist. Recommendations would
include rest periods, reduction of stress,
Rationale: If the infant has macrosomia, getting immunizations, and monitoring
is large for gestation age, and the for heart failure as demonstrated by a
mother has had poor blood-sugar nighttime cough and shortness of
control, the provider will want further breath. Consuming more sodium in the
information on the fetus and readiness diet is not recommended due of the
potential of developing hypertension. these symptoms, the nurse suspects
Warfarin is contraindicated during which of the following conditions?
pregnancy since it crosses the placental Appendicitis
barrier and can cause spontaneous
abortion, stillbirth or preterm birth. Rationale: With appendicitis, the
nausea and vomiting is much more
37. A young woman with scoliosis has intense than with morning sickness and
just learned that she is pregnant. the pain is sharp and localized at
Several years ago, she had McBurney's point (a point halfway
stainless-steel rods surgically implanted between the umbilicus and the iliac crest
on both sides of her vertebrae to on the lower right abdomen). With a
strengthen and straighten her spine. ruptured ectopic pregnancy, a woman
However, her pelvis is unaffected by the may experience abdominal pain that is
condition. Which of the following does either diffuse or sharp, but it is less likely
the nurse anticipate in this woman's to occur precisely at McBurney's point.
pregnancy? The symptoms described do not match
Potential for greater than usual back those of pulmonary embolism or
pain left-sided heart failure.

Rationale: Surgical correction of 39. In women with cardiac failure, the


scoliosis (lateral curvature of the spine) maternal blood pressure becomes
involves implanting stainless-steel rods insufficient to provide an adequate
on both sides of the vertebrae to supply of blood to the placenta. The
strengthen and straighten the spine. infant will likely experience some
Such rod implantations do not interfere undesired effects, including which of the
with pregnancy; a woman may notice following?
more than usual back pain, however, low birth weight
from increased tension on back
muscles. If a woman's pelvis is distorted Rationale: Cardiac failure can affect
due to scoliosis, a cesarean birth may fetal growth at the point at which
be scheduled to ensure a safe birth, but maternal blood pressure becomes
this is not required in this scenario. insufficient to provide an adequate
Vaginal birth, if permitted, requires the supply of blood and nutrients to the
same management as for any woman. placenta. For this reason, the infant may
With the improved management of tend to have a low birth weight, be
scoliosis, the high maternal and preterm, and respond poorly to labor.
perinatal risks associated with the
disorder reported in earlier literature no 40. Which factor would contribute to a
longer exist. high-risk pregnancy?
type 1 diabetes
38. A client in week 38 of her pregnancy
arrives at the emergency room reporting Rationale: A woman with a history of
a sharp pain between her umbilicus and diabetes has an increased risk for
the iliac crest in her lower right abdomen perinatal complications, including
that is increasing. She reports having hypertension, preeclampsia, and
experienced intense nausea and neonatal hypoglycemia. The age of 33
vomiting for the past 3 hours. Given without other risk factors does not
increase risk, nor does type O-positive client would be given antibiotics only if
blood or environmental allergens. there were evidence of an infection.

41. A nurse is providing education to a 44. A mother is talking to the nurse and
woman at 28 weeks' gestation who has is concerned about managing her
tested positive for gestational diabetes asthma while she is pregnant. Which
mellitus (GDM). What would be response to the nurse's teaching
important for the nurse to include in the indicates that the woman needs further
client teaching? instruction?
She is at increased risk for type 2 "I need to begin taking allergy shots like
diabetes mellitus after her baby is born. my friend to prevent me from having an
allergic reaction this spring."
Rationale: The woman who develops
GDM is at increased risk for developing Rationale: A pregnant woman with a
type 2 diabetes mellitus after pregnancy. history of asthma needs to be proactive,
taking her inhalers and other asthma
medications to prevent an acute asthma
attack. She needs to understand that it
42. The maternal health nurse is caring is far more dangerous to not take the
for a group of high-risk pregnant clients. medications and have an asthma attack.
Which client condition will the nurse She also needs to monitor her peak flow
identify as being the highest risk for for decreases, be aware of triggers, and
pregnancy? avoid them if possible. However, a
Pulmonary hypertension pregnant woman should never begin
allergy shots if she has not been taking
Rationale: Pulmonary hypertension is them previously, due to the potential of
considered the greatest risk to a an adverse reaction.
pregnancy because of the hypoxia that
is associated with the condition. The 45. The nurse is preparing information
remaining conditions represent potential for a client who has just been diagnosed
cardiac complications that may increase with gestational diabetes. Which
the client's risk in pregnancy; however, instruction should the nurse prioritize in
these do not present the greatest risk in this information?
pregnancy. Maintain a daily blood glucose log

43. A pregnant client with sickle cell Rationale: Control of the blood glucose
anemia is admitted in crisis. Which throughout the pregnancy is the primary
nursing intervention should the nurse goal to help decrease potential
prioritize? complications to both the mother and
IV fluids fetus. The mother should keep a daily
log of her blood glucose levels and bring
Rationale: A sickle cell crisis during this log to each visit for the nurse to
pregnancy is usually managed by evaluate. The other choices of reporting
exchange transfusion, oxygen, and IV possible signs of a UTI and working with
fluids. Antihypertensive drugs usually a dietitian to plan menus would also be
aren't necessary. Diuretics would not be important but would be secondary to the
used unless fluid overload resulted. The blood glucose control. It would be
inappropriate to discuss long-term goals
at this time. This would be handled at a Rationale: The upper normal level of
later time and would depend on the HbA1C is 6% of total hemoglobin.
mother's situation.
48. A pregnant woman who has been
46. A 32-year-old woman with epilepsy taking penicillin prophylactically because
mentions to the nurse during a routine she had rheumatic fever as a child tells
well-visit that she would like to have the nurse that she wants to stop taking it
children and asks the nurse for advice. now that she is pregnant. Which of the
Which response is most appropriate following is the best response by the
from the nurse? nurse?
"I'll let the doctor know so you can "You should continue taking this drug,
discuss your medications. In the because penicillin is not known to be a
meantime, I'll give you a list of folate-rich fetal teratogen."
foods you can add to your diet."
Rationale: A woman taking penicillin
Rationale: Any woman with epilepsy prophylactically because she had
needs to discuss the medication rheumatic fever as a child and wants to
management with her provider. The prevent a recurrence should continue
current research indicates the this drug during pregnancy. Penicillin is
medications used for epileptic not known to be a teratogen.
management are the major cause of
birth defects for these patients. The 49. A nursing instructor is teaching
nurse should be careful about students about caring for a pregnant
mentioning that some epileptics are patient with a pre-existing disease.
teratogenic; some women may stop Which of the following does the
taking their medications in order to get instructor suggest has added to an
pregnant. Suggesting adoption is increased incidence of pregnant women
inappropriate as the mother has given with a pre-existing disease?
no indication she is interested in More women waiting until after age 30
adoption; also, the mother needs to years to get pregnant
discuss this with the physician so that
she can get accurate information about Rationale: As more women wait until
being on anti-seizure medications and they are older than 30 years to have
being pregnant. The nurse should not their first child, more also enter
share personal information as it does pregnancy with a pre-existing disorder.
not assist this client in making a serious 50. The nurse is teaching a pregnant
decision. The client should be referred woman with type 1 diabetes about her
to the health care provider to help the diet during pregnancy. Which client
client make the best decision. statement indicates that the nurse's
teaching was successful?
47. A pregnant woman with type 2 "Pregnancy affects insulin production,
diabetes is scheduled for a laboratory so I'll need to make adjustments in my
test of glycosylated hemoglobin diet."
(HbA1C). What does the nurse tell the
client is a normal level for this test? Rationale: In pregnancy, placental
6% hormones cause insulin resistance at a
level that tends to parallel growth of the
fetoplacental unit. Nutritional
management focuses on maintaining Drug metabolism changes during
balanced glucose levels. Thus, the pregnancy
woman will probably need to make
adjustments in her diet. Protein needs Rationale: Drug metabolism changes
increase during pregnancy, but this is during pregnancy which may alter the
unrelated to diabetes. Blood glucose therapeutic AED levels in the pregnant
monitoring results typically guide client. Some AEDs cannot be given in
therapy. pregnancy due to risk of harm to the
fetus; however, there are some that may
51. The nurse is appraising the be given. Pregnant clients do not have
laboratory results of a pregnant client high rates of noncompliance and the
who is in her second trimester and notes action of medications do not change in
the following: thyroid stimulating pregnancy.
hormone (TSH) slightly elevated,
glucose in the urine, complete blood 53. A pregnant client with a history of
count (CBC) low normal, and normal asthma since childhood presents for a
electrolytes. The nurse prioritizes further prenatal visit. What statement by the
testing to rule out which condition? client would the nurse prioritize?
Gestational diabetes "I sometimes get a bit wheezy."

Rationale: Glycosuria, glucose in the Rationale: Wheezing is a classic


urine, may occur normally during symptom of asthma. This statement
pregnancy; however, if it appears in the should alert the nurse to the possibility
urine, the client should be sent for that the woman's asthma is not being
testing to rule out gestational diabetes. well-controlled and needs further
Preeclampsia, anemia, and evaluation and possible intervention.
hyperthyroidism are not related to The other statements do not relate to
glucose nor to renal function. A slightly the typical presentation of this disease
elevated TSH would indicate possible in pregnancy.
hypothyroidism instead of
hyperthyroidism. Anemia would be 54. A 38-year-old woman comes into the
indicated by below normal hematocrit. If obstetrician's office for prenatal care,
the client's CBC is low normal than the stating that she is about 12 weeks
nurse should monitor future results to pregnant with her first child. What
ensure the client's counts are not questions would the nurse ask this
dropping. It would also be appropriate client, considering her age and potential
for the nurse to investigate possible sensitivity to being labeled an "older"
dietary issues. Preeclampsia would be primipara?
best monitored by the blood pressure Be nonjudgmental in your history
readings. gathering and offer her pregnancy
52. The maternal health nurse is caring resources to read and explore.
for a pregnant client with a history of
epilepsy. The client's antiepileptic drug Rationale: Women are having babies
(AED) levels have been in the later in life and nurses must be
non-therapeutic range the last two times supportive of their choices to postpone
the labs were drawn. Which factor does pregnancy. Most women realize the
the nurse associate with this finding? increased risks for having a baby after
35 years of age and don't need constant
reminding of all the potentially bad
outcomes that can occur. The majority
of pregnancies to women over 35 years 57. A 29-year-old client has gestational
of age end up with healthy babies and diabetes. The nurse is teaching her
mothers. about managing her glucose levels.
Which therapy would be most
55. A woman with cardiac disease at 32 appropriate for this client?
weeks' gestation reports she has been diet
having spells of light-headedness and
dizziness every few days. Which Rationale: Clients with gestational
instruction should the nurse prioritize? diabetes are usually managed by diet
Decrease activity and rest more often. alone to control their glucose
intolerance. Long-acting insulin usually
Rationale: If the client is developing is not needed for blood glucose control
symptoms associated with her heart in the client with gestational diabetes.
condition, the first intervention is to Oral hypoglycemic drugs are
monitor activity levels, decrease activity, contraindicated in pregnancy. Glucagon
and treat the symptoms. At 32 weeks' raises blood glucose and is used to treat
gestation, the suggestion to induce labor hypoglycemic reactions.
is not appropriate, and without
knowledge of the type of heart condition 58. A pregnant woman with a history of
one would not recommend an increase mitral valve stenosis is to be prescribed
of fluids or vitamins. Total bed rest may medication as treatment. Which
be required if the symptoms do not medication class would the nurse expect
resolve with decreased activity. the client to be prescribed?
anticoagulant
56. The nurse is preparing to teach a
pregnant client with iron deficiency Rationale: In mitral valve stenosis, it is
anemia about the various iron-rich foods difficult for blood to leave the left atrium.
to include in her diet. Which food should A secondary problem of thrombus
the nurse point out will help increase the formation may develop as a result of
absorption of her iron supplement? noncirculating blood. A woman may
Orange juice need to be prescribed an anticoagulant
to prevent this complication.
Rationale: Anemia is a condition in Vasodilators are used for peripartum
which the blood is deficient in red blood cardiomyopathy. Inotropics are used for
cells, from an underlying cause. The heart failure. Angiotensin receptor
woman needs to take iron to blockers are used for congestive heart
manufacture enough red blood cells. failure.
Taking an iron supplement will help
improve her iron levels, and taking iron 59. The nurse is caring for a pregnant
with foods containing ascorbic acid, woman with diabetes mellitus. Which
such as orange juice, improves the potential fetal complications should the
absorption of iron. Dried fruit (such as nurse monitor the client for as she
apples), fortified grains, and dried beans presents for her scheduled visits? Select
are additional food choices that are rich all that apply.
in iron and should be included in her congenital malformations
daily diet. macrosomia
respiratory disorder administered. Constipation is not usually
a result of sickle cell anemia.
Rationale: Potential problems during
pregnancy involving maternal diabetes 62. The clinic nurse teaches a
mellitus include fetal death, macrosomia pregestational type 1 diabetic client that
(oversized fetus), a fetus with a constant insulin levels are very
respiratory disorder, difficult labor, important during pregnancy. The nurse
preeclampsia or eclampsia, tells the client that the best way to
polyhydramnios, and congenital maintain a constant insulin level is to
malformations. use:
an insulin pump.
60. The nurse is assessing a pregnant
client who has a history of heart Rationale: Because a pregnant client
disease. The nurse will prioritize will have some periods of relative
assessments focusing on the heart hyperglycemia and hypoglycemia no
during which time frame? matter how carefully she maintains her
28 to 32 weeks' gestation diet and balances her exercise levels,
an effective method to keep serum
Rationale: A pregnant woman with glucose levels constant is to administer
heart disease is most vulnerable for insulin with a continuous pump during
cardiac decompensation from 28 to 32 pregnancy.
weeks' gestation, just after the blood
volume peaks. It would be important to 64. A client is diagnosed with peripartum
assess the client's heart at each visit; cardiomyopathy (PPCM). Which therapy
however, the client's heart would be would the nurse expect to administer to
more stressed at this time due to the the client?
increased blood volume and identifying restricted sodium intake
a serious situation early provides the
best opportunity for treatment and Rationale: The client with peripartum
preventing complications. cardiomyopathy should be prescribed a
restricted sodium intake to control the
61. A nurse is teaching a 30-year-old blood pressure. Monoamine oxidase
gravida 1 who has sickle cell anemia. inhibitors are given to treat depression
Providing education on which topic is in pregnancy, not peripartum
the highest nursing priority? cardiomyopathy. Methadone is a drug
avoidance of infection given for the treatment of a substance
use disorder during pregnancy.
Rationale: Prevention of crises, if Complementary therapies like ginger
possible, is the focus of treatment for therapy help in the alleviation of
the pregnant woman with sickle cell hyperemesis gravidarum, not peripartum
anemia. Maintaining adequate cardiomyopathy.
hydration, avoiding infection, getting
adequate rest, and eating a balanced 65. A pregnant woman with diabetes at
diet are all common-sense strategies 10 weeks' gestation has a glycosylated
that decrease the risk of a crisis. Fat hemoglobin (HbA1c) level of 13%. At
intake does not need to be decreased this time the nurse should be most
and immunoglobulins are not normally concerned about which possible fetal
outcome?
congenital anomalies 67. The health care provider of a newly
pregnant client determines the woman
Rationale: A HbA1c level of 13% also has mitral stenosis and will need
indicates poor glucose control. This, in appropriate therapy. Which medication
conjunction with the woman being in the should the nurse prepare to teach this
first trimester, increases the risk for client to provide her with the best
congenital anomalies in the fetus. possible care?
Elevated glucose levels are not heparin
associated with incompetent cervix,
placenta previa, or placental abruption Rationale: This client has an increased
(abruptio placentae). risk for developing blood clots. If an
anticoagulant is required, heparin is the
66. The nurse is assessing a drug of choice as it does not cross the
35-year-old woman at 22 weeks' placenta barrier. Warfarin crosses the
gestation who has had recent laboratory placenta and may have teratogenic
work. The nurse notes fasting blood effects. Aspirin is not recommended in
glucose 146 mg/dl (8.10 mmol/L), this situation. Digoxin is not used to
hemoglobin 13 g/dl (130 g/L), and prevent blood clots.
hematocrit 37% (0.37). Based on these
results, which instruction should the
nurse prioritize?
Check blood sugar levels daily.
68. The nurse is caring for a pregnant
Rationale: An elevated blood glucose is woman determined to be at high risk for
concerning for diabetes. A fasting blood gestational diabetes. The nurse
glucose level of greater than 140 mg/dl prepares to rescreen this client at which
(7.77 mmol/L) or random level of greater time frame?
than 200 mg/dl (11.10 mmol/L) is 24 to 28 weeks
concerning; this must be followed up to
ensure the client is not developing Rationale: A woman identified as high
gestational diabetes. The hemoglobin risk for gestational diabetes would
and hematocrit are within normal limits undergo rescreening between 24 and 28
for this client. The values should be weeks; however, some health care
hemoglobin greater than 11 g/dl (110 providers can choose to conduct this
g/L) and hematocrit greater than 33% screening earlier.
(0.33). Values lower than that are
possible indications of anemia and 69. A nurse is assessing a client in her
would necessitate further evaluation. An seventh month of pregnancy who has
individual with higher than normal blood an artificial valve prosthesis. The client
glucose levels is at risk for developing is taking an oral anticoagulant to prevent
urinary tract infection. This will usually the formation of clots at the valve site.
happen after the glucose levels are Which of the following nursing
elevated. Anemia can be treated by interventions is most appropriate in this
increasing the consumption of situation?
iron-enriched foods and taking a daily Observe the client for signs of petechiae
iron supplement. and premature separation of the
placenta
Rationale: Subclinical bleeding from Rationale: The most important goal
continuous anticoagulant therapy in the when caring for a pregnant client with
woman has the potential to cause pregestational diabetes is to maintain
placental dislodgement. Observe a glycemic control. The scenario does not
woman who is taking an anticoagulant give enough information on the client's
for signs of petechiae and signs of weight to determine if the client should
premature separation of the placenta, gain only minimal weight during
therefore, during both pregnancy and pregnancy. Ensuring compliance of
labor. The nurse should not urge the glucose monitoring and monitoring for
client to discontinue the anticoagulant, associated complications are
as this is not within the nurse's scope of appropriate nursing interventions;
practice and, in any case, the client still however, these do not take priority.
needs the anticoagulant to prevent clots.
Bed rest is prescribed for clients with a 72. A pregnant client with cystic fibrosis
thrombus, to prevent it from moving and (CF) comes to the office for a prenatal
becoming a pulmonary embolus. visit. She asks the nurse for information
Avoiding the use of constrictive on breast-feeding. The best response by
knee-high stockings is to prevent the nurse is:
thrombus formation. "Breast-feeding is not a good idea.
Because your breast milk is high in
70. Which changes in pregnancy would sodium due to CF, there is a risk of the
the nurse identify as a contributing infant receiving too much sodium."
factor for arterial thrombosis, especially
for the woman with atrial fibrillation? Rationale: The milk of a nursing mother
hypercoagulable state with cystic fibrosis is high in sodium.
This potentially places the infant at risk
Rationale: The nurse should identify for hypernatremia, that is, too much
that the increased risk of arterial sodium. Provide the client with as much
thrombosis in atrial fibrillation is due to correct information as possible, and
the hypercoagulable state of pregnancy. explain medical terms in layperson's
During pregnancy, there is a state of language.
hypercoagulation. This increases the 73. When providing nutritional
risk of arterial thrombosis in clients counseling to a pregnant woman with
having atrial fibrillation and artificial diabetes, the nurse would urge the client
valves. Increased cardiac output and to obtain most of her calories from which
blood volume do not cause arterial source?
thrombosis. Elevation of the diaphragm complex carbohydrates
is due to the uterine distension, and it
causes a shift in the QRS axis and is not Rationale: The pregnant woman with
associated with arterial thrombosis. diabetes is encouraged to eat three
meals a day plus three snacks, with
71. The nurse is caring for a pregnant 40% of calories derived from
client with pregestational diabetes. good-quality complex carbohydrates,
Which goal does the nurse identify as 35% of calories from protein sources,
priority during the client's pregnancy? and 35% of calories from unsaturated
Maintain glycemic control fats. The intake of saturated fats should
be limited during pregnancy, just as they
should be for any person to reduce the 76. A nurse informs a pregnant woman
risk of heart disease. with cardiac disease that she will need
two rest periods each day and a full
74. The nurse is assessing a pregnant night's sleep. The nurse further instructs
client who has a long history of asthma. the client that which position for this rest
She states, "I'm trying not to use my is best?
asthma medications because I certainly left lateral recumbent
don't want my baby exposed to them."
What is the nurse's best response? Rationale: The pregnant woman should
"Actually, having uncontrolled asthma is rest in the left lateral recumbent position
much riskier for your baby than the to prevent supine hypotension
medication." syndrome and increased heart effort.

Rationale: It is important for pregnant 77. A pregnant client with deep vein
clients with asthma to keep taking their thrombosis has been diagnosed as
medications because the risks of having systemic lupus erythematosus
exacerbations exceed the risks of the (SLE). The nurse would monitor the
medications. client closely for the development of
which complication?
75. A 17-year-old primigravida at 37 Fetal malnutrition
weeks' gestation has been unable to
maintain adequate control of her blood Rationale: SLE is an autoimmune
glucose throughout her pregnancy. The disorder in which there is deposition of
nurse should prioritize which action after immune complexes in the capillaries
the health care provider suspects the and visceral structures. Clients with SLE
infant has macrosomia based on the who become pregnant are at an
recent ultrasound? increased risk of fetal malnutrition due to
Prepare for assessment of fetal lung decreased placental circulation.
maturity. Pregnancy-related problems in SLE
include prematurity, stillbirth, decreased
Rationale: If the infant has macrosomia, placental weight and thinner placental
is large for gestational age, and the villi. In clients with SLE there is preterm
mother has had poor blood sugar birth and decreased placental weight.
control, the provider will want further Fetal macrosomia is seen in clients
information on the fetus and readiness having gestational diabetes, not SLE.
for delivery before making any decisions
on delivery. This will best be
accomplished by an amniocentesis to
assess the fetal lung maturity.
Scheduling an induction of labor,
allowing the patient to continue without
plans for delivery, or scheduling a
cesarean delivery at 39 weeks would Chapter 21
not be appropriate nursing actions.
Scheduling an induction or a cesarean 1. After spontaneous rupture of
section is not in the province of a nurse membranes, the nurse notices a
without a physician's order. prolapsed cord. The nurse
immediately places the woman in which Rationale: Risk factors for placental
position? abruption include preeclampsia,
A. supine gestational hypertension,
B. side-lying seizure activity, uterine rupture, trauma,
C. sitting smoking, cocaine use, coagulation
D. knee–chest defects, previous
history of abruption, intimate partner
Rationale: Pressure on the cord needs violence, and placental pathology.
to be relieved. Therefore, the nurse Macrosomia, gestational
would position the diabetes, and low parity are not
woman in a modified Sims, considered risk factors.
Trendelenburg, or knee–chest position.
Supine, side-lying, or sitting 4. A woman in labor is experiencing
would not provide relief of cord hypotonic uterine dysfunction.
compression. Assessment reveals no
fetopelvic disproportion. Which group of
2. A primigravida whose labor was medications would the nurse expect to
initially progressing normally is now administer?
experiencing a decrease A. sedatives
in the frequency and intensity of her B. tocolytics
contractions. The nurse would assess C. uterine stimulants
the woman for which D. corticosteroids
condition?
A. a low-lying placenta Rationale: For hypotonic labor, a
B. fetopelvic disproportion uterine stimulant such as oxytocin may
C. contraction ring be prescribed once fetopelvic
D. uterine bleeding disproportion is ruled out. Sedatives
might be helpful for the woman with
Rationale: The woman is experiencing hypertonic
dystocia most likely due to hypotonic uterine contractions to promote rest and
uterine dysfunction relaxation. Tocolytics would be ordered
and fetopelvic disproportion associated to control preterm labor. Corticosteroids
with a large fetus. A low-lying placenta, may be given to enhance fetal lung
contraction ring, maturity for women
or uterine bleeding would not be experiencing preterm labor.
associated with a change in labor
pattern. 5. A woman gave birth to a newborn via
vaginal birth with the use of a vacuum
3. Which assessment finding will alert extractor. The nurse would be alert for
the nurse to be on the lookout for which possible effect in the newborn?
possible placental A. asphyxia
abruption during labor? B. clavicular fracture
A. macrosomia C. cephalhematoma
B. gestational hypertension D. central nervous system injury
C. gestational diabetes
D. low parity Rationale: Use of forceps or a vacuum
extractor poses the risk of tissue
trauma, such as
ecchymoses, facial and scalp B. shoulder dystocia
lacerations, facial nerve injury, C. uterine rupture
cephalhematoma, and caput D. umbilical cord prolapse
succedaneum. Asphyxia may be related
to numerous causes, but it is not Rationale: Uterine rupture is associated
associated with use of a with crack cocaine use disorder.
vacuum extractor. Clavicular fracture is Generally, the first and
associated with shoulder dystocia. most reliable sign is sudden fetal
Central nervous system injury is not distress accompanied by acute
associated with the use of a vacuum abdominal pain, vaginal
extractor. bleeding, hematuria, irregular wall
contour, and loss of station in the fetal
6. A pregnant client undergoing labor presenting part.
induction is receiving an oxytocin Amniotic fluid embolism often is
infusion. Which finding manifested with a sudden onset of
would require immediate intervention? respiratory distress.
A. fetal heart rate of 150 beats/minute Shoulder dystocia is noted when
B. contractions every 2 minutes, lasting continued fetal descent is obstructed
45 seconds after the fetal head is
C. uterine resting tone of 14 mm Hg delivered. Umbilical cord prolapse is
D. urine output of 20 mL/hour noted as the protrusion of the cord
alongside or ahead of
Rationale: Oxytocin can lead to water the presenting part of the fetus.
intoxication. Therefore, a urine output of
20 mL/hour is 8. A woman receives magnesium sulfate
below acceptable limits of 30 mL/hour as treatment for preterm labor. The
and requires intervention. FHR of 150 nurse assess and
beats/minute is maintains the infusion at the prescribed
within the accepted range of 120 to 160 rate based on which finding?
beats/minute. Contractions should occur A. Respiratory rate-16 breaths/minute
every 2 to 3 B. Decreased fetal heart rate variability
minutes, lasting 40 to 60 seconds. A C. Urine output 22 mL/hour
uterine resting tone greater than 20 mm D. Absent deep tendon reflexes
Hg would require
intervention. Rationale: A respiratory rate of 16
breaths per minute is appropriate and
7. A woman with a history of crack within acceptable
cocaine use disorder is admitted to the parameters to continue the infusion.
labor and birth area. When administering magnesium sulfate,
While caring for the client, the nurse the nurse would
notes a sudden onset of fetal immediately report decreaed fetal heart
bradycardia. Inspection of the rate variability, a urine output less than
abdomen reveals an irregular wall 30 mL/hour, and
contour. The client also reports acute decreased or absent deep tendon
abdominal pain that is reflexes.
continuous. Which condition would the
nurse suspect?
A. amniotic fluid embolism
9. The nurse is reviewing the physical C. "I guess the nurses will be getting me
examination findings for a client who is up and out of bed rather quickly."
to undergo labor D. "I'll probably have a tube in my
induction. Which finding would indicate bladder for about 24 hours or so."
to the nurse that a woman's cervix is
ripe in preparation Rationale: Typically, breastfeeding is
for labor induction? initiated early as soon as possible after
A. posterior position birth to promote bonding. The woman
B. firm may need to use alternate positioning
C. closed techniques to reduce incisional
D. shortened discomfort. Splinting with pillows helps
to reduce the discomfort associated with
Rationale: A ripe cervix is shortened, coughing. Early ambulation is
centered (anterior), softened, and encouraged to prevent respiratory and
partially dilated. An unripe cervix is long, cardiovascular problems and promote
closed, posterior, and firm. peristalsis. An indwelling urinary
catheter is typically inserted to drain the
10. A woman with preterm labor is bladder. It usually
receiving magnesium sulfate. Which remains in place for approximately 24
finding would require hours.
the nurse to intervene immediately? 12. The nurse is providing care to
A. respiratory rate of 16 breaths per several pregnant women who may be
minute scheduled for labor
B. 1+ deep tendon reflexes induction. The nurse identifies the
C. urine output of 45 mL/hour woman with which Bishop score as
D. alert level of consciousness having the best chance for
a successful induction and vaginal birth?
Rationale: Diminished deep tendon A. 11
reflexes (1+) suggest magnesium B. 7
toxicity, which requires C. 5
immediate intervention. Additional signs D. 3
of magnesium toxicity include a
respiratory rate less Rationale: The Bishop score helps
than 12 breaths/minute, urine output identify women who would be most
less than 30 mL/hour, and a decreased likely to achieve a
level of successful induction. The duration of
consciousness. labor is inversely correlated with the
Bishop score: a score
11. After teaching a couple about what over 8 indicates a successful vaginal
to expect with their planned cesarean birth. Therefore the woman with a
birth, which Bishop score of 11
statement indicates the need for would have the greatest chance for
additional teaching? success. Bishop scores of less than 6
A. "Holding a pillow against my incision usually indicate that a
will help me when I cough." cervical ripening method should be used
B. "I'm going to have to wait a few days prior to induction.
before I can start breastfeeding."
13. A nurse is reviewing the medical 15. A nurse is conducting a review
record of a pregnant client. The nurse course on tocolytic therapy for perinatal
suspects that the nurses. After
client may be at risk for dystocia based teaching the group, the nurse
on which factors? Select all that apply. determines that the teaching was
A. plan for pudendal block anesthetic successful when they identify
use which drugs as being used for tocolysis?
B. multiparity Select all that apply.
C. short maternal stature A. nifedipine
D. Body mass index 30.2 B. magnesium sulfate
E. breech fetal presentation C. dinoprostone
D. misoprostol
Rationale: Risk factors for dystocia E. indomethacin
may include maternal short stature,
obesity, hydramnios, Rationale: Medications most commonly
uterine abnormalities, fetal used for tocolysis include magnesium
malpresentation, cephalopelvic sulfate (which
disproportion, overstimulation with reduces the muscle's ability to contract),
oxytocin, maternal exhaustion, indomethacin (a prostaglandin
ineffective pushing, excessive size fetus, synthetase inhibitor), and
poor maternal nifedipine (a calcium channel blocker).
positioning in labor, and maternal These drugs are used "off label": this
anxiety and fear means they are
effective for this purpose but have not
14. A nurse is preparing an inservice been officially tested and developed for
education program for a group of nurses this purpose by
about dystocia the FDA. Dinoprostone and misoprostol
involving problems with the passenger. are used to ripen the cervix.
Which problem would the nurse likely 16. A nurse is teaching a pregnant
include as the woman at risk for preterm labor about
most common? what to do if she
A. macrosomia experiences signs and symptoms. The
B. breech presentation nurse determines that the teaching was
C. persistent occiput posterior position successful when
D. multifetal pregnancy the woman makes which statement?
A. "I'll sit down to rest for 30 minutes."
Rationale: Common problems involving B. "I'll try to move my bowels."
the passenger include occiput posterior C. "I'll lie down with my legs raised."
position, breech D. "I'll drink several glasses of water."
presentation, multifetal pregnancy,
excessive size (macrosomia) as it Rationale: If the woman experiences
relates to cephalopelvic any signs and symptoms of preterm
disproportion (CPD), and structural labor, she should stop
anomalies. Of these, persistent occiput what she is doing and rest for 1 hour,
posterior is the most empty her bladder, lie down on her side,
common malposition, occurring in about drink two to three
15% of laboring women.
glasses of water, feel her abdomen and 18. A nurse is explaining to a group of
note the hardness of the contraction, nurses new to the labor and birth unit
and call her health about methods used
care provider and describe the for cervical ripening. The group
contraction. demonstrates understanding of the
information when they
17. A nurse is describing the risks identify which method as a mechanical
associated with post-term pregnancies one?
as part of an inservice A. herbal agents
presentation. The nurse determines B. laminaria
thatmore teaching is needed when the C. membrane stripping
group identifies which D. amniotomy
factor as an underlying reason for
problems in the fetus? Rationale: Laminaria is a hygroscopic
A. aging of the placenta dilator that is used as a mechanical
B. increased amniotic fluid volume method for cervical
C. meconium aspiration ripening. Herbal agents are a
D. cord compression nonpharmacologic method. Membrane
stripping and amniotomy
Rationale: Fetal risks associated with a are considered surgical methods.
post-term pregnancy include
macrosomia, shoulder 19. The nurse notifies the obstetrical
dystocia, brachial plexus injuries, low team immediately because the nurse
Apgar scores, postmaturity syndrome suspects that the
(loss of subcutaneous fat and muscle pregnant woman may be exhibiting
and meconium staining), and signs and symptoms of amniotic fluid
cephalopelvic disproportion. As the embolism. When
placenta ages, its perfusion decreases reporting this suspicion, which finding(s)
and it becomes less efficient at would the nurse include in the report?
delivering oxygen and Select all that apply.
nutrients to the fetus. Amniotic fluid A. significant difficulty breathing
volume also begins to decline after 38 B. hypertension
weeks’ gestation, C. tachycardia
possibly leading to oligohydramnios, D. pulmonary edema
subsequently resulting in fetal hypoxia E. bleeding with bruising
and an increased
risk of cord compression because the Rationale: Anaphylactoid syndrome of
cushioning effect offered by adequate pregnancy (ASP), also known as
fluid is no longer amniotic fluid embolism, is an
present. Hypoxia and oligohydramnios unforeseeablelife-threatening
predispose the fetus to aspiration of complication of childbirth. The etiology
meconium, which is of ASP
released by the fetus in response to a remains an enigmatic, devastating
hypoxic insult (Norwitz, 2019). All of obstetric condition associated with
these issues can significant maternal and
compromise fetal well-being and lead to newborn morbidity and mortality. It is a
fetal distress. rare and often fatal event characterized
by the sudden
onset of hypotension, cardiopulmonary 21. A pregnant woman is receiving
collapse, hypoxia, and coagulopathy. misoprostol to ripen her cervix and
ASP should be induce labor. The nurse
suspected in any pregnant women with assesses the woman closely for which
an acute onset of dyspnea, hypotension, effect?
and DIC. By A. uterine hyperstimulation
knowing how to intervene, the nurse can B. headache
promote a better chance of survival for C. blurred vision
both the mother D. hypotension
and her newborn.
Rationale: A major adverse effect of the
20. A nurse is conducting an in-service obstetric use of misoprostol is
program for a group of labor and birth hyperstimulation of the
unit nurses about uterus, which may progress to uterine
cesarean birth. The group demonstrates tetany with marked impairment of
understanding of the information when uteroplacental blood
they identify which conditions as flow, uterine rupture (requiring surgical
appropriate indications? Select all that repair, hysterectomy, and/or
apply. salpingo-oophorectomy), or
A. active genital herpes infection amniotic fluid embolism. Headache,
B. placenta previa blurred vision, and hypotension are
C. previous cesarean birth associated with
D. prolonged labor magnesium sulfate.
E. fetal distress
22. A nurse is teaching a woman about
Rationale: The leading indications for measures to prevent preterm labor in
cesarean birth are previous cesarean future pregnancies
birth, breech because the woman just experienced
presentation, dystocia, and fetal preterm labor with her most recent
distress. Examples of specific pregnancy. The nurse
indications include active genital determines that the teaching was
herpes, fetal macrosomia, fetopelvic successful based on which statement by
disproportion, prolapsed umbilical cord, the woman?
placental A. "I'll make sure to limit the amount of
abnormality (placenta previa or long distance traveling I do."
placental abruption), previous classic B. "Stress isn't a problem that is related
uterine incision or scar, to preterm labor."
gestational hypertension, diabetes, C. "Separating pregnancies by about a
positive human immunodeficiency virus year should be helpful."
(HIV) status, and D. "I'll need extra iron in my diet so I
dystocia. Fetal indications include have extra for the baby."
malpresentation (nonvertex
presentation), congenital Rationale: Appropriate measures to
anomalies (fetal neural tube defects, reduce the risk for preterm labor include:
hydrocephalus, abdominal wall defects), avoiding travel for
and fetal distress. long distances in cars, trains, planes or
buses; achieving adequate iron store
through balanced
nutrition (excess iron is not necessary);
waiting for at least 18 months between
pregnancies, and using stress
management techniques for stress. 25. A nurse is reading a journal article
about cesarean births and the
23. A pregnant woman at 31-weeks' indications for them. Place
gestation calls the clinic and tells the the indications for cesarean birth below
nurse that she is having contractions in the proper sequence from most
sporadically. Which instructions would frequent to least
be most appropriate for the nurse to frequent. All options must be used.
give the woman? Select all that apply. A. Labor dystocia
A. "Walk around the house for the next B. Abnormal fetal heart rate tracing
half hour." C. Fetal malpresentation
B. "Drink two or three glasses of water." D. Multiple gestation
C. "Lie down on your back." E. Suspected macrosomia
D. "Try emptying your bladder." Answer: A, B, C, D, E
E. "Stop what you are doing and rest."
Rationale: The most common
Rationale: Appropriate instructions for indications for primary cesarean births
the woman who may be experiencing include, in order of
preterm labor frequency: labor dystocia as the labor
include having the client stop what she does not progress, abnormal fetal heart
is doing and rest for an hour, empty her rate tracing
bladder, lie down indicating fetal distress, fetal
on her left side, and drink two to three malpresentation making a difficult
glasses of water. progression of labor, multiple
gestation , and suspected macrosomia.
24. A pregnant client has received
dinoprostone. Following administration 26. A nurse is taking a history on a
of this medication, the woman who is at 20 weeks' gestation.
nurse assesses the client and The woman reports that she feels some
determines that the client is heaviness in her thighs since yesterday.
experiencing an adverse effect of the The nurse suspects that the woman
medication based on which client may be experiencing preterm labor
report? Select all that apply. based on which additional assessment
A. headache findings? Select all
B. nausea that apply.
C. diarrhea A. dull low backache
D. tachycardia B. viscous vaginal discharge
E. hypotension C. dysuria
D. constipation
Rationale: Adverse effects associated E. occasional cramping
with dinoprostone include headache,
nauseas and Rationale: Symptoms of preterm labor
vomiting, and diarrhea. Tachycardia and are often subtle and may include
hypotension are not associated with this change or increase in
drug.
vaginal discharge with mucus, water, or suggests fetal demise, but an ultrasound
blood in it; pelvic pressure; low, dull is necessary to confirm the absence of
backache; nausea, fetal cardiac
vomiting or diarrhea, and heaviness or activity. Once fetal demise is confirmed,
aching in the thighs. Constipation is not induction of labor or expectant
known to be a management is offered
sign of preterm labor. Preterm labor is to the client. An amniocentesis, hCG
assessed when there are more than six level, or triple marker screening would
contractions per not be used to
hour. Occasional asymptomatic confirm IUFD.
cramping can be normal.

27. A pregnant client at 24 weeks'


gestation comes to the clinic for an
evaluation. The client 28. A 32-year-old black woman in her
called the clinic earlier in the day stating second trimester has come to the clinic
that she had not felt the fetus moving for an evaluation.
since yesterday While interviewing the client, she reports
evening. Further assessment reveals a history of fibroids and urinary tract
absent fetal heart tones. Intrauterine infection. The client states, "I know
fetal demise is smoking is bad and I have tried to stop,
suspected. The nurse would expect to but it is impossible. I have cut
prepare the client for which testing to down quite a bit though, and I do not
confirm the drink alcohol." Complete blood count
suspicion? results reveal a low
A. Ultrasound red blood cell count, low hemoglobin,
B. Amniocentesis and low hematocrit. When planning this
C. Human chorionic gonadotropin (hCG) client's care, which factor(s) would the
level nurse identify as increasing the client's
D. Triple marker screening risk for preterm labor? Select
all that apply.
Rationale: A client experiencing an A. African heritage
intrauterine fetal demise (IUFD) is likely B. Maternal age
to seek care when C. History of fibroids
she notices that the fetus is not moving D. Cigarette smoking
or when she experiences contractions, E. History of urinary tract infections
loss of fluid, or F. Complete blood count results
vaginal bleeding. History and physical
examination frequently are of limited Rationale: For this client, risk factors
value in the associated with preterm labor and birth
diagnosis of fetal death, since many would include
times the only history tends to be recent African heritage, cigarette smoking,
absence of fetal uterine abnormalities, such as fibroids,
movement and no fetal heart beat urinary tract
heard. An inability to obtain fetal heart infection, and possible anemia based on
sounds on examination her complete blood count results.
Maternal age extremes
(younger than 16 years and older than pain; more than six contractions per
35 years) are also a risk factor but do hour; intestinal cramping with or without
not apply to this diarrhea.
client. Contractions also must be persistent,
such that four contractions occur every
29. A pregnant client at 30 weeks' 20 minutes or eight
gestation calls the clinic because she contractions occur in 1 hour. A report of
thinks that she may be in heartburn is unrelated to preterm labor.
labor. To determine if the client is
experiencing labor, which question(s) 30. When describing the stages of labor
would be appropriate to a pregnant woman, which would the
for the nurse to ask? Select all that nurse identify as
apply. the major change occurring during the
A. "Are you feeling any pressure or first stage?
heaviness in your pelvis?" A. Regular contractions
B. "Are you having contractions that B. Cervical dilation
come and go, off and on?" C. Fetal movement through the birth
C. "Have you noticed any fluid leaking canal
from your vagina?" D. Placental separation
D. "Are you having problems with
heartburn?" Rationale: The primary change
E. "Have you been having any nausea occurring during the first stage of labor
or vomiting?" is progressive cervical
dilation. Contractions occur during the
Rationale: Frequently, women are first and second stages of labor. Fetal
unaware that uterine contractions, movement through the birth canal is the
effacement, and dilation are major change during the second stage
occurring, thus making early intervention of labor. Placental separation occurs
ineffective in arresting preterm labor and during the third stage of labor.
preventing the birth of a premature
newborn. The nurse should ask the 31. The nurse is appraising the medical
client about any signs/symptoms, record of a pregnant client who is
being alert for subtle symptoms of resting in a darkened room and
preterm labor, which may include: a receiving oxytocin and magnesium
change or increase in sulfate. The nurse will continue to
vaginal discharge with mucous, water, monitor this client for progression to
or blood in it; pelvic pressure which condition?
(pushing-down sensation); eclampsia
low dull backache; menstrual-like
cramps; urinary tract infection Rationale: This woman is in severe
symptoms; feeling of pelvic preeclampsia and must be monitored for
pressure or fullness; gastrointestinal progression to eclampsia. The
upset like nausea, vomiting, and administration of magnesium sulfate is
diarrhea; general sense of to relax the skeletal muscles and raise
discomfort or unease; heaviness or the threshold for a seizure. The
aching in the thighs; uterine contractions administration of oxytocin is to stimulate
with or without uterine contractions to hasten birth. The
client has already progressed from mild
preeclampsia to severe preeclampsia, Onset of vaginal bleeding was sudden
and the nurse need to follow measures and painful
to prevent advancement of the disease
process. Although preeclampsia results Rationale: Sudden onset of abdominal
in a high blood pressure, the scenario pain and vaginal bleeding with a rigid
described does not indicate a client with uterus that does not relax are signs of a
hypertension. placental abruption (abruptio placentae).
The other findings are consistent with a
32. A client reporting she recently had a diagnosis of placenta previa.
positive pregnancy test has reported to
the emergency department stating 34. The nurse is caring for a woman at
one-sided lower abdominal pain. The 32 weeks' gestation with severe
health care provider has prescribed a preeclampsia. Which assessment
series of tests. Which test will provide finding should the nurse prioritize after
the most definitive confirmation of an the administration of hydralazine to this
ectopic pregnancy? client?
Abdominal ultrasound Tachycardia

Rationale: An ectopic pregnancy refers Rationale: Hydralazine reduces blood


to the implantation of the fertilized egg in pressure but is associated with adverse
a location other than the uterus. effects such as palpitation, tachycardia,
Potential sites include the cervix, uterus, headache, anorexia, nausea, vomiting,
abdomen, and fallopian tubes. The and diarrhea. It does not cause
confirmation of the ectopic pregnancy gastrointestinal bleeding, blurred vision
can be made by an ultrasound, which (halos around lights), or sweating.
would confirm that there was no uterine Magnesium sulfate may cause
pregnancy. A quantitative hCG level sweating.
may be completed in the diagnostic
plan. hCG levels in an ectopic 35. A 24-year-old client presents in
pregnancy are traditionally reduced. labor. The nurse notes there is an order
While this would be an indication, it to administer Rho(D) immune globulin
would not provide a positive after the birth of her infant. When asked
confirmation. The qualitative hCG test by the client the reason for this injection,
would provide evidence of a pregnancy, which reason should the nurse point
but not the location of the pregnancy. A out?
pelvic exam would be included in the prevent maternal D antibody formation.
diagnostic plan of care. It would likely
show an enlarged uterus and cause Rationale: Because Rho(D) immune
potential discomfort to the client but globulin contains passive antibodies, the
would not be a definitive finding. solution will prevent the woman from
forming long-lasting antibodies which
33. A woman at 34 weeks' gestation may harm a future fetus. The
presents to labor and delivery with administration of Rho(D) immune
vaginal bleeding. Which finding from the globulin does not promote the formation
obstetric examination would lead to a of maternal D antibodies; it does not
diagnosis of placental abruption stimulate maternal D immune antigens
(abruptio placentae)? or prevent fetal Rh blood formation.
36. A nurse in the maternity triage unit is Rationale: The woman is at risk for
caring for a client with a suspected magnesium toxicity. The antidote for
ectopic pregnancy. Which nursing magnesium sulfate is calcium gluconate,
intervention should the nurse perform and this should be readily available in
first? case the woman has signs and
Assess the client's vital signs. symptoms of magnesium toxicity.

Rationale: A suspected ectopic 39. A client is diagnosed with


pregnancy can put the client at risk for gestational hypertension and is
hypovolemic shock. The assessment of receiving magnesium sulfate. The nurse
vital signs should be performed first, determines that the medication is at a
followed by any procedures to maintain therapeutic level based on which
the ABCs. Providing emotional support finding?
would also occur, as would obtaining a deep tendons reflexes 2+
surgical consent, if needed, but these
are not first steps. Rationale: With magnesium sulfate,
deep tendon reflexes of 2+ would be
37. A pregnant client at 32 weeks' considered normal and therefore a
gestation is treated with magnesium therapeutic level of the drug. Urinary
sulfate for seizure management. The output of less than 30 mL, a respiratory
nurse assesses which of the following rate of less than 12 breaths/minute, and
for evidence of magnesium toxicity? a diminished level of consciousness
Absence of knee jerk response would indicate magnesium toxicity.

Rationale: Magnesium sulfate toxicity is 40. A woman at 8 weeks' gestation is


characterized by absence of deep admitted for ectopic pregnancy. She is
tendon reflexes like the knee jerk reflex. asking why this has occurred. The nurse
Urinary retention, and not frequency of knows that which factor is a known risk
micturition, is seen with magnesium factor for ectopic pregnancy?
sulfate toxicity. Magnesium sulfate is use of IUD for contraception
given to treat seizures associated with
hypertension and proteinuria in Rationale: Use of an IUD with
pregnancy, and therefore decreases the progesterone has a known increased
blood pressure. It does not cause an risk for development of ectopic
increase in blood pressure. There is pregnancies. The nurse needs to
respiratory depression, and not an complete a full history of the client to
increased rate of respiration, with determine if she had any other risk
magnesium sulfate toxicity. factors for an ectopic pregnancy.
Adhesions, scarring, and narrowing of
the tubal lumen may block the zygote's
38. A client with severe preeclampsia is progress to the uterus. Any condition or
receiving magnesium sulfate as part of surgical procedure that can injure a
the treatment plan. To ensure the client's fallopian tube increases the risk.
safety, which compound would the nurse Examples include salpingitis, infection of
have readily available? the fallopian tube, endometriosis, history
calcium gluconate of prior ectopic pregnancy, any type of
tubal surgery, congenital malformation of
the tube, and multiple elective abortions.
Conditions that inhibit peristalsis of the when developing magnesium toxicity
tube can result in tubal pregnancy. may be a decrease in reflex activity. The
Hormonal factors may play a role health care provider needs to be notified
because tubal pregnancy occurs more immediately. A change in lung sounds
frequently in women who take fertility and oxygen saturation are not indicative
drugs or who use progesterone of magnesium sulfate toxicity. Hourly
intrauterine contraceptive devices blood draws to gain information on the
(IUDs). A high number of pregnancies, magnesium sulfate level are not
multiple gestation pregnancy, and the indicated.
use of oral contraceptives are not known
risk factors for ectopic pregnancy. 43. A client with a molar pregnancy is
scheduled for a dilatation and curettage
41. The nurse is appraising the medical (D&C). The nurse is educating that
record of a pregnant client who is client about postoperative care. The
resting in a darkened room and nurse will instruct the client that she will
receiving betamethasone and have the first serum hCG level drawn at
magnesium sulfate. The nurse how many hours after the D&C?
recognizes the client is being treated for 48
which condition?
severe preeclampsia Rationale: A serum hCG level is drawn
48 hours after the procedure to assess if
Rationale: This woman is in severe the level is decreasing. Drawing it earlier
preeclampsia and must be monitored for does not allow adequate time for the
progression to eclampsia. The body to begin clearing hCG after the
administration of magnesium sulfate is uterus is evacuated. Drawing it much
to relax the skeletal muscles and raise later may mask the effectiveness of the
the threshold for a seizure. The D&C procedure.
administration of the betamethasone is
to try and hasten the maturity of the 44. A primigravida 28-year-old client is
fetus's lungs for birth for a preterm fetus. noted to have Rh negative blood and
The scenario described does not her husband is noted to be Rh positive.
indicate a client with hypertension, The nurse should prepare to administer
gestational diabetes, or post-term (>42 RhoGAM after which diagnostic
weeks) pregnancy. procedure?
Amniocentesis
42. The following hourly assessments
are obtained by the nurse on a client Rationale: Amniocentesis is a
with preeclampsia receiving magnesium procedure requiring a needle to enter
sulfate: 97.3oF (36.2oC), HR 88, RR 12 into the amniotic sac. There is a risk of
breaths/min, BP 148/110 mm Hg. What mixing of the fetal and maternal blood
other priority physical assessments by which could result in blood
the nurse should be implemented to incompatibility. A contraction test, a
assess for potential toxicity? nonstress test, and biophysical profile
Reflexes are not invasive, so there would be no
indication for Rho(D) immune globulin to
Rationale: Reflex assessment is part of be administered.
the standard assessment for clients on
magnesium sulfate. The first change
45. A client with a history of cervical avoid pregnancy for 1 year because it
insufficiency is seen for reports of can interfere with the monitoring of hCG
pink-tinged discharge and pelvic levels. Iron intake and blood pressure
pressure. The primary care provider are not important aspects of follow up
decides to perform a cervical cerclage. after evacuation of a hydatiform mole.
The nurse teaches the client about the Use of a reliable contraceptive is
procedure. Which client response strongly recommended so that
indicates that the teaching has been pregnancy is avoided.
effective?
"Purse-string sutures are placed in the 48. The nurse is caring for a client who
cervix to prevent it from dilating." has a multifetal pregnancy. What topic
should the nurse prioritize during health
Rationale: The cerclage, or purse string education?
suture, is inserted into the cervix to Signs of preterm labor
prevent preterm cervical dilation
(dilatation) and pregnancy loss. Staples, Rationale: The client with a multifetal
glue, or a cervical cap will not prevent pregnancy must be made aware of the
the cervix from dilating. risks posed by preterm labor. There is
no corresponding increase in the risk for
46. A nurse suspects that a client is hypertension or blood incompatibilities.
developing HELLP syndrome. The Parenting skills are secondary to
nurse notifies the health care provider physiologic needs at this point.
based on which finding?
elevated liver enzymes 49. A woman at 31 weeks' gestation
presents to the emergency department
Rationale: HELLP is an acronym for with bright red vaginal bleeding,
hemolysis, elevated liver enzymes, and reporting that the onset of the bleeding
low platelets. Hyperglycemia is not a was sudden and without pain. Which
part of this syndrome. HELLP may diagnostic test should the nurse
increase the woman's risk for DIC but it prioritize?
is not an assessment finding. A transvaginal ultrasound

47. After teaching a woman who has Rationale: For any pregnant woman
had an evacuation for gestational who presents with painless bleeding,
trophoblastic disease (hydatidiform mole placenta previa needs to be ruled out by
or molar pregnancy) about her either transvaginal or abdominal
condition, which statement indicates that ultrasound. A digital cervical exam is
the nurse's teaching was successful? contraindicated until placenta previa has
"I will be sure to avoid getting pregnant been ruled out, as digital manipulation of
for at least 1 year." placental tissue through the cervical os
can cause uncontrollable bleeding. The
Rationale: After evacuation of nonstress test and electronic fetal
trophoblastic tissue (hydatiform mole), monitoring would be utilized after the
long-term follow-up is necessary to woman is stabilized and placenta previa
make sure any remaining trophoblastic has either been diagnosed or ruled out.
tissue does not become malignant.
Serial hCG levels are monitored closely 50. A 28-year-old client with a history of
for 1 year, and the client is urged to endometriosis presents to the
emergency department with severe weeks and has symptoms (blurred
abdominal pain and nausea and vision) of preeclampsia. She also had
vomiting. The client also reports her an increase of protein in her urine (2+)
periods are irregular with the last one and a 15% increase in her BP. The
being 2 months ago. The nurse nurse will need more information to
prepares to assess for which possible determine if the 38-week client may be
cause for this client's complaints? in the early stages of labor, and if the
Ectopic pregnancy 31-week client with flank pain has a
kidney infection. The client with malaise
Rationale: The most commonly and rhinitis will need to talk to the nurse
reported symptoms of ectopic last to find out what over-the-counter
pregnancy are pelvic pain and/or vaginal medication she is able to take.
spotting. Other symptoms of early
pregnancy, such as breast tenderness, 51. A woman at 35 weeks' gestation
nausea, and vomiting, may also be with severe hydramnios is admitted to
present. The diagnosis is not always the hospital. The nurse recognizes that
immediately apparent because many which concern is greatest regarding this
women present with complaints of client?
diffuse abdominal pain and minimal to preterm rupture of membranes followed
no vaginal bleeding. Steps are taken to by preterm birth
diagnose the disorder and rule out other
causes of abdominal pain. Given the Rationale: Even with precautions, in
history of the client and the amount of most instances of hydramnios, there will
pain, the possibility of ectopic pregnancy be preterm rupture of the membranes
needs to be considered. A healthy because of excessive pressure, followed
pregnancy would not present with by preterm birth. The other answers are
severe abdominal pain unless the client less concerning than preterm birth in
were term and she was in labor. With a this pregnancy.
molar pregnancy the woman typically
presents between 8 to 16 weeks' 52. Which medication will the nurse
gestation reporting painless (usually) anticipate the health care provider will
brown to bright red vaginal bleeding. prescribe as treatment for an unruptured
Placenta previa typically presents with ectopic pregnancy?
painless, bright red bleeding that begins methotrexate
with no warning.
Rationale: Methotrexate, a folic acid
50. The nurse is transcribing messages antagonist that inhibits cell division in
from the answering service. Which the developing embryo, is most
phone message should the nurse return commonly used to treat ectopic
first? pregnancy. Oxytocin is used to stimulate
a 35-year-old, 21-week G3P2 client with uterine contractions and would be
blood pressure of 160/110 mm Hg, inappropriate for use with an ectopic
blurred vision, and whose last blood pregnancy. Promethazine and
pressure was 143/99 mm Hg and urine ondansetron are antiemetics that may
dipstick showed a +2 proteinuria be used to treat hyperemesis
gravidarum.
Rationale: The nurse should call the
at-risk 35-year-old client first. She is 21
53. Current research indicates that only 10 weeks' pregnant and early
supplementation with what before miscarriage occurs before 12 weeks.
pregnancy may reduce the risk of
placental abruption? 56. A client at 27 weeks' gestation is
Folic acid admitted to the obstetric unit after
reporting headaches and edema of her
Rationale: New research indicates that hands. Review of the prenatal notes
folic acid supplementation before or reveals blood pressure consistently
during pregnancy reduces the risk of above 136/90 mm Hg. The nurse
placental abruption. Neither anticipates the health care provider will
supplementation with vitamin C, iron, prescribe magnesium sulfate to
nor calcium is associated with a accomplish which primary goal?
decreased risk for placental abruption. Prevent maternal seizures

54. A client at 25 weeks' gestation Rationale: The primary therapy goal for
presents with a blood pressure of any client with preeclampsia is to
152/99 mm Hg, pulse 78 beats/min, no prevent maternal seizures. Use of
edema, and urine negative for protein. magnesium sulfate is the drug therapy
What would the nurse do next? of choice for severe preeclampsia and is
Notify the health care provider only used to manage and attempt to
prevent progression to eclampsia.
Rationale: The client is exhibiting a sign Magnesium sulfate therapy does not
of gestational hypertension, elevated have as a primary goal of decreasing
blood pressure greater than or equal to blood pressure, decreasing protein in
140/90 mm Hg that develops for the first the urine, or reversing edema.
time during pregnancy. The health care
provider should be notified to assess the 57. A nurse is reviewing a client's history
client. Without the presence of edema or and physical examination findings.
protein in the urine, the client does not Which information would the nurse
have preeclampsia. identify as contributing to the client's risk
55. A woman in her 20s has for an ectopic pregnancy?
experienced a spontaneous abortion recurrent pelvic infections
(miscarriage) at 10 weeks' gestation and
asks the nurse at the hospital what went Rationale: In the general population,
wrong. She is concerned that she did most cases of ectopic pregnancy are the
something that caused her to lose her result of tubal scarring secondary to
baby. The nurse can reassure the pelvic inflammatory disease. Oral
woman by explaining that the most contraceptives, ovarian cysts, and
common cause of miscarriage in the first heavy, irregular menses are not
trimester is related to which factor? considered risk factors for ectopic
Chromosomal defects in the fetus pregnancy.

Rationale: Fetal factors are the most 58. A client whose membranes have
common cause of early miscarriages, prematurely ruptured is admitted to the
with chromosomal abnormalities in the hospital. Which nursing intervention is a
fetus being the most common reason. priority?
This client fits the criteria for early Routine monitoring of vital signs
spontaneous abortion since she was
Rationale: Rupture of the membranes should be evaluated carefully. It may be
without the onset of labor places the life-threatening or it may be something
woman at risk for infection. The priority that is not a threat to the mother and/or
is to monitor temperature routinely by fetus. Regardless, it needs to be
the completion of vital signs. Antibiotic evaluated quickly and carefully. Telling
therapy is often initiated as well, the client it may be harmless is a
depending upon closeness of labor reassuring statement, but does not
initiation (naturally or induced). The suggest the need for urgent evaluation.
fetus will be monitored on a regular Having the mother lay on her left side
basis and then continuously when the and drink water is indicated for
labor process occurs. Urine analysis cramping.
and strict intake and output are not
typically completed. 61. A nurse is taking a history of a client
at 5 weeks' gestation in the prenatal
59. A pregnant client diagnosed with clinic; however, the client is reporting
hyperemesis gravidarum is prescribed dark brown vaginal discharge, nausea,
intravenous fluids for rehydration. When and vomiting. Which diagnosis should
preparing to administer this therapy, the nurse suspect?
which solution would the nurse gestational trophoblastic disease
anticipate being prescribed initially?
normal saline Rationale: This client has risk factors of
a "molar" pregnancy: nausea and
Rationale: For the client with vomiting at an early gestational week
hyperemesis gravidarum, parenteral and dark brown vaginal discharge. The
fluids and drugs are prescribed to early nausea/vomiting can be due to a
rehydrate the client and reduce the high hCG level, which is a sign of
symptoms. The first choice for fluid gestational trophoblastic disease. There
replacement is generally isotonic, such is only one sign/symptom of
as normal saline, which aids in hyperemesis gravidarum. Placenta
preventing hyponatremia, with vitamins previa is marked by bright red bleeding
(pyridoxine, or vitamin B6) and and tends to happen later in gestation.
electrolytes added. Dextrose 5% and There are no data to support any
water and 0.45% sodium chloride are psychosis at this stage.
hypotonic solutions that would cause the
cells to swell and possibly burst. 62. A woman who is 10 weeks' pregnant
Albumin could lead to fluid overload. calls the physician's office reporting
"morning sickness" but, when asked
about it, tells the nurse that she is
nauseated and vomiting all the time and
60. A pregnant women calls the clinic to has lost 5 pounds. What interventions
report a small amount of painless would the nurse anticipate for this
vaginal bleeding. What response by the client?
nurse is best? Lab work will be drawn to rule out
"Please come in now for an evaluation acid-base imbalances.
by your health care provider." Rationale: Morning sickness that lasts
all day and is severe is called
Rationale: Bleeding during pregnancy hyperemesis gravidarum. It is much
is always a deviation from normal and more serious than "morning sickness"
and can lead to significant weight loss 65. During a routine prenatal visit, a
and electrolyte imbalance. Lab work client is found to have proteinuria and a
needs to be drawn to determine the blood pressure rise to 140/90 mm Hg.
extent of electrolyte loss and acid-base The nurse recognizes that the client has
balance. An ultrasound is performed but which condition?
it is done to determine if the mother is mild preeclampsia
experiencing a molar pregnancy.
Treatment for hyperemesis gravidarum Rationale: A woman is said to have
requires much more care than just rest, gestational hypertension when she
drinking fluids and eating crackers. develops an elevated blood pressure
(140/90 mm Hg) but has no proteinuria
63. A 35-year-old client is seen for her or edema. If a seizure from gestational
2-week postoperative appointment after hypertension occurs, a woman has
a suction curettage was performed to eclampsia, but any status above
evacuate a hydatidiform mole. The gestational hypertension and below a
nurse explains that the human chorionic point of seizures is preeclampsia. A
gonadotropin (hCG) levels will be woman is said to be mildly preeclamptic
reviewed every 2 weeks and teaches when she has proteinuria and a blood
about the need for reliable contraception pressure rise to 140/90 mm Hg, taken
for the next 6 months to a year. The on two occasions at least 6 hours apart.
client states, "I'm 35 already. Why do I A woman has passed from mild to
have to wait that long to get pregnant severe preeclampsia when her blood
again?" What is the nurse's best pressure rises to 160 mm Hg systolic
response? and 110 mm Hg diastolic or above on at
"A contraceptive is used so that a least two occasions 6 hours apart at bed
positive pregnancy test resulting from a rest (the position in which blood
new pregnancy will not be confused with pressure is lowest) or her diastolic
the increased level of hCG that occurs pressure is 30 mm Hg above her
with a developing malignancy." prepregnancy level. Marked proteinuria,
3+ or 4+ on a random urine sample or
Rationale: Because of the risk of more than 5 g in a 24-hour sample, and
choriocarcinoma, the woman receives extensive edema are also present. A
extensive treatment. Therapy includes woman has passed into eclampsia when
baseline chest X-ray to detect lung cerebral edema is so acute a grand-mal
metastasis, plus a physical exam seizure (tonic-clonic) or coma has
(including a pelvic exam). Serum B-hCG occurred.
levels weekly until negative results are
obtained three consecutive times, then 66. A pregnant woman with
monthly for 6 to 12 months. The woman preeclampsia is to receive magnesium
is cautioned to avoid pregnancy during sulfate IV. Which assessment should the
this time because the increasing B-hCG nurse prioritize before administering a
levels associated with pregnancy would new dose?
cause confusion as to whether cancer patellar reflex
had developed. If after a year B-hCG
serum titers are within normal levels, a Rationale: A symptom of magnesium
normal pregnancy can be achieved. sulfate toxicity is loss of deep tendon
reflexes. Assessing for the patellar
reflex or ankle clonus before
administration is assurance the drug Rationale: The most common causes of
administration will be safe. Assessing preterm labor include alcohol or drug
the blood pressure, heart rate, or use during pregnancy, lack of or no
anxiety level would not reveal a potential prenatal care, victim of intimate partner
magnesium toxicity. violence, black race, in vitro fertilization,
67. The obstetric nurse is caring for a and a lack of support. Hyperemesis
pregnant client who has been diagnosed gravidarum and Asian descent are not
with hydatidiform mole. What risk factors for preterm labor.
assessment should the nurse prioritize?
Vaginal bleeding
70. A client with a multiple gestation has
Rationale: Molar pregnancies constitute come to a health care facility for a
a major risk factor for vaginal bleeding. regular antenatal check-up. When
The client does not normally have an educating the client on pregnancy, about
increased risk for nausea, pain, or which complication should the nurse
hypertension. inform the client?
Placental dysfunction
68. The nurse is teaching a client who is
diagnosed with preeclampsia how to Rationale: The nurse should inform the
monitor her condition. The nurse client that placental dysfunction might
determines the client needs more occur as a complication of multiple
instruction after making which pregnancies. Other complications of
statement? multiple pregnancies include preterm
"If I have changes in my vision, I will lie labor, hypertension, anemia, cord
down and rest." abnormalities, congenital anomalies,
intrauterine growth restriction, and low
Rationale: Changes in the visual field birth weight. Hypertension, and not
may indicate the client has moved from hypotension, is seen in multiple
preeclampsia to severe preeclampsia pregnancies. Fetal macrosomia is not
and is at risk for developing a seizure seen in cases of multiple gestation.
due to changes in cerebral blood flow. Constipation, and not diarrhea, is also
The client would require immediate seen as a complication of multiple
assessment and intervention. Gaining pregnancies. This is due to the
weight is not necessarily a sign of decreased functioning of the
worsening preeclampsia. The other gastrointestinal system in multiple
choices are instructions which the client pregnancy.
may be given to follow.
71. After a regular prenatal visit, a
69. The nurse is educating a group of pregnant client asks the nurse to
pregnant women about risk factors describe the differences between
associated with preterm labor. Which placental abruption (abruptio placentae)
factor would the nurse include in the and placenta previa. Which statement
teaching? Select all that apply. will the nurse include in the teaching?
alcohol use during pregnancy Placenta previa is an abnormally
lack of prenatal care implanted placenta that is too close to
victim of intimate partner violence the cervix.
Rationale: Placenta previa is a ultrasound indicates a gestation sac is
condition of pregnancy in which the found in the right lower quadrant?
placenta is implanted abnormally in the Immediate surgery
lower part of the uterus and is the most
common cause of painless, bright red Rationale: The client presents with the
bleeding in the third trimester. Placental signs and symptoms of an ectopic
abruption is the premature separation of pregnancy, which is confirmed by the
a normally implanted placenta that pulls transvaginal ultrasound. Ectopic
away from the wall of the uterus either pregnancy means an embryo has
during pregnancy or before the end of implanted outside the uterus. Surgery is
labor. Placental abruption can result in necessary to remove the growing
concealed or apparent dark red bleeding structure before damage can occur to
and is painful. Immediate intervention is the woman's internal organs. Bed rest, a
required for placental abruption. tocolytic, and internal uterine monitoring
will not correct the situation. The
growing structure must be removed
72. A nurse is caring for a client who just surgically.
experienced a spontaneous abortion
(miscarriage) in her first trimester. When
asked by the client why this happened,
which is the best response from the
nurse? 74. A nurse is caring for a pregnant
abnormal fetal development client admitted with mild preeclampsia.
Which assessment finding should the
Rationale: The most frequent cause of nurse prioritize?
spontaneous abortion (miscarriage) in urine output of less than 15 ml/hr
the first trimester of pregnancy is
abnormal fetal development, due either Rationale: Severe preeclampsia may
to a teratogenic factor or to a develop suddenly and bring with it high
chromosomal aberration. In other blood pressure of more than 160/110
miscarriages, immunologic factors may mm Hg, proteinuria of more than 500
be present or rejection of the embryo mg in 24 hours, oliguria of less than 15
through an immune response may ml/hr, cerebral and visual symptoms,
occur. Another common cause of early and rapid weight gain. Mild facial edema
miscarriage involves implantation or hand edema occurs with mild
abnormalities. Miscarriage may also preeclampsia. A urinary output of 15
occur if the corpus luteum on the ovary ml/hr would result in an output of 360
fails to produce enough progesterone to ml/24 hours, which would be below the
maintain the decidua basalis. recommended range and should be
reported. Ankle edema of 1+ could be
73. A 24-year-old woman presents with related to regular pregnancy and not
vague abdominal pains, nausea, and necessarily just severe preeclampsia. A
vomiting. An urine hCG is positive after finding of 3+ to 4+ pitting edema would
the client mentioned that her last be more alarming and require
menstrual period was 2 months ago. intervention.
The nurse should prepare the client for
which intervention if the transvaginal 75. The nurse is caring for a woman at
32 weeks' gestation with severe
preeclampsia. Which assessment miscarriage. Which action by the nurse
finding should the nurse prioritize after is most appropriate?
the administration of hydralazine to this Contact the health care provider to
client? report the client's feelings.
Tachycardia
Rationale: The client may be
Explanation: experiencing a psychological situation
Hydralazine reduces blood pressure but that needs intervention by a trained
is associated with adverse effects such professional in the area of mental
as palpitation, tachycardia, headache, health. The hyperemesis gravidarum
anorexia, nausea, vomiting, and may worsen her feelings toward the
diarrhea. It does not cause pregnancy, so reporting her feelings to
gastrointestinal bleeding, blurred vision the health care provider is the best
(halos around lights), or sweating. action at this time. Although the nurse
Magnesium sulfate may cause will continue to monitor the client's
sweating. hyperemesis gravidarum, this is not the
only action needed at this time and
76. A pregnant client with preeclampsia there is a better action. Encouraging the
with severe features has developed client to be positive about her situation
HELLP syndrome. In addition to the may obstruct therapeutic
observations necessary for communication. Sharing the information
preeclampsia, what other nursing with the client's family is not appropriate,
intervention is critical for this client? because the scenario described does
observation for bleeding not indicate that the nurse has the
client's permission to share this
information with the family.
Explanation:
Because of the low platelet count 78. A client reporting she recently had a
associated with this condition, women positive pregnancy test has reported to
with HELLP syndrome need extremely the emergency department stating
close observation for bleeding, in one-sided lower abdominal pain. The
addition to the observations necessary health care provider has prescribed a
for preeclampsia. Maintaining a patent series of tests. Which test will provide
airway is a critical intervention needed the most definitive confirmation of an
for a client with eclampsia while she is ectopic pregnancy?
having a seizure. Administration of a Abdominal ultrasound
tocolytic would be appropriate for halting
labor. Monitoring for infection is not a Rationale: An ectopic pregnancy refers
priority intervention in this situation. to the implantation of the fertilized egg in
a location other than the uterus.
77. An 18-year-old pregnant client is Potential sites include the cervix, uterus,
hospitalized as she recovers from abdomen, and fallopian tubes. The
hyperemesis gravidarum. The client confirmation of the ectopic pregnancy
reveals she wanted to have an abortion can be made by an ultrasound, which
(elective termination of pregnancy) but would confirm that there was no uterine
her cultural background forbids it. She is pregnancy. A quantitative hCG level
very unhappy about being pregnant and may be completed in the diagnostic
even expresses a wish for a plan. hCG levels in an ectopic
pregnancy are traditionally reduced. replacement is generally isotonic, such
While this would be an indication, it as normal saline, which aids in
would not provide a positive preventing hyponatremia, with vitamins
confirmation. The qualitative hCG test (pyridoxine, or vitamin B6) and
would provide evidence of a pregnancy, electrolytes added. Dextrose 5% and
but not the location of the pregnancy. A water and 0.45% sodium chloride are
pelvic exam would be included in the hypotonic solutions that would cause the
diagnostic plan of care. It would likely cells to swell and possibly burst.
show an enlarged uterus and cause Albumin could lead to fluid overload.
potential discomfort to the client but
would not be a definitive finding. 81. A client tells that nurse in the
doctor's office that her friend developed
79. The following hourly assessments high blood pressure on her last
are obtained by the nurse on a client pregnancy. She is concerned that she
with preeclampsia receiving magnesium will have the same problem. What is the
sulfate: 97.3oF (36.2oC), HR 88, RR 12 standard of care for preeclampsia?
breaths/min, BP 148/110 mm Hg. What Have her blood pressure checked at
other priority physical assessments by every prenatal visit.
the nurse should be implemented to
assess for potential toxicity? Rationale: Preeclampsia and eclampsia
reflexes are common problems for pregnant
clients and require regular blood
Rationale: Reflex assessment is part of pressure monitoring at all prenatal visits.
the standard assessment for clients on Antihypertensives are not prescribed
magnesium sulfate. The first change unless the client is already hypertensive.
when developing magnesium toxicity Monitoring for headaches and swelling
may be a decrease in reflex activity. The is a good predictor of a problem but
health care provider needs to be notified doesn't address prevention—nor does it
immediately. A change in lung sounds predict who will have hypertension.
and oxygen saturation are not indicative Taking aspirin has shown to reduce the
of magnesium sulfate toxicity. Hourly risk in women who have moderate to
blood draws to gain information on the high risk factors, but has shown no
magnesium sulfate level are not effect on those women with low risk
indicated. factors.

80. A pregnant client diagnosed with 82. A nurse in the maternity triage unit is
hyperemesis gravidarum is prescribed caring for a client with a suspected
intravenous fluids for rehydration. When ectopic pregnancy. Which nursing
preparing to administer this therapy, intervention should the nurse perform
which solution would the nurse first?
anticipate being prescribed initially? Assess the client's vital signs.
normal saline
Rationale: A suspected ectopic
Rationale: For the client with pregnancy can put the client at risk for
hyperemesis gravidarum, parenteral hypovolemic shock. The assessment of
fluids and drugs are prescribed to vital signs should be performed first,
rehydrate the client and reduce the followed by any procedures to maintain
symptoms. The first choice for fluid the ABCs. Providing emotional support
would also occur, as would obtaining a common cause of miscarriage in the first
surgical consent, if needed, but these trimester is related to which factor?
are not first steps. Chromosomal defects in the fetus

83. A client at 25 weeks' gestation Rationale: Fetal factors are the most
presents with a blood pressure of common cause of early miscarriages,
152/99 mm Hg, pulse 78 beats/min, no with chromosomal abnormalities in the
edema, and urine negative for protein. fetus being the most common reason.
What would the nurse do next? This client fits the criteria for early
Notify the health care provider spontaneous abortion (miscarriage)
since she was only 10 weeks' pregnant
Explanation: and early miscarriage occurs before 12
The client is exhibiting a sign of weeks.
gestational hypertension, elevated blood
pressure greater than or equal to 140/90 85. A woman at 31 weeks' gestation
mm Hg that develops for the first time presents to the emergency department
during pregnancy. The health care with bright red vaginal bleeding,
provider should be notified to assess the reporting that the onset of the bleeding
client. Without the presence of edema or was sudden and without pain. Which
protein in the urine, the client does not diagnostic test should the nurse
have preeclampsia. prioritize?
A transvaginal ultrasound
84. A woman at 34 weeks' gestation
presents to labor and delivery with Rationale: For any pregnant woman
vaginal bleeding. Which finding from the who presents with painless bleeding,
obstetric examination would lead to a placenta previa needs to be ruled out by
diagnosis of placental abruption either transvaginal or abdominal
(abruptio placentae)? ultrasound. A digital cervical exam is
Onset of vaginal bleeding was sudden contraindicated until placenta previa has
and painful been ruled out, as digital manipulation of
placental tissue through the cervical os
Explanation: can cause uncontrollable bleeding. The
Sudden onset of abdominal pain and nonstress test and electronic fetal
vaginal bleeding with a rigid uterus that monitoring would be utilized after the
does not relax are signs of a placental woman is stabilized and placenta previa
abruption (abruptio placentae). The has either been diagnosed or ruled out.
other findings are consistent with a
diagnosis of placenta previa. 87. The nurse is teaching a client who is
diagnosed with preeclampsia how to
85. A woman in her 20s has monitor her condition. The nurse
experienced a spontaneous abortion determines the client needs more
(miscarriage) at 10 weeks' gestation and instruction after making which
asks the nurse at the hospital what went statement?
wrong. She is concerned that she did "If I have changes in my vision, I will lie
something that caused her to lose her down and rest."
baby. The nurse can reassure the
woman by explaining that the most Rationale: Changes in the visual field
may indicate the client has moved from
preeclampsia to severe preeclampsia hyperemesis gravidarum. The client
and is at risk for developing a seizure reveals she wanted to have an abortion
due to changes in cerebral blood flow. (elective termination of pregnancy) but
The client would require immediate her cultural background forbids it. She is
assessment and intervention. Gaining very unhappy about being pregnant and
weight is not necessarily a sign of even expresses a wish for a
worsening preeclampsia. The other spontaneous abortion (miscarriage).
choices are instructions which the client Which action by the nurse is most
may be given to follow. appropriate?
Contact the health care provider to
88. The nurse is assessing a client at 12 report the client's feelings.
weeks' gestation at a routine prenatal
visit who reports something doesn't feel Rationale: The client may be
right. Which assessment findings should experiencing a psychological situation
the nurse prioritize? that needs intervention by a trained
gestational hypertension, hyperemesis professional in the area of mental
gravidarum, absence of FHR health. The hyperemesis gravidarum
may worsen her feelings toward the
Rationale:Rationale: The early pregnancy and needs to be monitored,
development of gestational so reporting her feelings to the health
hypertension/preeclampsia, care provider is the best action at this
hyperemesis gravidarum, and the time. Although the nurse will continue to
absence of FHR are suspicious for monitor the client's hyperemesis
gestational trophoblastic disease. The gravidarum, this is not the only action
elevated levels of hCG lead to the needed at this time and there is a better
severe morning sickness. There is no action. Encouraging the client to remain
fetus, so FHR, quickening, and evidence silent about her feelings may obstruct
of a fetal skeleton would not be seen. therapeutic communication. Sharing the
The abdominal enlargement is greater information with the client's family is not
than expected for pregnancy dates, but appropriate because the scenario
hCG, not hPL, levels are increased. described does not indicate that the
nurse has the client's permission to
89. The nurse is caring for a client who share this information with her family.
has a multifetal pregnancy. What topic
should the nurse prioritize during health 90. A woman at 35 weeks' gestation
education? with severe polyhydramnios is admitted
signs of preterm labor to the hospital. The nurse recognizes
that which concern is greatest regarding
Rationale: The client with a multifetal this client?
pregnancy must be made aware of the preterm rupture of membranes followed
risks posed by preterm labor. There is by preterm birth
no corresponding increase in the risk for
hypertension or blood incompatibilities. Rationale: Even with precautions, in
Parenting skills are secondary to most instances of polyhydramnios, there
physiologic needs at this point. will be preterm rupture of the
membranes because of excessive
89. An 18-year-old pregnant client is pressure, followed by preterm birth. The
hospitalized as she recovers from
other answers are less concerning than b) Is characterized by hemolysis,
preterm birth in this pregnancy. elevated liver enzymes, and low
platelets.
91. A client in her 20th week of c) Is a mild form of preeclampsia.
gestation d) Can be diagnosed by a nurse
develops HELLP syndrome. What are alert to its symptoms.
features of HELLP syndrome? Select all
that apply. 93. Magnesium sulfate is given to
elevated liver enzymes women with preeclampsia and
hemolysis eclampsia to:
low platelet count a) Prevent a boggy uterus and lessen
lochial flow.
Rationale: The HELLP syndrome is a b) Improve patellar reflexes and
syndrome involving hemolysis increase respiratory efficiency.
(microangiopathic hemolytic anemia), c) Prevent and treat convulsions.
elevated liver enzymes, and a low d) Shorten the duration of labor.
platelet count. Hyperthermia and
leukocytosis are not features of HELLP 94. A pregnant woman has been
syndrome. receiving a magnesium sulfate infusion
for treatment of severe preeclampsia for
91. A pregnant women calls the clinic to 24 hours. On assessment the nurse
report a small amount of painless finds the following vital signs:
vaginal bleeding. What response by the temperature of 37.3° C, pulse rate of 88
nurse is best? beats/min, respiratory rate of 10
"Please come in now for an evaluation breaths/min, blood pressure (BP) of
by your health care provider." 148/90 mm Hg, absent deep tendon
reflexes, and no ankle clonus. The
Explanation: client's response to questions are
Bleeding during pregnancy is always a sluggish are hard to understand. The
deviation from normal and should be nurse:
evaluated carefully. It may be a) Calls for a stat magnesium
life-threatening or it may be something sulfate level.
that is not a threat to the mother and/or b) Discontinues the magnesium
fetus. Regardless, it needs to be sulfate infusion.
evaluated quickly and carefully. Telling c) Prepares to administer
the client it may be harmless is a hydralazine.
reassuring statement, but does not d) Administers oxygen.
suggest the need for urgent evaluation.
Having the mother lay on her left side 95. The labor of a pregnant woman with
and drink water is indicated for preeclampsia is going to be induced.
cramping. Before initiating the Pitocin infusion, the
nurse reviews the woman’s latest
92. Nurses should be aware that HELLP laboratory test findings, which reveal an
syndrome: hematocrit of 41%, a creatinine of 6.2, a
a) Is associated with preterm labor platelet count of 90,000, an elevated
but not perinatal mortality. aspartate transaminase (AST) level, and
decreased serum haptoglobin. The
nurse notifies the physician because the 100. A woman with preeclampsia has a
laboratory results are indicative of: seizure. The nurse’s primary duty during
a) Disseminated intravascular the seizure is to:
coagulation (DIC). a) Administer oxygen by mask.
b) Eclampsia. b) Suction the mouth to prevent
c) HELLP syndrome. aspiration.
d) Idiopathic thrombocytopenia. c) Stay with the client and call for
help.
97. A woman with severe preeclampsia d) Insert an oral airway.
has been receiving magnesium sulfate
by intravenous infusion for 8 hours. The 101. A woman with severe preeclampsia
nurse assesses the woman and is receiving a magnesium sulfate
documents the following findings: infusion. The nurse becomes concerned
temperature of 37.1° C, pulse rate of 96 after assessment when the woman
beats/min, respiratory rate of 24 exhibits:
breaths/min, blood pressure (BP) of a) Absent ankle clonus.
175/112 mm Hg, 3+ deep tendon b) A sleepy, sedated affect.
reflexes, and no ankle clonus. The c) A respiratory rate of 10
nurse calls the physician, anticipating an breaths/min.
order for: d) Deep tendon reflexes of 2.
a) Diazepam.
b) Hydralazine. 102. Your client has been on
c) Calcium gluconate. magnesium sulfate for 20 hours for
d) Magnesium sulfate bolus. treatment of preeclampsia. She just
delivered a viable infant girl 30 minutes
98. A primigravida is being monitored in ago. What uterine findings would you
her prenatal clinic for preeclampsia. expect to observe/assess in this client?
What finding should concern her nurse? a) A fundus firm below the level of
a) Weight gain of 0.5 kg during the the umbilicus
past 2 weeks b) A boggy uterus with heavy lochia
b) Pitting pedal edema at the end of flow
the day c) Scant lochia flow
c) Blood pressure (BP) increase to d) Absence of uterine bleeding in
138/86 mm Hg the postpartum period
d) A dipstick value of 3+ for protein
in her urine 103. A woman has been having
contractions since 4 A.M. At 8 A.M., her
99. What nursing diagnosis would be cervix is dilated to 5 cm. At 10 am her
the most appropriate for a woman cervix is still 5 cm. Contractions are
experiencing severe preeclampsia? frequent, and mild to moderate in
a) Risk for injury to the fetus related intensity. Cephalopelvic disproportion
to uteroplacental insufficiency (CPD) has been ruled out. The nurse
b) Risk for eclampsia would anticipate preparing for:
c) Risk for deficient fluid volume a) Cesarean section.
related to increased sodium retention b) Documentation your assessment
secondary to administration of MgSO4 and assess again in an hour.
d) Risk for increased cardiac output c) Increased intravenous infusion.
related to use of antihypertensive drugs d) Oxytocin induction of labor.
105. A nurse is comparing advantages prediction is so hit-and-miss, teaching
of using active management of labor pregnant women the symptoms
(AMOL) with a less interventional probably causes more harm through
approach. The nurse knows that the false alarms.
goal of AMOL is: b) Because preterm labor is likely to
a) Preventing protracted labor and be the start of an extended labor, a
arrest of progress. woman with symptoms can wait several
b) Relieving the anxiety and fear hours before contacting the primary
accompanying labor. caregiver.
c) Detecting and intervening in c) Braxton Hicks contractions often
cases of precipitous labor. signal the onset of preterm labor
d) Preventing infection from d) The diagnosis of preterm labor is
prolonged rupture of membranes. based on gestational age, uterine
activity, and progressive cervical
106. Which of the following is true with change.
respect to chorioamnionitis?
a) Once a woman who has had 110. A woman in preterm labor at 30
chorioamnionitis has delivered the weeks of gestation receives two 12-mg
antibiotics will be stopped. doses of betamethasone
b) If a woman has chorioamnionitis intramuscularly. The purpose of this
she will be treated with broad spectrum pharmacologic treatment is to:
antibiotics and allowed to continue in a) Stimulate fetal surfactant
labor. production.
c) Most often chorioamnionitis is b) Suppress uterine contractions.
caused by pathogens such as GBBS. c) Maintain adequate maternal
d) An epidural can cause maternal respiratory effort and ventilation during
fever and fetal tachycardia. magnesium sulfate therapy.
d) Reduce maternal and fetal
107. Which symptoms diagnose tachycardia associated with ritodrine
chorioamnionitis? (Select all that apply) administration.
a) Maternal or Fetal Tachycardia
b) Maternal fever > or = 38o C 111. Complications and risks associated
c) Foul discharge with cesarean births include (choose all
d) Uterine tenderness that apply):
a) Wound dehiscence.
108. Approximately 50% of all women b) Pulmonary edema.
who give birth prematurely have no c) Urinary tract infections.
identifiable risk factors, and about 50% d) Fetal injuries.
of preterm births could not be e) Hemorrhage.
prevented.
a) False 112. During labor, the patient at 4 cm
b) True suddenly becomes dyspneic, cyanotic,
and hypotensive. The nurse must
109. With regard to the care prepare immediately for: (Select all that
management of preterm labor, nurses apply.)
should be aware that: a) CPR.
a) Because all women must be b) McRobert's maneuver.
considered at risk for preterm labor and c) Immediate vaginal delivery.
d) Cesarean delivery 117. After delivery it is determined that
there is a placenta accreta. Which
113. Nurses should be aware that the intervention should the nurse anticipate?
induction of labor: a) Surgery with possible
a) Is also known as a trial of labor Hysterectomy
(TOL). b) 2 L oxygen by mask
b) Is rated for likelihood of induction c) Intravenous antibiotics
success by a Bishop score. d) Intravenous oxytocin
c) Is almost always done for
medical reasons. 118. Which of the following is true about
d) Can be achieved by external and placenta previa.
internal version techniques a) Once placenta previa is
diagnosed by a 20 week ultrasound, it is
114. A client is in active labor. The nurse very likely the placenta previa will
determines that the fetus's position is resolve in the third trimester.
occiput posterior. Which nursing b) In evaluating the bleeding, a
diagnosis will apply to this woman's care vaginal exam would be done to
if the occiput posterior position becomes determine the cause of the bleeding.
persistent? c) Symptoms of placenta previa are
a) Risk for injury painful frequent contractions and bright
b) Acute pain red vaginal bleeding
c) Fluid volume deficit d) The bleeding from placenta
d) Impaired gas exchange previa usually occurs late in pregnancy
at term.
115. A pregnant client was in an
automobile accident, and presents to 118. With regard to the process of
the labor suite from the Emergency inducing labor, nurses should be aware
Department after she presented at the that:
ED with a deep gash on her forearm a) Ripening the cervix usually
from the crash. The client was not results in a decreased success rate for
wearing a seat belt, and was speeding. induction
She now seems highly agitated, is b) Amniotomy can be used to make
pacing continuously, and is talking very the cervix more favorable for labor.
rapidly. The nurse should suspect that c) Oxytocin is less expensive than
this client has what type of prostaglandins and more effective but
psychological disorder? creates greater health risks.
a) Schizophrenia d) Labor sometimes can be induced
b) Social anxiety disorder with balloon catheters or laminaria tents.
c) Obsessive-compulsive disorder
d) Bipolar disorder, manic phase 120. When the provider indicates a
shoulder dystocia is occurring during the
116. The most prevalent clinical delivery of a macrosomic fetus, the
manifestation of abruptio placentae (as nurse would assist by:
opposed to placenta previa) is: a) Calling a second physician to
a) Intense abdominal pain. assist.
b) Uterine activity. b) Utilizing fundal pressure to push
c) Cramping. the fetus out.
d) Bleeding.
c) Assisting the woman into electronic fetal monitor for a minimum of
McRobert's maneuver. 4 hours.
d) Preparing for an immediate d) Rhogam is not necessary for rH
cesarean delivery. negative pregnant women after a blunt
force trauma.
121. A primigravida is admitted to the
birth setting in early labor. She is 3 cm 124. A woman has been in labor for 16
dilated, -2 station, with intact hours. Her cervix is dilated to 3 cm and
membranes and FHR of 150 bpm. Her is 80% effaced. The fetal presenting part
membranes rupture spontaneously, and is not engaged. The nurse would
the FHR drops to 90 bpm with variable suspect:
decelerations. The initial response from a) Abruptio placentae
the nurse would be to: b) Breech malpresentation.
a) Notify the physician. c) Fetal demise.
b) Administer oxygen at 2 l per d) Cephalopelvic disproportion
nasal cannula. (CPD).
c) Perform a vaginal exam.
d) Place the client in a left lateral 125. For a woman at 42 weeks of
position gestation, which finding would require
more assessment by the nurse?
122. A multipara is having intense a) Score of 8 on the biophysical
uterine contractions with little uterine profile
relaxation between contractions. Vaginal b) One fetal movement noted in 1
examination reveals rapid cervical hour of assessment by the mother
dilation and fetal descent. The nurse c) Cervix dilated 2 cm and 50%
should effaced
a) Place the woman in knee—chest d) Fetal heart rate of 116 beats/min
position
b) Notify the physician of these 126. The nurse is caring for a client
findings. whose labor is being augmented with
c) Turn off the lights to make it oxytocin. He or she recognizes that the
easier for the woman to relax. oxytocin should be discontinued
d) Assemble supplies to prepare for immediately if there is evidence of:
a cesarean birth. a) Rupture of the client’s amniotic
membranes.
123. Which of the following is the correct b) Uterine contractions occurring
about care for a pregnant woman who every 8 to 10 minutes
has experienced blunt trauma in a car c) A fetal heart rate (FHR) of 180
accident? with absence of variability.
a) The two most common risks are d) The client needing to void.
preterm labor and fetal death.
b) If the woman does not have more 127. In evaluating the effectiveness of
than 6 ctx an hour she may go home oxytocin induction, the nurse would
after 4 hours. expect:
c) In the ER she is evaluated and a) At least 30 mU/min of oxytocin
treated to hemodynamically stabilize will be needed to achieve cervical
her, then she is evaluated with an dilation
b) Contractions lasting 40 to 90 e) Position on her side and give her
seconds, 2 to 3 minutes apart. oxygen if the fetal heart rate was
c) The intensity of contractions to category II.
be at least 110 to 130 mm Hg.
d) Labor to progress at least 2 133. The nurse providing care for the
cm/hr dilation. laboring woman should understand that
amnioinfusion is used to treat
128. A pregnant woman’s amniotic a) Variable decelerations
membranes rupture. Prolapsed cord is (intermittent umbilical cord compression)
suspected. What intervention would be b) Fetal tachycardia.
the top priority? c) Late decelerations.
a) Preparing the woman for a d) Fetal bradycardia.
cesarean birth
b) Placing the woman in the
knee-chest position
c) Covering the cord in sterile gauze
soaked in saline
d) Starting oxygen by face mask

129. With regard to the use of tocolytic


therapy to suppress premature uterine
activity, nurses should be aware that:
a) There are no important maternal
(as opposed to fetal) contraindications.
b) Its most important function is to
afford the opportunity to administer
antenatal glucocorticoids.
c) The drugs can be given
efficaciously up to the designated
beginning of term at 37 weeks.
d) If the client develops pulmonary
edema while on tocolytics, intravenous
(IV) fluids should be given.

131. A woman arrive in the admission


area of L&D. She is complaining of
severe abdominal pain which she thinks
are contractions and vaginal bleeding.
You notice the sheet on the bed is about
1/3 covered with port wine fluid. You
would do all of the following EXCEPT:
a) NOtify the charge nurse and
patient's provider.
b) Put her on the monitor
c) Take a complete medical history
and measure her vital signs.
d) Start an IV
Chapter 22 clients in the labor and birth unit. Which
measure would the nurse emphasize as
1. Review of a primiparous woman's part of this
labor and birth record reveals a program?
prolonged second stage of labor and A. administering broad-spectrum
extended time in the stirrups. Based on antibiotics
an interpretation of these findings, the B. inspecting the placenta after delivery
nurse would be especially alert for which for intactness
condition? C. manually removing the placenta at
A. retained placental fragments birth
B. hypertension D. applying pressure to the umbilical
C. thrombophlebitis cord to remove the placenta
D. uterine subinvolution
5. A multipara client develops
2. The nurse is conducting a class for thrombophlebitis after birth. Which
postpartum women about mood assessment findings would
disorders. The nurse describes a lead the nurse to intervene
transient, self-limiting mood disorder immediately?
that affects mothers after birth. The A. dyspnea, diaphoresis, hypotension,
nurse determines that the women and chest pain
understood the description when they B. dyspnea, bradycardia, hypertension,
identify the condition as postpartum: and confusion
A. depression. C. weakness, anorexia, change in level
B. psychosis. of consciousness, and coma
C. bipolar disorder. D. pallor, tachycardia, seizures, and
D. blues. jaundice

3. A woman who is 2 weeks postpartum 6. A client experienced prolonged labor


calls the clinic and says, "My left breast with prolonged premature rupture of
hurts." After membranes. The
further assessment on the phone, the nurse would be alert for which condition
nurse suspects the woman has mastitis. in the mother and the newborn?
In addition to pain, A. infection
the nurse would question the woman B. hemorrhage
about which symptom? C. trauma
A. an inverted nipple on the affected D. hypovolemia
breast
B. no breast milk in the affected breast 7. When assessing the postpartum
C. an ecchymotic area on the affected woman, the nurse uses indicators other
breast than pulse rate and
D. hardening of an area in the affected blood pressure for postpartum
breast hemorrhage because:
A. these measurements may not change
4. A nurse is developing a program to until after the blood loss is large.
help reduce the risk of late postpartum B. the body's compensatory
hemorrhage in mechanisms activate and prevent any
changes.
C. they relate more to change in 11. A client is experiencing postpartum
condition than to the amount of blood hemorrhage, and the nurse begins to
lost. massage her fundus.
D. maternal anxiety adversely affects Which action would be most appropriate
these vital signs. for the nurse to do when massaging the
woman's
8. A nurse is assessing a postpartum fundus?
client. Which finding would the cause A. Place the hands on the sides of the
the nurse the greatest abdomen to grasp the uterus.
concern? B. Use an up-and-down motion to
A. leg pain on ambulation with mild massage the uterus.
ankle edema C. Wait until the uterus is firm to express
B. calf pain with dorsiflexion of the foot clots.
C. perineal pain with swelling along the D. Continue massaging the uterus for at
episiotomy least 5 minutes.
D. sharp, stabbing chest pain with
shortness of breath 12. After teaching a woman with a
postpartum infection about care after
9. A woman who is experiencing discharge, which client
postpartum hemorrhage is extremely statement indicates the need for
apprehensive and additional teaching?
diaphoretic. The woman's extremities A. "I need to call my doctor if my
are cool and her capillary refill time is temperature goes above 100.4° F (38°
increased. Based on C).”
this assessment, the nurse suspects B. "When I put on a new pad, I'll start at
that the client is experiencing the back and go forward."
approximately how much blood C. "If I have chills or my discharge has a
loss? strange odor, I'll call my doctor."
A. 20% D. "I'll point the spray of the peri-bottle
B. 30% so it the water flows front to back."
C. 40%
D. 60% 13. A nurse is making a home visit to a
postpartum client. Which finding would
10. A postpartum client is prescribed lead the nurse to
medication therapy as part of the suspect that a woman is experiencing
treatment plan for postpartum psychosis?
postpartum hemorrhage. Which A. delirium
medication would the nurse expect to B. feelings of guilt
administer in this C. sadness
situation? D. insomnia
A. Magnesium sulfate
B. methylergonovine 14. A nurse is reviewing a journal article
C. Indomethacin on the causes of postpartum
D. nifedipine hemorrhage. Which
Answer: B condition would the nurse most likely
find as the most common cause?
A. labor augmentation
B. uterine atony
C. cervical or vaginal lacerations 18. A postpartum client comes to the
D. uterine inversion clinic for her routine 6-week visit. The
nurse assesses the
15. A postpartum woman is diagnosed client and suspects that she is
with endometritis. The nurse interprets experiencing subinvolution based on
this as an infection which finding?
involving which area? Select all that A. nonpalpable fundus
apply. B. moderate lochia serosa
A. endometrium C. bruising on arms and legs
B. decidua D. fever
C. myometrium
D. broad ligament 19. Assessment of a postpartum client
E. ovaries reveals a firm uterus with bright-red
F. fallopian tubes bleeding and a
localized bluish bulging area just under
16. A group of nurses are reviewing the skin at the perineum. The woman
information about mastitis and its also reports
causes in an effort to significant pelvic pain and is
develop a teaching program on experiencing problems with voiding. The
prevention for postpartum women. The nurse suspects which
nurses demonstrate condition?
understanding of the information when A. hematoma
they focus the teaching on ways to B. laceration
minimize risk of C. bladder distention
exposure to which organism? D. uterine atony
A. E. coli Answer: A
B. S. aureus
C. Proteus 20. A postpartum woman is prescribed
D. Klebsiella oxytocin to stimulate the uterus to
Answer: B contract. Which action
would be most important for the nurse to
17. A home health care nurse is do?
assessing a postpartum woman who A. Administer the drug as an IV bolus
was discharged 2 days ago. injection.
The woman tells the nurse that she has B. Give as a vaginal or rectal
a low-grade fever and feels "lousy." suppository.
Which finding would C. Piggyback the IV infusion into a
lead the nurse to suspect endometritis? primary line.
Select all that apply. D. Withhold the drug if the woman is
A. lower abdominal tenderness hypertensive.
B. urgency
C. flank pain 21. Assessment of a postpartum woman
D. breast tenderness experiencing postpartum hemorrhage
E. anorexia reveals mild
Answer: A, E shock. Which finding would the nurse
expect to assess? Select all that apply.
A. diaphoresis
B. tachycardia
C. oliguria anticoagulant therapy. After teaching the
D. cool extremities woman about this treatment, the nurse
E. confusion determines that
additional teaching is needed when the
22. A nurse is providing a refresher woman makes which statement?
class for a group of postpartum nurses. A. "I will use a soft toothbrush to brush
The nurse reviews my teeth."
the risk factors associated with B. "I can take ibuprofen if I have any
postpartum hemorrhage. The group pain."
demonstrates understanding C. "I need to avoid drinking any alcohol."
of the information when they identify D. "I will call my health care provider if
which risk factors associated with my stools are black and tarry."
uterine tone? Select all
that apply.
A. rapid labor 26. The nurse is developing a discharge
B. retained blood clots teaching plan for a postpartum woman
C. hydramnios who has
D. operative birth developed a postpartum infection.
E. fetal malpostion Which measures would the nurse most
likely include in this
23. A nurse is massaging a postpartum teaching plan? Select all that apply.
client's fundus and places the A. taking the prescribed antibiotic until it
nondominant hand on the is finished
area above the symphysis pubis based B. checking temperature once a week
on the understanding that this action: C. washing hands before and after
A. determines that the procedure is perineal care
effective. D. handling perineal pads by the edges
B. helps support the lower uterine E. directing peribottle to flow from back
segment. to front
C. aids in expressing accumulated clots.
D. prevents uterine muscle fatigue. 27. A nurse is assessing a postpartum
Answer: B client who is at home. Which statement
by the client would
24. A nurse is developing a plan of care lead the nurse to suspect that the client
for a woman who is at risk for may be developing postpartum
thromboembolism. Which depression?
measure would the nurse include as the A. "I just feel so overwhelmed and tired."
most cost-effective method for B. "I'm feeling so guilty and worthless
prevention? lately."
A. prophylactic heparin administration C. "It's strange, one minute I'm happy,
B. compression stockings the next I'm sad."
C. early ambulation D. "I keep hearing voices telling me to
D. warm compresses take my baby to the river."

25. A postpartum woman who


developed deep vein thrombosis is
being discharged on
28. As part of an in-service program to a the pad is now saturated. The uterus is
group of home health care nurses who firm and approximately at the level of
care for the umbilicus.
postpartum women, a nurse is Further inspection of the perineum
describing postpartum depression. The reveals an area, bluish in color and
nurse determines that the bulging just under the skin
teaching was successful when the group surface. Which action would the nurse
identifies that this condition becomes do next?
evident at which A. Apply warm soaks to the area.
time after birth of the newborn? B. Notify the health care provider.
A. in the first week C. Massage the uterine fundus.
B. within the first 2 weeks D. Encourage the client to void.
C. in approximately 1 month
D. within the first 6 weeks 32. A nurse is providing education to a
woman who is experiencing postpartum
29. A nurse suspects that a client may hemorrhage and
be developing disseminated is to receive a uterotonic agent. The
intravascular coagulation. The nurse determines that additional
woman has a history of placental teaching is needed when the
abruption (abruptio placentae) during woman identifies which drug as possibly
birth. Which finding being prescribed as treatment?
would help to support the nurse's A. oxytocin
suspicion? B. methylergonovine
A. severe uterine pain C. carboprost
B. board-like abdomen D. magnesium sulfate
C. appearance of petechiae
D. inversion of the uterus 1. An adolescent client in labor is dilated
4 cm and on admission asks for an
30. On a follow-up visit to the clinic, a epidural. For cultural reasons, the
nurse suspects that a postpartum client client's mother states that her daughter
is experiencing "has to bite the bullet, just like I did." The
postpartum psychosis. Which finding adolescent does not respond to the
would most likely lead the nurse to mother's statement. What should the
suspect this condition? nurse do to make sure the client's
A. delusional beliefs request is honored?
B. feelings of anxiety
C. sadness a) Request that an anesthesiologist
D. insomnia administer the epidural at this time
Answer: A because the client is uncomfortable and
has requested it
31. A nurse is assessing a client who b) Knowing the client's cultural
gave birth vaginally about 4 hours ago. background, suggest that the family call
The client tells the a meeting to make the best decision for
nurse that she changed her perineal pad the client
about an hour ago. On inspection, the c) Follow the mother's request as the
nurse notes that client did not indicate a continued desire
for an epidural once the mother made
the request
d) Ask the client in a nonthreatening a) Nutrition assessment
way if she wishes to have an epidural, b) Biophysical assessment
and then speak with the primary health c) Chromosomal assessment
care provider d) Mental health assessment

2. A labor and delivery nurse is caring 5. What is the first thing the nurse
for a woman with no prenatal care who should do when caring for a pregnant
presented with vaginal bleeding and woman with a disability?
abdominal pain. Late decelerations were
noted on the fetal heart monitor. Which a) Offer guidance for care measures
finding in the client history would cause during pregnancy
the nurse to suspect placental b) Establish the impact of the disability
abruption? on a woman's lifestyle
c) Make arrangements for a home birth
a) The client has had multiple sexual d) Involve community and social
partners and is positive for HIV supports for the client
b) The client works 12-hour shifts in a
busy restaurant in the city 6. While in the waiting room of the clinic,
c) The client stated, "I use cocaine a pregnant client begins to choke on a
about four times per week" piece of food. Which action by the nurse
d) The client rides a motorcycle to and would be appropriate?
from work
a) Insert a finger in the mouth to grasp
3. A pregnant client in the first trimester the object
is experiencing severe nausea. The b) Perform backward thrusts on the
client asks the nurse if marijuana could chest with the fists
be used as a natural remedy to treat the c) Place the client in a supine position
nausea. How does the nurse best on the floor
respond? d) Use two fingers to compress the
abdomen
a) "Marijuana smoking is not
recommended; tinctures or edible 7. What complication will the nurse be
preparations have less risk" alert for in the client who uses cocaine
b) "Marijuana use is illegal and should during the pregnancy?
not be used in pregnancy"
c) "Marijuana use may affect fetal a) Placenta previa
neurologic development and is not b) Gestational diabetes
recommended" c) Hyperemesis gravidarum
d) "Using small doses of marijuana is d) Placental abruption (abruptio
believed to pose a low risk to the fetus" placentae)

4. A 41-year-old client at 11 weeks' 8. All of the following are commonly


gestation has arrived for a prenatal visit. seen in infants born to cocaine-addicted
When reviewing the client's past office mothers except:
visit report, which assessment would the
nurse anticipate specifically related to a) Intracranial hemorrhage
the client's age? b) Lethargy
c) Irritability
d) Muscle rigidity e) Stress the importance of attending
monthly health care provider
9. A pregnant adolescent tells the appointments
school nurse, "I want to go to the
primary health care provider and have 12. The nurse is preparing for a prenatal
them check my baby, but I don't want to visit with a pregnant woman who cannot
tell my parents and the doctor probably ambulate because of a thoracic spine
will not see me without them." What is injury several years ago. What should
the best response by the nurse? the nurse do first to prepare for this
client's visit?
a) "You are considered emancipated
and may receive health care for you and a) Educate the client on the effects of
your baby without parental consent" pregnancy and birth on women with
b) "You are probably correct. However, disabilities
you should tell your parents and have b) Have assistants available to lift the
them take you to see a health care client onto the examining room table
provider" c) Discuss the client's inability to provide
c) "I am sorry that you are in this care for her child adequately
situation, but it is important for you to d) Assess the impact of the disability on
see the health care provider" the client's lifestyle before offering
d) "I am sure your parents will be upset assistance
when they find out you are pregnant, but
they will surely take you for health care" 13. The nurse is eating lunch in the
hospital cafeteria when she notices a
10. Which postpartum complication is pregnant woman, apparently in her third
the nurse most likely to see in a mother trimester, choking on her food. After
who abuses marijuana? assessing that the client cannot speak,
what should be the nurse's next
a) Infection intervention?
b) Postpartum hemorrhage
c) Hemorrhoids a) Deliver back blows
d) Inadequate milk production b) Deliver abdominal thrusts
c) Deliver chest thrusts
11. The nurse is caring for a pregnant d) Perform a blind finger sweep
client considered at high risk for
pregnancy complications. What nursing 14. The nurse is caring for a pregnant
action(s) included in the plan of care client who has a preexisting spinal cord
help achieve a positive outcome? Select injury. During labor, the client suddenly
all that apply. experiences a headache and blood
pressure of 298/100 mm Hg. What is the
a) Make sure the client maintains bed nurse's priority intervention?
rest for the duration of the pregnancy
b) Provide educational pamphlets on a) Call for assistance
topics such as nutrition and exercise b) Elevate the client's head, maintaining
c) Provide time for the client and support spinal precautions
person to ask questions c) Contact the health care provider
d) Inform the client of consequences d) Rotate the client into the side-lying
and potential negative outcomes position using the log-rolling method
15. The nurse is caring for a pregnant 18. The nurse is asking a pregnant
adolescent client. Which complication(s) adolescent client about dietary intake.
of pregnancy is this client at increased The client states, "I eat well." How will
risk for due to age? Select all that apply. the nurse respond?

a) Multiple gestation a) "When you say well, would you say


b) Gestational diabetes that includes fruits and vegetables but
c) Iron-deficiency anemia not refined sugar?"
d) Preterm labor b) "If you are not eating fruits and
e) Preeclampsia vegetables, you will need to take your
prenatal vitamins daily"
16. A pregnant client at 22 weeks' c) "Would you please tell me what you
gestation is cut on the finger by some ate in the last 24 hours? I would like a
rusty metal fencing. What medication(s) clear idea of your intake"
will the nurse anticipate in the primary d) "That is great. It is good to hear that
health care provider's prescriptions for you know you will need to eat well for
this client? Select all that apply. you and the baby"

a) Hepatitis B vaccine 19. The nurse is caring for a pregnant


b) Hepatitis B immunoglobulin client with cerebral palsy who uses
c) Rh immunoglobulin forearm crutches for mobility. Which
d) Tetanus, diphtheria, pertussis (Tdap) question will help the nurse assess this
vaccine client's mobility needs as the pregnancy
e) Tetanus immunoglobulin progresses?

17. The parent of a pregnant adolescent a) "How will you maintain intimacy with
says to the nurse, "I don't know what is your partner during the pregnancy?"
wrong with my daughter. I found her b) "Will you need a crib with an
eating chalk the other day and she says adjustable side-rail to use for your
she is craving it!" What information can baby?"
the nurse give to the parent? c) "As your abdomen gets larger, what
will you need to maintain your mobility?"
a) "Gastrointestinal upset may be d) "Do you have a wagon you can use to
causing this behavior. Is she having a lot transport your baby?"
of morning sickness with her
pregnancy?" 20. A pregnant 16-year-old has decided
b) "That is not so unusual. Pregnant to have an amniocentesis for a positive
people often crave strange things during Down syndrome screen. How should the
pregnancy and this must just be a nurse counsel this client?
craving"
c) "This could be related to a substance a) "Because the father of the baby is
use disorder. Do you know of any other over 18, he can sign the consent form
high-risk behavior?" for the procedure"
d) "We will check her hemoglobin level. b) "Because you are not married, you
Often, iron deficiency anemia will cause and the father of the baby have to sign
a craving for inedible substances" for the procedure"
c) "Because you are a minor, you will c) Because the daughter is still a minor,
need one of your parents to sign the nurse may not speak with her alone
consent for the procedure" without the mother's permission
d) "This is ultimately your decision. You d) If the daughter acquires permission
do not need your parent's permission for from her father, she can speak with the
this procedure" nurse confidentially without the mother's
permission
21. A nurse is conducting a class for
pregnant women about prevention of
unintentional (accidental) injury. The 23. The nurse obtained a blood
nurse determines the teaching was pressure of 160/96 on a pregnant
successful when the group makes which adolescent at 32 weeks' gestation. A
statement? Select all that apply. baseline blood pressure of 130/60 was
obtained on her first visit. What
a) "If we use throw rugs, they need to intervention does the nurse anticipate
have non-skid backing on them" advising the adolescent to begin?
b) "We need to have good lighting when
reading labels, especially ones on a) She should reduce her sodium intake
medication containers" b) She should begin taking a diuretic to
c) "We need to take frequent rest decrease the amount of fluid
periods so we don't get overtired" accumulating
d) "It's okay to use a small step stool but c) She should begin bed rest, preferably
we shouldn't use any stepladders" in a side-lying position
e) "It's important to stand for longer d) She will need to prepare for a
periods of time so we don't have our cesarean birth immediately to deliver the
knees flexed so much" baby

22. A mother and her 17-year-old 24. A pregnant woman is in an


daughter, who is in her first trimester, automobile accident and does not
arrive for the daughter's first prenatal survive the trauma. The spouse
visit. The daughter will turn 18 in 2 requested that the physician do an
weeks. The mother is visibly upset emergency postmortem cesarean birth.
about her daughter's pregnancy and Which conditions would most promote
interrupts her daughter to answer the infant survival?
nurse's questions that were addressed
to the daughter. The nurse says that she a) The fetus is past 24 weeks and no
would like to talk with the daughter more than 30 minutes have passed
alone. The mother objects. Which of the since the mother died
following should the nurse mention to b) The fetus is past 20 weeks and no
the mother? more than 30 minutes have passed
since the mother died
a) The daughter can simply return in 2 c) The fetus is past 24 weeks and no
weeks, when she is 18, and speak with more than 5 minutes have passed since
the nurse confidentially without the the mother died
mother's permission d) The fetus is past 32 weeks and no
b) The daughter is an emancipated more than 45 minutes have passed
minor and has the right to speak with since the mother died
the nurse confidentially
25. A pregnant adolescent comes to the 28. The nurse is caring for a 16-year-old
clinic and reports that she "feels tired all client who has been in the first stage of
the time." The nurses assesses pale labor for 20 hours. Despite cervical
mucous membranes. Laboratory tests dilation, the client has had a lack of fetal
show a hemoglobin of 10.2 g/dl (102 engagement with poor fetal descent. As
g/L). The nurse is most correct to base the client begins the 21st hour of labor,
the plan of care off which anticipated the nurse suspects which complication
complication? is affecting the client's labor
progression?
a) A physiologic illness such as
gastrointestinal bleeding from a peptic a) Cephalopelvic disproportion
ulcer b) High client anxiety
b) A lack of knowledge on which foods c) A short umbilical cord
to consume to have adequate iron d) Placenta previa
intake
c) Inadequate exercise to compensate 29. A pregnant client at the initial
for the increased blood to the tissues prenatal visit reports using prescription
d) Unsure of how to maintain restful methadone maintenance. What will the
sleep during pregnancy nurse include in the plan of care related
to this medication?
26. The maternal health nurse is
developing the plan of care for an a) Buprenorphine should be substituted
adolescent client who is pregnant. for methadone due to lower
Which risk will the nurse prioritize in the teratogenicity
care plan? b) Methadone use should be tapered
during pregnancy
a) Fetal injury c) Breastfeeding will be contraindicated
b) Imbalanced nutrition due to methadone use
c) Disruption of social interactions d) The neonate should be observed
d) Social isolation closely for abstinence symptoms after
birth
27. Why is a young adolescent at higher
risk for postpartum hemorrhage? 30. A 38-year-old woman at 15 weeks'
gestation comes in for a prenatal visit.
a) Because of inadequate protein intake, What test should the nurse offer at this
her body tissue integrity is weak and time to detect an open fetal spinal cord
feeble or chromosomal defects?
b) A young adolescent is more likely to
be iron deficient a) A clean catch urinalysis and basic
c) A young adolescent has a higher metabolic panel (BMP) and human
likelihood of cephalopelvic disproportion chorionic gonadotropin (HCG) levels
(CPD) b) A nonstress test using a fetal monitor
d) The uterus may not be fully and serum human chorionic
developed and may become gonadotropin (HCG), hemoglobin, and
overdistended hematocrit levels
c) Maternal serum alpha-fetoprotein,
human chorionic gonadotropin (HCG),
and unconjugated estriol levels
d) Screening for venereal diseases: The client stated, "I use cocaine about
syphilis, gonorrhea, and human four times per week."
immunodeficiency virus (HIV)
What is the first thing the nurse should
31. The nurse is caring for a 44-year-old do when caring for a pregnant woman
client experiencing postpartum with a disability?
hemorrhage soon after birth. This is the Establish the impact of the disability on
client's first child. The client has a a woman's lifestyle.
history of a sedentary lifestyle, anemia,
and poor nutritional intake. The client Which of the following statements is true
did not obtain prenatal care until 32 regarding trauma in pregnancy?
weeks' gestation. Which piece of data The uterus is a peripheral organ and will
collected indicated to the nurse that the see severely decreased blood flow in
woman was at risk of postpartum the event of trauma.
hemorrhage?
The maternal health nurse is caring for a
a) A sedentary lifestyle pregnant woman with a physical
b) Age disability. Which action is the nurse's
c) Late prenatal care priority when caring for the client?
d) Primigravida Establish the impact of the disability on
the woman's lifestyle.
The nurse is caring for a laboring client
whose urine drug screen is positive for An adolescent client in labor is dilated 4
cocaine. The client has an epidural. cm and on admission asks for an
Which finding indicates to the nurse this epidural. For cultural reasons, the
client has an actual substance client's mother states that her daughter
dependence? "has to bite the bullet, just like I did." The
The client becomes agitated, very adolescent does not respond to the
restless, and reports increased mother's statement. What should the
discomfort when laying in the bed. nurse do to make sure the client's
request is honored?
The nurse is preparing for a prenatal Ask the client in a nonthreatening way if
visit with a pregnant woman who cannot she wishes to have an epidural, and
ambulate because of a thoracic spine then speak with the primary health care
injury several years ago. What should provider.
the nurse do first to prepare for this
client's visit? At a prenatal visit, an adolescent client
Assess the impact of the disability on expresses that recently she has had a
the client's lifestyle before offering strange craving for chalk. When
considering this statement, which
A labor and delivery nurse is caring for a diagnostic test would the nurse
woman with no prenatal care who anticipate the primary health care
presented with vaginal bleeding and provider prescribing?
abdominal pain. Late decelerations were complete blood count
noted on the fetal heart monitor. Which
finding in the client history would cause
the nurse to suspect placental
abruption?
Which postpartum complication is the The maternal health nurse is developing
nurse most likely to see in a mother who the plan of care for an adolescent client
abuses marijuana? who is pregnant. Which risk will the
inadequate milk production nurse prioritize in the care plan?
imbalanced nutrition
A 41-year-old client at 11 weeks'
gestation has arrived for a prenatal visit. The nurse is caring for a pregnant client
When reviewing the client's past office considered at high risk for pregnancy
visit report, which assessment would the complications. What nursing action(s)
nurse anticipate specifically related to included in the plan of care help achieve
the client's age? a positive outcome? Select all that
chromosomal assessment apply.
"Provide educational pamphlets on
The nurse is eating lunch in the hospital topics such as nutrition and exercise."
cafeteria when she notices a pregnant "Stress the importance of attending
woman, apparently in her third trimester, monthly health care provider
choking on her food. After assessing appointments."
that the client cannot speak, what "Provide time for the client and support
should be the nurse's next intervention? person to ask questions."
Deliver chest thrusts.
The nurse obtained a blood pressure of
A mother and her 17-year-old daughter, 160/96 on a pregnant adolescent at 32
who is in her first trimester, arrive for the weeks' gestation. A baseline blood
daughter's first prenatal visit. The pressure of 130/60 was obtained on her
daughter will turn 18 in 2 weeks. The first visit. What intervention does the
mother is visibly upset about her nurse anticipate advising the adolescent
daughter's pregnancy and interrupts her to begin?
daughter to answer the nurse's She should begin bed rest, preferably in
questions that were addressed to the a side-lying position.
daughter. The nurse says that she
would like to talk with the daughter The nurse is completing the first
alone. The mother objects. Which of the prenatal visit assessment for a pregnant
following should the nurse mention to adolescent in her second trimester of
the mother? pregnancy. The adolescent is homeless
The daughter is an emancipated minor and is currently staying with a friend.
and has the right to speak with the Which nursing consideration is
nurse confidentially. important based on the client's
situation? Select all that apply.
What is the best way for a woman with a "Nutritional assessment'"
spinal cord injury who cannot feel labor "Screening for gestational hypertension"
contractions to know that she is in "Laboratory tests for iron-deficiency
labor? anemia"
She will be able to palpate her "Measurement for cephalopelvic
contractions with her hand. disproportion"
"Safety and long-term planning"
The parent of a pregnant adolescent A client who is at 33 weeks' gestation
says to the nurse, "I don't know what is with twins presents to the emergency
wrong with my daughter. I found her room with a wrist injury and suspected
eating chalk the other day and she says fracture after a fall (above). What will the
she is craving it!" What information can nurse include in the care plan for this
the nurse give to the parent? client? Select all that apply.
"We will check her hemoglobin level. "Ensure abdominal shielding during
Often, iron deficiency anemia will cause diagnostic x-ray."
a craving for inedible substances." "Assess for signs of preterm labor."
"Assess for signs of preterm labor."
A 40-year-old primipara has been in
labor for 12 hours and is still only 6
centimeters dilated. What does the
nurse understand may be the cause for
this prolonged labor?
Cervical dilation (dilatation) may not
occur as spontaneously as in a younger
woman.

A pregnant adolescent tells the school


nurse, "I want to go to the primary health
care provider and have them check my
baby, but I don't want to tell my parents
and the doctor probably will not see me
without them." What is the best
response by the nurse?
"You are considered emancipated and
may receive health care for you and
your baby without parental consent."

A pregnant adolescent says to the


nurse, "I do not like this brown pigment
over my face. It makes me feel
self-conscious." What is the best
response by the nurse?
"This will usually fade after you have the
baby. You can use a cover makeup in
the meantime."
D. Discuss the care they will be giving
the newborn upon discharge.
Chapter 23
4. Rapid assessment of a newborn
1. The nurse is teaching a group of indicates the need for resuscitation. The
parents who have preterm newborns newborn has copious
about the differences secretiohs. The newborn is dried and
between a full-term newborn and a placed under a radiant warmer. Which
preterm newborn. Which characteristic action would the
would the nurse nurse do next?
describe as associated with a preterm A. Intubate with an appropriate-sized
newborn but not a term newborn? endotracheal tube.
A. fewer visible blood vessels through B. Give chest compressions at a rate of
the skin 80 times per minute.
B. more subcutaneous fat in the neck C. Administer epinephrine intravenously.
and abdomen D. Clear the airway with a bulb syringe.
C. well-developed flexor muscles in the
extremities 5. The nurse prepares to assess a
D. greater body surface area in newborn who is considered to be
proportion to weight large-for-gestational-age
(LGA). Which characteristic would the
2. A nurse is assessing a postterm nurse correlate with this gestational age
newborn. Which finding would the nurse variation?
correlate with this A. strong, brisk motor skills
gestational age variation? B. difficulty in arousing to a quiet alert
A. moist, supple, plum skin appearance state
B. abundant lanugo and vernix C. birthweight of 7 lb, 14 oz (3,572 g)
C. thin umbilical cord D. wasted appearance of extremities
D. absence of sole creases
6. A nurse is providing care to a large
3. The parents of a preterm newborn for gestational age newborn. The
being cared for in the neonatal intensive newborn's blood glucose
care unit (NICU) level was 32 mg/dL one hour ago.
are coming to visit for the first time. The Breast-feeding was initiated. The nurse
newborn is receiving mechanical checks the newborn's
ventilation, blood glucose level and finds it to be 23
intravenous fluids and medications and mg/dL. Which action would the nurse do
is being monitored electronically by next?
various devices. A. Administer intravenous glucose.
Which action by the nurse would be B. Feed the newborn 2 ounces of
most appropriate? formula.
A. Suggest that the parents stay for just C. Initiate blow-by oxygen therapy.
a few minutes to reduce their anxiety. D. Place the newborn under a radiant
B. Reassure them that their newborn is warmer.
progressing well.
C. Encourage the parents to touch their
preterm newborn.
7. A preterm newborn has received 11. A nurse is reviewing the maternal
large concentrations of oxygen therapy history of a large-for-gestational-age
during a 3-month stay (LGA) newborn.
in the NICU. As the newborn is prepared Which factor, if noted in the maternal
to be discharged home, the nurse history, would the nurse identify as
anticipates a referral possibly contributing
for which specialist? to the birth of this newborn?
A. ophthalmologist A. substance use disorder
B. nephrologist B. diabetes
C. cardiologist C. preeclampsia
D. neurologist D. infection

8. A nurse is developing the plan of care 12. A nurse is assessing a preterm


for a small-for-gestational-age newborn. newborn. Which finding would alert the
Which action would the nurse determine nurse to suspect that a
as a priority? preterm newborn is in pain?
A. Preventing hypoglycemia with early A. bradycardia
feedings B. oxygen saturation level of 94%
B. Observing for newborn reflexes C. decreased muscle tone
C. Promoting bonding between the D. sudden high-pitched cry
parents and the newborn
D. Monitoring vital signs every 2 hours 13. A group of pregnant women are
discussing high-risk newborn conditions
9. The nurse is providing care to a as part of a prenatal
newborn who was born at 36 weeks' class. When describing the
gestation. Based on the complications that can occur in these
nurse's understanding of gestational newborns to the group, which
age, the nurse identifies this newborn would the nurse include as being at
as: lowest risk?
A. preterm. A. small-for-gestational-age (SGA)
B. late preterm. newborns
C. term. B. large-for-gestational-age (LGA)
D. postterm. newborns
C. appropriate-for-gestational-age
10. Which intervention would be most (AGA) newborns
appropriate for the nurse to do when D. low–birth-weight newborns
assisting parents
who have experienced the loss of their 14. A nurse is conducting a class for
preterm newborn? expectant parents about newborns. As
A. Avoid using the terms "death" or part of the class, the
"dying." nurse describes newborns with birth
B. Provide opportunities for them to hold weight variations. The nurse determines
the newborn. that the teaching
C. Refrain from initiating conversations was successful when the class identifies
with the parents. which variation if a newborn weighs 5.2
D. Quickly refocus the parents to a more lb (2,358 g) at
pleasant topic. any gestational age?
A. small for gestational age
B. low birth weight C. irritability
C. very low birth weight D. hypertonicity
D. extremely low birth weight E. feeble cry

15. A nurse is assessing a newborn who 19. The nurse is assessing a preterm
has been classified as small for newborn's fluid and hydration status.
gestational age. Which Which finding would
characteristics would the nurse expect alert the nurse to possible
to find? Select all that apply. overhydration?
A. wasted extremity appearance A. decreased urine output
B. increased amount of breast tissue B. tachypnea
C. sunken abdomen C. bulging fontanels
D. adequate muscle tone over buttocks D. elevated temperature
E. narrow skull sutures
20. The nurse is assessing a preterm
16. A nurse is preparing a presentation newborn who is in the neonatal
for a group of perinatal nurses about intensive care unit (NICU)
common problems for signs and symptoms of
associated with preterm birth. When overstimulation. Which assessment
describing the preterm newborn's risk finding would the nurse correlate
for perinatal with this situation?
asphyxia, the nurse includes which A. increased respirations
factor as contributing to the newborn's B. flaying hands
risk? Select all that C. eupnea
apply. D. increased heart rate
A. surfactant deficiency
B. placental deprivation 21. A nurse is reviewing a journal article
C. immaturity of the respiratory control about newborn pain prevention and
centers management.
D. decreased amounts of brown fat Which information would the nurse most
E. depleted glycogen stores likely find discussed in the article?
A. Newborn pain is frequently
17. After a rapid assessment determines recognized and treated.
that a newborn is in need of B. Newborns rarely experience pain with
resuscitation, the nurse procedures.
would perform which action first? C. Pain is frequently mistaken for
A. Dry the newborn thoroughly. irritability or agitation.
B. Suction the airway. D. Newborns may be less sensitive to
C. Administer ventilations. pain than adult.
D. Give volume expanders.
22. A nurse is developing a plan of care
18. A nurse suspects that a preterm for a preterm newborn to address the
newborn is having problems with nursing diagnosis
thermal regulation. Which of risk for delayed development. Which
findings would support the nurse's measures would the nurse include?
suspicion? Select all that apply. Select all that apply.
A. shallow, slow respirations A. clustering care to promote rest
B. cyanotic hands and feet B. positioning newborn in extension
C. using kangaroo care 26. While caring for a preterm newborn
D. loosely covering the newborn with receiving oxygen therapy, the nurse
blankets monitors the oxygen
E. providing nonnutritive sucking therapy duration closely based on the
understanding that the newborn is at
23. A nurse is assisting the anxious risk for which
parents of a preterm newborn to cope condition?
with the situation. A. retinopathy of prematurity
Which statement by the nurse would be B. metabolic acidosis
least appropriate? C. infection
A. "I'll be here to help you all along the D. cold stress
way."
B. "What has helped you to deal with 27. A woman gives birth to a newborn at
stressful situations in the past?" 36 weeks' gestation. She tells the nurse,
C. "Let me tell you about what you will "I'm so glad
see when you visit your baby." that my baby isn't premature." Which
D. "Forget about what's happened in the response by the nurse would be most
past, and focus on the now." appropriate?
A. "You are lucky to have given birth to a
24. The nurse frequently assesses the term newborn."
respiratory status of a preterm newborn B. "We still need to monitor him closely
based on the for problems."
understanding that the newborn is at C. "How do you feel about giving birth to
increased risk for respiratory distress your baby at 36 weeks?"
syndrome because of D. "Your baby is premature and needs
which factor? monitoring in the NICU."
A. inability to clear fluids
B. immature respiratory control center 28. A 20-hour-old neonate is suspected
C. deficiency of surfactant of having polycythemia. Which nursing
D. smaller respiratory passages intervention(s)
will the nurse utilize to provide care for
25. An LGA newborn has a blood this neonate? Select all that apply.
glucose level of 30 mg/dL and is A. Obtain hemoglobin and hematocrit
exhibiting symptoms of laboratory tests
hypoglycemia. Which action would the B. Provide early feedings to prevent
nurse do next? hypoglycemia
A. Encourage frequent feedings C. Maintain oxygen saturation
B. Feed the newborn 2 ounces of parameters
dextrose water. D. Monitor urinary output
C. Initiate blow-by oxygen therapy. E. Insert a peripheral IV
D. Place the newborn under a radiant
warmer. 29. A nurse is developing a plan of care
for a preterm infant experiencing
respiratory distress.
Which measure will the nurse include in
this plan?
A. Stimulate the infant with frequent
handling.
B. Keep the newborn in an open Shoulder dystocia is a true medical
bassinet. emergency that can cause fetal demise
C. Administer oxygen using an oxygen because the baby cannot be born. Stuck
hood. in the birth canal, the infant cannot take
D. Give intermittent tube feedings. its first breath. Which maneuver is first
attempted to deliver an infant with
30. A late preterm newborn is being shoulder dystocia?
prepared for discharge to home after McRoberts maneuver
being in the neonatal
intensive care unit for 4 days. The nurse After spontaneous rupture of
instructs the parents about the care of membranes, the nurse notices a
their newborn and prolapsed cord. The nurse immediately
emphasizes warning signs that should places the woman in which position?
be reported to the pediatrician knee-chest
immediately. The nurse
determines that additional teaching is A nursing student has learned that
needed based on which parental precipitous labor is when the uterus
statement? contracts so frequently and with such
A. "We will call 911 if we start to see that intensity that a very rapid birth will take
our newborn's lips or skin are looking place. This means the labor will be
bluish." completed in which span of time?
B. "If our newborn's skin turns yellow, it less than 3 hours
is from the treatments and our newborn
is okay. " A client with a pendulous abdomen and
C. "If our newborn does not have a wet uterine fibroid tumors has just begun
diaper in 12 hours, we will call our labor and arrived at the hospital. After
pediatrician." examining the client, the primary care
D. "We will let the pediatrician know if provider informs the nurse that the fetus
our newborn's temperature goes above appears to be malpositioned in the
100.4°F (38°C)." uterus. Which fetal position or
presentation should the nurse most
A 26-year-old primigravida has brought expect in this woman?
her doula to the birthing center for transverse lie
support during her labor and birth. The
doula has been helping her through the Although many women envision a plan
past 16 hours of labor. The laboring of how labor will go, sometimes
woman is now 6 cm dilated. She complications happen, and their plan is
continues to report severe pain in her no longer achievable. When this
back with each contraction. The client happens, what is the best question the
finds it comforting when her doula uses nurse can ask the woman at this time?
the ball of her hand to put "What do you consider your primary
counterpressure on her lower back. goal for the outcome of this pregnancy?"
What is the likely cause of the woman's
back pain?
Occiput posterior position
A woman in labor is having very intense What terminology would the nurse use
contractions with a resting uterine tone to document a newborn who weighs
>20 mm Hg. The woman is screaming 4,000 grams (8.8 lb) or more at birth?
out every time she has a contraction. macrosomia
What is the highest priority fetal
assessment the health care provider A nursing student is learning about fetal
should focus on at this time? presentation. The nursing instructor
Look for late decelerations on monitor, realizes a need for further instruction
which is associated with fetal anoxia. when the student makes which of the
following statements?
A woman presents at Labor and "Transverse lie is the same as when the
Delivery very upset. She reports that fetal buttocks present to the birth canal."
she has not felt her baby moving for the
last 6 hours. The nurse listens for a fetal A laboring client has been pushing
heart rate and cannot find a heartbeat. without delivering the fetal shoulders.
An ultrasound confirms fetal death and The primary care provider determines
labor induction is started. What the fetus is experiencing shoulder
intervention by the nurse would be dystocia. What intervention can the
appropriate for this mother at this time? nurse assist with to help with the birth?
Offer to take pictures and footprints of McRoberts maneuver
the infant once it is delivered.
A 16-year-old client has been in the
A pregnant client at 24 weeks' gestation active phase of labor for 14 hours. An
comes to the clinic for an evaluation. ultrasound reveals that the likely cause
The client called the clinic earlier in the of delay in dilation (dilatation) is
day stating that she had not felt the cephalopelvic disproportion. Which
fetus moving since yesterday evening. intervention should the nurse most
Further assessment reveals absent fetal expect in this case?
heart tones. Intrauterine fetal demise is cesarean birth
suspected. The nurse would expect to
prepare the client for which testing to A client's membranes have just
confirm the suspicion? ruptured. Her fetus is presenting breech.
ultrasound Which action should the nurse do
immediately to rule out prolapse of the
A pregnant client at 30 weeks' gestation umbilical cord in this client?
calls the clinic because she thinks that Assess fetal heart sounds.
she may be in labor. To determine if the
client is experiencing labor, which A woman in active labor with a history of
question(s) would be appropriate for the two previous cesarean births is being
nurse to ask? Select all that apply. monitored frequently as she tries to
"Are you feeling any pressure or have a vaginal birth. Suddenly, the
heaviness in your pelvis?" woman grabs the nurse's hand and
"Are you having contractions that come states, "Something inside me is tearing."
and go, off and on?" The nurse notes her blood pressure is
"Have you noticed any fluid leaking from 80/50 mm Hg, pulse rate is 130 bpm
your vagina?" and weak, the skin is cool and clammy,
"Have you been having any nausea or and the fetal monitor shows
vomiting?" bradycardia. The nurse activates the
code team because the nurse suspects The nurse in a busy L & D unit is caring
the client may be experiencing which for a woman beginning induction via
complication? oxytocin drip. Which prescription should
uterine rupture the nurse question with regard to
titrating the infusion upward for
A woman at 39 weeks' gestation is adequate contractions?
brought to the emergency department in Begin infusion at 10 milliunits (mu)/min
labor following blunt trauma from an and titrate every 15 minutes upward by
vehicle accident. The labor has been 5 mu/min.
progressing well after the epidural when
suddenly the woman reports severe The nurse is assisting with a G2P1,
pain in her back and shoulders. Which 24-year-old client who has experienced
potential situation should the nurse an uneventful pregnancy and is now
suspect? progressing well through labor. Which
Uterine rupture action should be prioritized after noting
the fetal head has retracted into the
A G2P1 woman is in labor attempting a vagina after emerging?
VBAC, when she suddenly complains of Use McRoberts maneuver.
light-headedness and dizziness. An
increase in pulse and decrease in blood A laboring client is experiencing
pressure is noted as a change from the dysfunctional labor or dystocia due to
vital signs obtained 15 minutes prior. the malfunction of one or more of the
The nurse should investigate further for "four Ps" of labor. Which scenario best
additional signs or symptoms of which illustrates a power problem?
complication? Uterine contractions are weak and
Uterine rupture ineffective.

After only 45 minutes of labor, the client The nurse is monitoring the uterine
feels the urge to push. She pushes once contractions of a woman in labor. The
and the baby's head is visible. With the nurse determines the woman is
next push, the head emerges. What is experiencing hypertonic uterine
the immediate risk when the head is dysfunction based on which contraction
delivered too fast? finding?
Perineal tearing erratic

The nurse cared for a client who gave A client arrives in labor and delivery
birth. The duration of labor from the following a motor vehicle accident. She
onset of contractions until the birth of was sitting in the passenger seat and
the baby was 2 hours. How will the wearing a seat belt. On admission the
nurse document the client's labor in the nurse notes vaginal bleeding. The client
health record? says she is 30 weeks' pregnant, but only
Precipitous labor had an initial prenatal visit during which
the pregnancy was confirmed. The
external monitor shows irregular uterine
contractions and a fetal heart rate of
152. The provider orders an ultrasound
examination prior to establishing a plan
of care. What is the priority purpose for
an ultrasound examination in this A woman in active labor suddenly
situation? experiences a sharp, excruciating low
Determine placental location abdominal pain, which the nurse
suspects may be a uterine rupture since
A nurse assesses a client in labor and the shape of the abdomen has changed.
suspects dysfunctional labor (hypotonic The nurse calls a code, and a cesarean
uterine dysfunction). The woman's birth is performed stat, but the infant
membranes have ruptured and does not survive the trauma. A few
fetopelvic disproportion is ruled out. hours later, after the woman has
Which intervention would the nurse stabilized, she asks to hold and touch
expect to include in the plan of care for her infant, and the nurse arranges this.
this client? Later, the nurse's documentation should
administering oxytocin include which outcome statement?
The parents are beginning to
A woman in labor is experiencing demonstrate positive grieving behaviors.
hypotonic uterine dysfunction.
Assessment reveals no fetopelvic A woman has been in labor for the past
disproportion. Which group of 8 hours, and she has progressed to the
medications would the nurse expect to second stage of labor. However, after 2
administer? hours with no further descent, the
uterine stimulants provider diagnoses an "arrested
descent." The woman asks, "Why is this
When caring for a client requiring a happening?" Which response is the best
forceps-assisted birth, the nurse would answer to this question?
be alert for: "More than likely you have
potential lacerations and bleeding. cephalopelvic disproportion (CPD)
where baby's head cannot make it
The nurse would prepare a client for through the canal."
amnioinfusion when which action
occurs? A primigravida whose labor was initially
Severe variable decelerations occur and progressing normally is now
are due to cord compression. experiencing a decrease in the
frequency and intensity of her
Which intervention would be most contractions. The nurse would assess
important when caring for the client with the woman for which condition?
breech presentation confirmed by fetopelvic disproportion
ultrasound?
continuing to monitor maternal and fetal A woman with a history of crack cocaine
status use disorder is admitted to the labor and
birth area. While caring for the client, the
A nursing instructor is teaching students nurse notes a sudden onset of fetal
about fetal presentations during birth. bradycardia. Inspection of the abdomen
The most common cause for increased reveals an irregular wall contour. The
incidence of shoulder dystocia is: client also reports acute abdominal pain
increasing birth weight. that is continuous. Which condition
would the nurse suspect?
uterine rupture
A nursing instructor identifies which of Which finding would lead the nurse to
the following as increasing the chances suspect that the fetus of a woman in
of infection when coupled with labor is in hypertonic uterine
prolonged labor? dysfunction?
ruptured membranes contractions most forceful in the middle
of uterus rather than the fundus
A nurse is caring for a client who is
diagnosed with a breech presentation A multigravida client at 31 weeks'
and in the transition stage of labor. The gestation is admitted with confirmed
nurse is aware that which is common at preterm labor. As the nurse continues to
birth? monitor the client now receiving
A thick meconium magnesium sulfate, which assessment
findings will the nurse prioritize and
The nurse is caring for a client report immediately to the RN or health
suspected to have a uterine rupture. care provider?
The nurse predicts the fetal monitor will Respiratory depression, hypotension,
exhibit which pattern if this is true? absent tendon reflexes
Late decelerations
A client has been in labor for 10 hours,
The nurse is caring for a client in active with contractions occurring consistently
labor. Which assessment finding should about 5 minutes apart. The resting tone
the nurse prioritize and report to the of the uterus remains at about 9 mm Hg,
team? and the strength of the contractions
Sudden shortness of breath averages 21 mm Hg. The nurse
recognizes which condition in this
The nurse is assessing a woman who client?
had a forceps-assisted birth for hypotonic contractions
complications. Which condition would
the nurse assess in the fetus? A client is giving birth when shoulder
caput succedaneum dystocia occurs in the fetus. The nurse
recognizes that which condition in the
A client has arrived at the labor and client is likely to increase the risk for
delivery suite for a scheduled induction shoulder dystocia?
of labor. Which nursing intervention diabetes
should the nurse implement before
starting the oxytocin infusion? The nurse is caring for a client in the
Assessing for uterine contractions transition stage of labor. In which
scenario would the nurse predict the use
The nursing student demonstrates an of forceps may be used to assist with
understanding of dystocia with which the birth?
statement? Abnormal position of the fetal head
"Dystocia is diagnosed after labor has
progressed for a time."
A shoulder dystocia situation is called in
room 4. The nurse enters the room to The nurse is caring for a woman
help and the health care provider says undergoing cervical dilation. Which
to the nurse, "McRoberts maneuver." assessment finding would alert the
What does the nurse do next? nurse to the complication of vasa
Bring the client's knees back toward the previa?
shoulders, causing hyperflexion of the Painless bleeding at the beginning of
hips and rotation of the pubic symphysis cervical dilation

A pregnant client's labor has been A woman receives magnesium sulfate


progressing slower than normal. The as treatment for preterm labor. The
client is visibly anxious and tense, telling nurse assess and maintains the infusion
the nurse, "I am so worried about what at the prescribed rate based on which
is going to happen. And I am so tired finding?
and feel so helpless." Other underlying Respiratory rate of 16 breaths/minute
issues that may be contributing to the
client's slow labor progress have been
ruled out. Which response(s) by the
nurse would be appropriate? Select all
that apply.
"Maybe dimming the lights or some soft
music will help you relax a bit."
"I will keep you updated often on how
you and your baby are doing."
"Things are moving along but
sometimes it can take a little longer."

Which statement describes why


hypertonic contractions tend to become
very painful?
The myometrium becomes sensitive
from the lack of relaxation and anoxia of
uterine cells.

The nurse is assessing a multipara


woman who presents to the hospital
after approximately 2 hours of labor and
notes the fetus is in a transverse lie.
After notifying the RN and primary care
provider, which action should the LPN
prioritize?
Prepare to assist with external version.

The fetus of a woman in labor is


determined to be in a persistent occiput
posterior position. Which intervention
would the nurse prioritize?
Pain relief measures
Chapter 24 B. contaminated formula
C. nonsterile catheter insertion
1. A newborn with severe meconium D. mother's birth canal
aspiration syndrome (MAS) is not
responding to 5. Which action would be most
conventional treatment. Which measure appropriate for the nurse to take when a
would the nurse anticipate as possibly newborn has an
necessary for this unexpected anomaly at birth?
newborn? A. Show the newborn to the parents as
A. extracorporeal membrane soon as possible while explaining the
oxygenation (ECMO) defect.
B. respiratory support with a ventilator B. Remove the newborn from the
C. insertion of a laryngoscope for deep birthing area immediately.
suctioning C. Inform the parents that there is
D. replacement of an endotracheal tube nothing wrong at the moment.
via X-ray D. Tell the parents that the newborn
must go to the nursery immediately.
2. A nurse is providing care to a
newborn. The nurse suspects that the 6. The nurse prepares to administer a
newborn is developing gavage feeding for a newborn with
sepsis based on which assessment transient tachypnea
finding? based on the understanding that this
A. increased urinary output type of feeding is necessary because:
B. interest in feeding A. lactase enzymatic activity is not
C. temperature instability adequate.
D. wakefulness B. oxygen demands need to be
reduced.
3. A nurse is providing care to a C. renal solute lead must be considered.
newborn who is receiving phototherapy. D. hyperbilirubinemia is likely to
Which action would develop.
the nurse most likely include in the plan Answer: B
of care?
A. keeping the newborn in the supine 7. Which information would the nurse
position include when teaching a new mother
B. covering the newborn's eyes while about the difference
under the bililights between pathologic and physiologic
C. ensuring that the newborn is covered jaundice?
or clothed A. Physiologic jaundice results in
D. reducing the amount of fluid intake to kernicterus.
8 ounces daily B. Pathologic jaundice appears within
24 hours after birth.
4. A newborn has been diagnosed with C. Both are treated with exchange
a group B streptococcal infection shortly transfusions of maternal O- blood.
after birth. The D. Physiologic jaundice requires transfer
nurse understands that the newborn to the NICU.
most likely acquired this infection from
which cause?
A. improper hand washing
8. A nurse is teaching the mother of a
newborn experiencing cocaine 12. While reviewing a newborn's
withdrawal about caring for medical record, the nurse notes that the
the neonate at home. The mother chest X-ray shows a
stopped using cocaine near the end of ground glass pattern. The nurse
her pregnancy. The nurse interprets this as indicative of:
determines that additional teaching is A. respiratory distress syndrome.
needed when the mother identifies B. transient tachypnea of the newborn.
which action as C. asphyxia.
appropriate for her newborn? D. persistent pulmonary hypertension.
A. wrapping the newborn snugly in a Answer: A
blanket
B. waking the newborn every hour 13. A newborn is suspected of
C. checking the newborn's fontanels developing persistent pulmonary
D. offering a pacifier hypertension. The nurse would
expect to prepare the newborn for which
9. A newborn is suspected of having procedure to confirm the suspicion?
fetal alcohol syndrome. Which finding A. chest X-ray
would the nurse B. blood cultures
expect to assess? C. echocardiogram
A. bradypnea D. stool for occult blood
B. hydrocephaly
C. flattened maxilla 14. A preterm newborn is receiving
D. hypoactivity enteral feedings. Which finding would
Answer: C alert the nurse to
suspect that the newborn is developing
10. After teaching the parents of a NEC?
newborn with periventricular A. irritability
hemorrhage about the disorder B. sunken abdomen
and treatment, which statement by the C. clay-colored stools
parents indicates that the teaching was D. feeding intolerance
successful? Answer: D
A. "We'll make sure to cover both of his
eyes to protect them." 15. A nurse is reviewing the plan of care
B. "Our newborn could develop a for a newborn who has been diagnosed
learning disability later on." respiratory
C. "Once the bleeding ceases, there distress syndrome. Which intervention
won't be any more worries." would the nurse closely monitor to
D. "We need to get family members to reduce the child's risk
donate blood for transfusion." for chronic lung disease?
A. Rescue surfactant therapy
B. Mechanical ventilation
C. Oxygen therapy
D. Radiant warmer use
16. A nurse is presenting a review class 19. A nurse is presenting a review class
for a group of neonatal nurses about for a group of neonatal nurses on the
congenital different types of
conditions in the newborn. The nurse congenital heart disease in infants. The
determines that the teaching was group demonstrates a need for
effective based on which additional teaching when
statement made by the group? they identify which condition as an
A. "Usually a definitive cause can be example of increased pulmonary blood
identified." flow (left-to-right
B. "Congenital conditions typically affect shunting)?
a specific body system." A. atrial septal defect
C. "A congenital condition is an anomaly B. tetralogy of Fallot
that develops immediately after birth." C. ventricular septal defect
D. "These conditions are responsible for D. patent ductus arteriosus
nearly half the deaths in term
newborns." 20. The nurse is assessing the newborn
of a mother who had gestational
17. After teaching the parents of a diabetes. Which findings
newborn with retinopathy of prematurity would the nurse expect? Select all that
(ROP) about the apply.
disorder and treatment, which statement A. pale skin color
by the parents indicates that the B. buffalo hump
teaching was C. distended upper abdomen
successful? D. excessive subcutaneous fat
A. "Can we schedule follow-up vision
screenings with the pediatric
ophthalmologist now?"
B. "We can fix the problem with
surgery."
C. "We'll make sure to administer eye
drops each day for the next few weeks."
D. "I'm sure the baby will grow out of it."

18. The nurse frequently assesses the


respiratory status of a preterm newborn
based on the
understanding that the newborn is at
increased risk for respiratory distress
syndrome because of:
A. inability to clear fluids.
B. immature respiratory control center.
C. deficiency of surfactant.
D. smaller respiratory passages.
CH 24
4. The nurse is monitoring a client at 41
1. A client who was in labor for 32 hours weeks' gestation receiving IV oxytocin.
gave birth to a healthy newborn by Which action should the nurse prioritize
emergency cesarean section. Which if noticeable contractions are occurring
statement by this client would alert the every 2 minutes, lasting 60 to 90
nurse that the client is having problems seconds on the fetal monitor?
with her self-esteem?
a) Stop the IV oxytocin infusion
a) "I am surprised that I need a narcotic b) Change the client's position
pain medication. I am in more pain than c) Notify the health care provider
I expected" d) Administer oxygen 10L via face mask
b) "I feel like I'm not a real mother
because I could not give birth to my
baby the natural way" 5. The health care provider has ordered
c) "I feel like the event was such a a cesarean birth for an exhausted client
whirlwind as everything about the who has been in labor for many hours
delivery happened so fast" with the fetus now showing increasing
d) "I'm disappointed that the labor did signs of distress. As the client and
not progress as expected. I am glad that partner express disappointment in not
it is all over" having a natural birth (and anxiety in not
knowing what will now happen), which
2. The nurse is caring for a client who is response will the nurse prioritize?
being prepared for a cesarean birth due
to nonprogession of the labor process. a) Briefly describe what will be
The nurse will direct the client to the experienced, explain each procedure,
proper placement on the operating table and encourage the partner to participate
and abdominal preparation for which b) Compare the risks of cesarean birth
type of incision? with those of continuing to attempt
vaginal birth
a) Vertical incision c) Tell them firmly that the client must be
b) Low transverse incision ("bikini cut") prepped for surgery and place an IV and
c) Low-abdominal incision catheter
d) Classical incision d) Calmly and confidently describe the
qualifications of the surgical team
3. The client has been progressing well
through the labor process and the health 6. A pregnant client tells the nurse,
care provider prepares to deliver the "Labor and birth sounds so awful. Can't I
infant and performs an episiotomy. The just schedule a cesarean birth instead?"
nurse predicts which situation is the How will the nurse reply?
reason for this procedure?
a) "Our hospital has anesthesiologists
a) Previous perineal laceration with her on staff who can provide you with an
last child epidural for labor to limit pain"
b) Client request so she does not tear b) "Can you tell me more about your
c) Prior episiotomy with last child concerns regarding the labor and birth
d) Shoulder dystocia processes?"
c) "Would you consider taking prenatal 10. A nurse is preparing a client for a
classes to learn more about labor and scheduled cesarean birth. The client is
birth before making this decision?" getting on the stretcher to move from
d) "Your primary health care provider the admission room to the surgical suite.
can review the risks of a cesarean birth When placing the client on the stretcher,
to determine if this is okay" which nursing intervention will be helpful
to prevent supine hypotension
syndrome in the client?
7. To prevent tearing of the perineum of
a client during birth, a physician a) Ensure that the client is warm by
performs a mediolateral episiotomy. The covering with a blanket or sheet
nurse recognizes that an advantage of a b) Place the client's newly inserted
mediolateral episiotomy over a midline Foley catheter lower on the frame of the
episiotomy is which of the following? stretcher
c) Open the clamp on the intravenous
a) Less postpartal discomfort fluids (IV) to quickly infuse
b) Lower risk for rectal mucosal tear d) Have the client lie on the left side
c) Less blood loss during transport
e) Easier healing
11. The nursing instructor is teaching a
8. The nursing instructor is leading a group of student nurses about the
group discussion on the various types of current use of episiotomies during the
birth. The instructor determines the labor process. The instructor determines
session is successful when the students the session is successful when the
correctly categorize forceps or students correctly choose which
vacuum-assisted birth as what type? situation that may require the health
care provider to perform an episiotomy?
a) Spontaneous vaginal birth
b) Surgical birth a) VBAC delivery
c) Failed vaginal birth b) Persistent occiput anterior position
d) Operative vaginal birth c) Shoulder dystocia
d) Multifetal births

9. A pregnant client who had a previous 12. The nurse cares for a client
cesarean birth for breech presentation is scheduled for a cesarean birth for a
planning a vaginal birth and asks the breech presentation. The client requests
nurse how to increase the chances of general anesthesia instead of regional
having a vaginal birth. What will the anesthesia for the procedure because of
nurse include in the response? Select all increased anxiety about being awake
that apply. during the surgery. How does the nurse
respond?
a) Hire a doula for labor support
b) Choose exercises such as swimming a) "The general anesthetic creates a
that promote a cephalic presentation higher risk for your baby, so it is not
c) Seek out nurse-midwife care recommended"
d) Pray regularly for a positive outcome b) "Can you tell me more about what
e) Plan an unassisted birth without you are concerned about during the
intervention surgery?"
c) "There will be a drape placed, so you a) "This procedure helps to push your
do not have to worry about seeing the baby through the birth canal"
surgery" b) "With this procedure, your body
d) "Would you like to have your sends out hormones that make labor
significant other present for the birth?" more effective"
c) "By doing this, the passageway for
13. Which of the following measures is your baby isn't blocked anymore"
the most effective way for the nurse to d) "The fluid that comes out helps
assist the woman recovering from flushes away any debris in the birth
cesarean birth to stimulate lower canal"
extremity circulation?

a) Help the woman to ambulate as soon 16. A nurse is assisting with


as possible preparations for a cesarean birth due to
b) Have the client use a pillow to splint nonprogression during labor. The
the incision when moving about surgeon has arrived, as has the
c) Place ice packs on the woman's anesthesiologist, a student assistant to
calves the surgeon, and multiple nurses. To
d) Apply TED hose whom will the nurse hand the informed
consent paperwork, to be completed
14. The nurse is instructing a primipara prior to the procedure?
who has concerns about the need for a
cesarean birth due to her sister's a) Anesthesiologist
obstetrical history. Which statement by b) Surgeon
the client needs further instruction? c) Student assistant
d) Any of the nurses
a) "My sister is tiny like me and was
unable to have the infant fit through the 17. Eight hours after a cesarean section,
pelvis" a postpartum woman is having heavy
b) "My sister's infant was breech and lochia. She informs the nurse, who
was unable to be turned" suspects which of the following causes?
c) "My sister was in labor for 24 hours
and the labor did not progress" a) Another cause other than the birth
d) "My sister had a cesarean birth with b) Postpartum hemorrhage
the first infant and now needs it for all c) Normal for a cesarean section
subsequent pregnancies" d) Infection

15. A woman arrives at labor and 18. How should the nurse counsel a
delivery unit with contractions every 2-3 client who has arrived for a scheduled
minutes lasting 30-45 seconds. After repeat cesarean birth?
several hours of labor have passed and
not making progress, the health care a) "A Foley catheter will be inserted
provider prepares to perform an before surgery and will be removed
amniotomy. The woman asks the nurse, once you arrive to the postpartum unit"
"How will this help my labor?" Which b) "You will undergo general anesthesia,
response by the nurse would be so you will be asleep during the
appropriate? procedure"
c) "An IV catheter will be placed, and we d) When a transverse abdominal
will do some preoperative blood work. incision ("bikini cut") has been made
Then we will give you some antibiotics" previously
d) "After surgery, you will be
immediately transferred to the 22. The nurse is assessing a neonate
postpartum unit" after a cesarean birth. Which most
common complication should the nurse
be prepared for?
19. A nurse is caring for a client who
requires a cesarean birth because of a) Respiratory distress
labor dystocia. The woman's husband b) Shoulder dystocia
signs the consent form. Which of the c) A facial nerve injury
following individuals is responsible for d) Hemorrhage
obtaining the informed consent prior to a
cesarean birth? 23. A 21-year-old has been in labor for 4
hours; her examination 2 hours ago
a) Admitting nurse revealed 6 cm/100%/-3. During a
b) Physician contraction, she spontaneously ruptures
c) Senior staff nurse her membranes. The doctor checks the
d) The client patient, finds her to be 9 cm/100%/-3,
and states that the cord is palpable.
20. The client plans to undergo a What should the nurse do?
vaginal birth after cesarean (VBAC) and
has recently begun labor. The labor a) Call for help and prepare the client for
nurse is following the fetal monitor an emergency cesarean birth
closely. The results of which monitoring b) Call the pediatricians into the room
will best help the nurse assess for imminent vaginal delivery
contraction strength in the client? c) Set up for imminent vacuum delivery
d) Set up for imminent forceps delivery
a) Internal electronic monitoring
b) Fetal blood sampling 24. When counseling a client about
c) Maternal heart rate monitoring maternal risks and benefits of cesarean
d) Fetal oxygen saturation level birth, which of the following would not
apply?
21. There is much discussion in the
medical community about vaginal birth a) "We will be giving you antibiotics
after a cesarean delivery (VBAC). When preoperatively, but it is still possible to
a woman has had a previous develop an infection at the incision site
emergency cesarean delivery, she is at after the birth"
high risk for a ruptured uterus. When is b) "Complications that may occur
VBAC contraindicated? include hemorrhage, infection, and
damage to the bladder and bowel"
a) When a low cervical uterine incision c) "Although unlikely, if severe
has been made previously hemorrhage does occur, you may
b) When a classical uterine incision has require a hysterectomy"
been made previously d) "A surgical incision may be made at
c) When a vertical abdominal incision the perineum to enlarge the vagina just
has been made previously before the birth of the baby"
25. A client who planned an 28. What is not part of the discharge
unmedicated vaginal birth at a planning for a woman who has had a
freestanding birth center is transferred cesarean birth?
to the hospital for a cesarean birth due
to cephalopelvic disproportion. The a) "Drink plenty of fluids—8 to 10
client says to the nurse, "I feel like such glasses of water a day"
a failure. I could not even give my baby b) "If you have severe abdominal pain or
the gentle start that I wanted!" How does a high fever, contact your health care
the nurse respond? provider"
c) "If you have any vaginal bleeding, you
a) "It sounds like you are disappointed should contact your health care
by the outcome of the birth. Would you provider"
like to talk more about this?" d) "Rest when your baby rests, and don't
b) "You tried your best, and sometimes reject help from family and friends"
these things just do not turn out the way
we expect" 29. The client is 6 hours postoperative
c) "You should focus on the positive from a cesarean birth where epidural
outcome and that you have a healthy morphine was administered. What
baby" assessment(s) will the nurse complete
d) "I am concerned that you might be related to this medication? Select all that
experiencing postpartum depression. apply.
Can we look at a screening
questionnaire?" a) Lochia
b) Level of consciousness
26. Which considerations are addressed c) Oxygen saturation
in the plan of care for a mother healing d) Fundal tone
from a cesarean birth? Select all that e) Respiratory rate
apply. 29. The client is 6 hours postoperative
from a cesarean birth where epidural
a) Interference with organ function morphine was administered. What
b) Stress responses assessment(s) will the nurse complete
c) Problems with self-image related to this medication? Select all that
d) Challenges in bonding with the infant apply.
e) Circulatory complications
a) Lochia
27. What will the nurse include in the b) Level of consciousness
care plan of a client who underwent c) Oxygen saturation
cesarean birth to reduce the risk of d) Fundal tone
postoperative urinary tract infection? e) Respiratory rate

a) Deep breathing and spirometry 30. A woman has just entered the
exercises recovery room after cesarean birth of a
b) Postoperative antibiotic prophylaxis baby weighing 9 lb 14 oz. After
c) Early ambulation after the procedure connecting the client to the blood
d) Removal of indwelling catheter as pressure monitor, it is noted her heart
soon as possible rate (HR) is 120 and blood pressure
(BP) is 80/40. What nursing action
should be a priority?
a) Call the MD a) Gestational diabetes
b) Assess for bleeding b) Poor nutritional status
c) Administer oxygen c) Obesity
d) Administer an IV bolus of fluids d) Venous stasis

31. A client is 6 hours post-cesarean 35. The nurse is teaching a prenatal


birth and is reluctant to ambulate. The class about a cesarean birth. What
client tells the nurse "Won't it hurt? And I benefit of a low transverse uterine
need my rest to recover after surgery. incision should the nurse point out in the
Can't I wait until tomorrow?" What teaching?
should the nurse include in the response
and education for this client? Select all a) Quickest way to deliver a baby
that apply. b) Easier to extend the incision in the
uterus if needed
a) "You will either need to ambulate or c) Best way to deliver a premature
you will need to take an anticoagulant breech baby
medication" d) Least likely to rupture during
b) "If you ambulate and experience pain, subsequent labors
you can take a warm shower for
comfort"
c) "Additional analgesia can be provided A woman is dilated 9 cm, and the fetus
if this is needed for pain with is in an anterior position at +1 station.
ambulation" The bag of water is ruptured. She has
d) "Breastmilk production is enhanced been in labor for several hours and is
by early mobility, making mobility exhausted, so she asks the nurse, "My
important for you" friend was tired at the end of her labor,
e) "Ambulation is important to decrease and the doctor used forceps to help
the risk of complications after surgery" deliver the baby. Could the doctor pull
the baby out with forceps so I can get
32. A client in labor has just learned that this over with?" What is the nurse's best
she will have to undergo a cesarean response?
birth due to failure to progress. The "I don't see why not. Let me ask the
woman looks frightened and stressed. doctor."
Which of the following is a physiological "The cervix must be completely dilated
stress response that the nurse would before forceps can be applied safely."
expect to see in such a client? "No. Once the water bag ruptures, the
doctor cannot apply forceps."
a) Peripheral vasoconstriction "Forceps are too dangerous. I'll get a
b) Decreased blood glucose level vacuum extractor ready for your
c) Decreased heart rate delivery."
d) Bronchial constriction

33. The nurse is caring for a 45-year-old


client who will undergo cesarean birth.
Which age-related finding should the
nurse treat to reduce the client's surgical
risk?
An elective induction is when the birth A primigravida whose baby is presenting
attendant and the pregnant woman breech is scheduled to have a cesarean
agree to the induction of labor without birth. Which of the following would you
medical indications. What should the prepare her for postoperatively?
birth attendant explain to the woman presence of an indwelling catheter
before she can give informed consent to bed rest for the first 4 days
induce her labor? insertion of a nasogastric tube
Induced labor can result in higher costs separation from her infant for 72 hours
for the delivery.
Induced labor decreases the need for
interventions during labor and delivery. A woman whose fetus at 30 weeks'
Induced labors are less painful and gestation is failing to thrive in utero is
progress faster. told her physician wants to deliver the
Induced labor decreases the possibility baby by cesarean birth today. She asks
of cesarean birth. the nurse why this would be preferred to
a vaginal birth. Which of the following
Mrs. Atkins is 40 weeks' pregnant by would be your best response?
ultrasound, and the induction of labor is "There's not an advantage; it's just more
being discussed by Mrs. Atkins and her convenient."
birth attendant. The birth attendant tells "Cesarean birth will reduce pressure on
Mrs. Atkins, "I am going to do a pelvic the immature head."
exam so that I can assess your "You will have reduced pain afterward."
readiness for labor. I will obtain what is "Cesarean birth allows the placenta to
known as a Bishop Score, and it will tell deliver easier."
me how ready you are to go into labor."
What Bishop Score would indicate a A woman who has had a cesarean birth
favorable response to oxytocin-induced asks you if she will always need to have
labor? cesarean births in the future. What
5 or below would be the nurse's best response?
6 or above "You will like cesarean birth so much
3 or below that you will want repeat cesarean births
8 or above in the future."
"There is no way to predict that; it will
depend on your individual uterine
A cesarean delivery is a major surgery anatomy."
and carries with it many risks for "Yes. 'Once a cesarean always a
complications. The most common cesarean' is a well-known rule."
complication is infection. At what site is "Although there are some exceptions,
the infection likely to occur? surgical techniques allow for vaginal
Vagina birth after cesarean birth."
Urinary tract
Uterus
Rectus muscle wall
A nursing student is asking the A nurse correctly recognizes which of
instructor how to assess whether a the following as a current trend within
cervix is ready and favorable for labor the population of birthing women?
induction. The nurse informs the student more natural births with fewer medical
that a score is used known as which of interventions
the following? increased public funds for using doulas
Braden scale increased rates of cesarean sections
Bishop score decreased use of epidurals
Norton scale
Apgar score A patient who had a previous cesarean
birth asks the nurse if all future births
A student observing in labor and must occur the same way. Which
delivery watches a physician introduce a response should the nurse make to
hard plastic instrument with a hook on support the 2020 National Health Goals
the end into the vagina during a digital regarding cesarean births?
examination. The physician proceeds to "All future births must be done through
guide the hook to snag a hole into the cesarean."
membranes. This process causes the "Not if you fulfill the criteria for vaginal
body to release prostaglandins, which birth after cesarean."
induces labor and is known as which of "Your health care provider will let you
the following? know what kind of birth you can have."
membrane stripping "Most women prefer cesarean births
artificial rupture of membranes because they are quicker and cause
mechanical dilatation less pain."
none of the above
A patient who has been in labor for 20
A nursing student correctly identifies hours is being prepared for an emergent
that an episiotomy that extends straight cesarean birth. Which action will help
down into the true perineum is which of ensure the patient's fluid status during
the following? the procedure?
mediolateral episiotomy Provide with a clear liquid tray.
midline episiotomy Encourage intake with ice chips.
lateral episiotomy Initial intravenous fluid therapy.
unilateral episiotomy Administer an antiemetic as prescribed.

Eight hours after a cesarean section, a The physician has just examined the
postpartum woman is having heavy patient and determined that she needs
lochia. She informs the nurse, who to have a cesarean section. He notifies
suspects which of the following causes? the nurse that he will be doing a low
normal for a cesarean section cervical vertical incision into the uterus.
postpartum hemorrhage The nurse knows that the physician has
infection chosen this type of incision over the
another cause other than the birth classical incision because the low
cervical vertical incision:
is larger than a classical incision and will
allow for easier delivery.
reduces the risk of uterine rupture.
is less complicated to perform.
has a lower risk of maternal injury. The maternal health nurse assists the
birth attendant in a forceps-assisted
birth. After the birth of the infant, what is
The nursing instructor is conducting a the nurse's priority?
session exploring the various factors Assess the infant for trauma
related to induction of labor. The Increase the rate of oxytocin
instructor determines the session is Assess the mother for bleeding
successful after the students correctly Apply supplemental oxygen to the
choose which factors are used to mother
determine if the cervix is ripe enough for
induction?
Station, effacement, cervical
consistency, dilatation, and cervical While the nurse is assessing the
position prenatal client's understanding of the
Dilatation, cervical consistency, information provided at a recent
presentation, station, and effacement appointment, the client states, "I want to
Dilatation, cervical position, avoid the pain and long hours of labor,
presentation, station, and effacement so I'm electing to have a cesarean birth."
Effacement, dilatation, presentation, How should the nurse respond?
attitude, and cervical position "A cesarean birth is a method to be
used when vaginal birth is not
possible—it is not a true option."
The nursing instructor is conducting a "It is important to inform the health care
class presenting the various aspects of provider of your decision so that we can
a cesarean delivery. The instructor plan appropriately for when the birth
determines the class is successful after occurs."
the students correctly choose which "Cesarean birth is a low-risk surgery that
complication as the most common is the ideal option in many cases."
postoperative complication? "Many women elect to have a cesarean
Thrombosis birth to avoid the risk of anoxia."
Infection
Laceration of the uterine artery
Pneumonia It was once thought that an episiotomy
made the birth less painful and heal
faster than a spontaneous laceration of
The nurse is assessing a neonate after the perineum. Research has not shown
a cesarean delivery. Which most these assumptions to be true. What is
common complication should the nurse another finding in the research on
be prepared for? episiotomies?
Respiratory distress Causes loss of bowel control after the
Shoulder dystocia birth
A facial nerve injury Increases risk of blood loss immediately
Hemorrhage after delivery
Suturing of episiotomy increases sexual
pleasure following delivery.
Decreases risk of infection after delivery
The nurse is caring for a woman who A nursing student correctly identifies
has had a baby by cesarean birth. which of the following as medical
Which of the following would be the reasons to induce labor? Select all that
most important assessment to make? apply.
whether her abdomen is soft or not postdate pregnancy
whether her perineum is edematous pregnancy that persists beyond the due
if her breasts fill by the third day date
if she wants to breastfeed or not gestational hypotension
premature rupture of membranes
without spontaneous labor
A woman having a cesarean birth will preeclampsia
have a low transverse incision ("bikini
cut"). Which of the following would the
nurse cite as an advantage? To prevent tearing of the perineum of a
The uterine incision will be vertical. client during birth, a physician performs
The skin incision will be just above her a mediolateral episiotomy. The nurse
pubic hair. recognizes that an advantage of a
Because the cervix is cut, the operation mediolateral episiotomy over a midline
proceeds rapidly. episiotomy is which of the following?
Because the fundus of the uterus is cut, lower rlsk for rectal mucosal tear
the infant can be resuscitated rapidly. easier healing
less blood loss
less postpartal discomfort
A 35-year-old P1001 has been admitted
for a scheduled repeat cesarean. As the
nurse prepares the client for surgery,
what is the best way to begin The nurse is caring for a patient
preoperative teaching? recovering from a cesarean birth. Which
Ask her husband, who is fearful and assessment should the nurse make a
anxious, to wait outside. priority for this patient?
Assess how much the woman already Breast filling
knows about cesarean. Plan to breastfeed
Start by going over the risks of cesarean Abdominal texture
so she has enough time to go over each Perineum for edema
one.
Wait until the physician and the
anesthesiologist have completed their
history and assessment, so the teaching A client is being admitted at 41-weeks
can be more focused and directed. gestation with a cervix which is not
ready for labor and delivery. Which
procedure might the nurse prepare the
client for? (Select all that apply.)
membrane stripping
mechanical dilation
locally applied prostaglandins
artificial rupture of membranes
episiotomy
A client is being prepared for a A woman is scheduled to have epidural
scheduled cesarean delivery by the anesthesia for a cesarean birth. Which
medical team. Which intervention will be of the following would the nurse
most critical for the team to monitor in anticipate including in the preoperative
the first 24 hours post cesarean plan of care while she waits for the
delivery? anesthetic?
Urinary output encouraging her to ambulate
Administration of sodium citrate administering an oral antacid
Preparing for blood transfusion administering morphine sulfate IM
Maintaining IV access keeping her turned on her side

The nurse is instructing a primipara who


has concerns about the need for a Following a cesarean birth, a woman
cesarean birth due to her sister's has 3000 ml of intravenous fluid
obstetrical history. Which statement by ordered. The nurse anticipates in the
the client needs further instruction? plan of care that she will be kept NPO
"My sister is tiny like me and was unable except for minimal ice chips until which
to have the infant fit through the pelvis." time?
"My sister's infant was breech and was until 24 hours post-procedure
unable to be turned." until 48 hours post-procedure
"My sister was in labor for 24 hours and until bowel sounds have returned
the labor did not progress." until her bladder tone has returned
"My sister had a cesarean birth with the
first infant and now needs it for all
subsequent pregnancies." How should the nurse counsel a client
who has arrived for a scheduled repeat
cesarean birth?
A multigravid woman has been in labor "You will undergo general anesthesia, so
for several hours and has dilated to 8 you will be asleep during the
cm. Her fetal membranes have not yet procedure."
ruptured and her contractions are not as "A Foley catheter will be inserted before
strong as the birth attendant would like surgery and will be removed once you
them to be. The decision is made to arrive to the postpartum unit."
artificially rupture the woman's "After surgery, you will be immediately
membranes. The LPN/LVN is new on transferred to the postpartum unit."
the unit. What is the LPN/LVN's role in "An IV catheter will be placed, and we
this procedure? will do some preoperative blood work.
Obtain the amniohook and prepare the Then we will give you some antibiotics."
woman for delivery.
Document fetal heart rate before and
after the procedure.
Get the delivery room ready.
Document the woman's vital signs
before and after the procedure.
A 21-year-old has been in labor for 4 Following a cesarean birth, a patient is
hours; her examination 2 hours ago prescribed to receive intravenous fluids.
revealed 6 cm/100%/-3. During a At which time should the nurse
contraction, she spontaneously ruptures anticipate that this patient will be able to
her membranes. The doctor checks the resume an oral intake?
patient, finds her to be 9 cm/100%/-3, 24 hours postprocedure
and states that the cord is palpable. 48 hours postprocedure
What should the nurse do? When bladder tone returns
Call for help and prepare the client for When bowel sounds return
an emergency cesarean birth.
Set up for imminent vacuum delivery.
Call the pediatricians into the room for After learning about the need for a
imminent vaginal delivery. cesarean birth, the pregnant patient
Set up for imminent forceps delivery. begins to cry and hyperventilate. Which
nursing diagnosis should the nurse use
to guide the care that the patient needs
When counseling a client about at this time?
maternal risks and benefits of cesarean Fear related to impending surgery
birth, which of the following would not Risk for infection related to a surgical
apply? incision
"Complications that may occur include Powerlessness related to medical need
hemorrhage, infection, and damage to for cesarean birth
the bladder and bowel." Risk for impaired parent/infant
"Although unlikely, if severe hemorrhage attachment related to unplanned method
does occur, you may require a of birth
hysterectomy."
"We will be giving you antibiotics A client presents at 41 weeks gestation
preoperatively, but it is still possible to for induction of labor and an attempt at
develop an infection at the incision site VBAC. The nurse is aware that which
after the birth." method would be contraindicated for this
"A surgical incision may be made at the client?
perineum to enlarge the vagina just Oxytocin
before the birth of the baby." Artificial rupture of membranes
Balloon catheter insertion
Dinoprostone vaginal insert
A woman is scheduled to have epidural
anesthesia for a cesarean birth. What
should the nurse anticipate to include in A new mother calls the clinic on her
the preoperative plan of care while the fourth day after delivery and reports
patient waits for the anesthetic? difficulty urinating and defecating
Encouraging her to ambulate because of the perineal pain. What does
Administering an oral antacid the nurse suspect is causing these
Administering morphine sulfate problems?
Keeping in a side-lying position trauma from a Foley catheter
trauma from a cesarean birth
episiotomy
infection
The maternal health nurse is caring for the mucus in their lungs removed by the
multiple women who wish to attempt a pressure experienced in the birth canal."
vaginal birth after cesarean (VBAC). "There is no scientific reason for this
Which client will not be able to attempt a occurrence."
VBAC due to contraindications? "All babies are different so it is unwise to
28-year-old with a placenta previa make comparisons."
41-year-old with two previous cesarean "The pain medication given during the
births surgery may have contributed to the
32-year-old with gestational diabetes mucus build up."
22-year-old with Rh-negative blood

What is the most effective way to


The maternal health nurse is caring for a stimulate circulation after cesarean
group of pregnant clients. Which client birth?
will the nurse determine is most ready ambulation within 4 hours of birth
for labor? ambulation within 8 hours of birth
The client whose Bishop score is 9 Venodyne boots
today. TEDS stockings
The client whose cervical length via
endovaginal ultrasound has increased.
The client who is 41 weeks gestation The nurse is teaching a prenatal class
today. about a cesarean delivery. What benefit
The client whose cervical secretions are of a low transverse uterine incision
negative for fetal fibronectin. should the nurse point out in the
teaching?
Least likely to rupture during
The nurse is caring for a postoperative subsequent labors.
client who has experienced a cesarean Easier to extend the incision in the
birth. Which physiologic response(s) uterus is needed.
should the nurse anticipate? Select all Quickest way to delivery a baby.
that apply. Best way to deliver a premature breech
Increased heart rate baby.
Bronchodilation
Elevated blood glucose level
Peripheral vasoconstriction A woman has just arrived to the
Decreased blood pressure recovery room after an emergency
cesarean delivery and is anxious to
begin breast-feeding her baby. What is
A woman who underwent a primary the best response from the nurse?
cesarean birth for a breech presentation "Sure, let me help you with that."
states that her neonate seems to have "I need to monitor you closely for 30
so much more mucus than her first minutes, first."
baby. Concerned, she asks why this has "Just as soon as you get to the
happened. How should the nurse postpartum unit in an hour or 2."
respond? "You will not be able to breastfeed until
"Babies born by cesarean section have you can get up and walk around."
more respiratory complications because
they do not have the benefit of having
The nurse is caring for a client who is The need for an amniocentesis to do
postoperative after a cesarean birth and cervical length measurements.
receiving treatment with antibiotics. For Increased costs associated with both of
which condition(s) should the nurse these 2 newer methods.
monitor? Select all that apply. Shorter time need for obtaining the
Opportunistic infections Bishop score results
Renal insufficiency Increased level of predictabiity with fetal
Elevated liver enzymes fibronectin levels.
Hypokalemia
Tachycardia
A patient having a cesarean birth will
have a low segment incision. What
A client underwent a cesarean birth due should the nurse explain to the patient
to fetal distress. The medical records as an advantage for this type of
reflect a low transverse uterine incision incision?
was used. When speaking with the The uterine incision will be vertical.
nurse the client questions her ability to Vaginal deliveries can occur with future
give birth vaginally with her next births.
pregnancy. What response by the nurse Because the cervix is cut, the operation
is most appropriate? proceeds rapidly.
"You will be able to have a vaginal birth Because the fundus of the uterus is cut,
with your next birthing experience." the infant can be resuscitated rapidly.
"The type of incision used on your
uterus may hinder your ability to have a
vaginal birth with your next birthing A woman is being seen in the clinic for
experience." care during her first pregnancy and
"The reason for your cesarean birth may birth. The nurse will educate this woman
indicate an inability to have a vaginal and her significant other about their
birth in the future." planned cesarean delivery and what can
"Having a cesarean birth does not mean be expected. When should the nurse
you will have a cesarean birth with provide this family education?
future births." At a preset time one month prior to the
scheduled delivery
Only when she and her significant other
A pregnant woman who required labor can come to the clinic together
induction for a previous pregnancy says When the client presents for admission
the to nurse, "The last time the for the procedure
practitioner used a score to see if I was Each time the nurse sees the woman
ready for induction. But I've heard of before surgery
other methods that have been
developed since then." The nurse
understands that the woman is referring The maternal health nurse is assisting a
to cervical length measurements and birth attendant in the use of low-outlet
fetal fibronectin levels. When forceps for a forceps-assisted birth. At
responding to the woman, which which point of the birth will the nurse
information would the nurse need to expect the use of the device?
keep in mind when telling the woman When the fetus requires rotation to an
about these methods? anterior position
When the fetal head can be seen in the The type of skin incision done for a
introitus cesarean section will indicate the type of
When the fetal station is equal or uterine incision.
greater than +2 true
When the fetal head is well engaged but false
station is less than +2

The nurse is caring for a client who is


A primigravida is admitted to the insisting on a vaginal delivery. For which
labor-and-delivery suite for an elective situation should the nurse explain a
induction with oxytocin. The nurse vaginal delivery is contraindicated?
caring for this client knows that one of Select all that apply.
the risks with oxytocin inductions is: Spina bifida
uterine hyperstimulation. Macrosomia
fetal distress. Transverse lie
maternal hypertension. Breech presentation
spontaneous rupture of membranes. Preterm delivery
uterine hyperstimulation.
According to the National Vital Statistics
System (NVSS), what was the national The nurse is caring for a group of clients
cesarean rate in 2014? on the birthing unit. For which client(s)
10% should the nurse anticipate an
15% emergency cesarean birth? Select all
25% that apply.
32% Client with bright red, painless vaginal
35% bleeding
Client with a fetus exhibiting variable
decelerations
Place the following events in order of Client with abnormal amniotic fluid
occurrence for administration of spinal volume
anesthesia before an elective primary Client with three contractions in 20
cesarean birth in a woman who is not in minutes, lasting 35 seconds each, in
labor. active phase of labor
1. The woman enters the OR. Client whose labor pain is not
5. The client lies down on the delivery adequately controlled by epidural
table, and a wedge is placed. analgesia
2. The nurse instructs the client to sit
and make a C-curve with her back.
3. The anesthesiologist cleanses the
back.
4. Spinal anesthesia is administered.
There is much discussion in the medical The nurse is caring for a client who is
community about vaginal birth after a Jehovah's Witness and was expecting a
cesarean delivery (VBAC). When a vaginal birth. The client needs a
woman has had a previous emergency cesarean birth during labor and is at
cesarean delivery, she is at high risk for high risk for hemorrhage. Which action
a ruptured uterus. When is VBAC should the nurse prioritize at this time?
contraindicated? Evaluate whether the client should be
when a transverse abdominal incision cared for in another setting
("bikini cut") has been made previously Provide the client and family with a
when a classical uterine incision has private time to pray before surgery
been made previously Inform the client that certain life-saving
when a vertical abdominal incision has interventions may be required if
been made previously complications arise
when a low cervical uterine incision has Determine if chaplain services that align
been made previously with the client's beliefs are available
when a classical uterine incision has
been made previously

Which woman is most likely to have a


successful VBAC (vaginal birth after
cesarean)?
A 41-year-old who has had 1 previous
cesarean delivery and has gone into
labor spontaneously
A 20-year-old who has had 2 previous
cesarean deliveries
A 35-year-old who has had 1 previous
cesarean delivery and has been induced
with Pitocin
A 32-year-old who has had 1 previous
cesarean delivery, has gone into labor
spontaneously, and then has been
augmented with AROM

The nurse is monitoring the labor of a


woman attempting a vaginal birth after
cesarean (VBAC) delivery. The nurse
correctly suspects a uterine rupture
when which of the following occur?
Select all that apply.
Onset of fetal tachycardia
Excessive maternal pain
Increased vaginal bleeding
Unrelenting uterine contraction
Signs of maternal shock
A nurse is caring for the following four
Chapter 25 laboring patients. Which clients should
the nurse be prepared to monitor closely
for signs of postpartum hemorrhage
A client, 1 day postpartum (PP), is being (PPH)? Select all that apply.
monitored carefully after a significant 1. G1 P0000, delivered a fetal demise at
postpartum hemorrhage. Which of the 29 weeks' gestation.
following should the nurse report to the 2. G2 P1001, prolonged first stage of
obstetrician? labor.
1. Urine output 200 mL for the past 8 3. G2 P0010, delivered by cesarean
hours. section for failure to progress.
2. Weight decrease of 2 pounds since 4. G3 P0200, delivered vaginally a
delivery. 42-week, 2,200-gram neonate.
3. Drop in hematocrit of 2% since 5. G4 P3003, with a succenturiate
admission. placenta.
4. Pulse rate of 68 beats per minute
A client just delivered the placenta
pictured below. For which of the
A mother, G1 P1, who delivered a 2,800 following complications should the nurse
gram baby vaginally 30 minutes earlier, carefully observe the woman?
is transferred to the postpartum unit. 1. Endometrial ischemia.
She pushed for 45 minutes and the 2. Postpartum hemorrhage.
placenta was delivered 10 minutes later. 3. Prolapsed uterus.
She is receiving an intravenous with 20 4. Vaginal hematoma.
units oxytocin added. The postpartum
nurse questions why the oxytocin was Which of the following is a priority
added to the IV bag. Which of the nursing diagnosis for a woman, G10
following responses by the transferring P6226, who is PP1 from a spontaneous
nurse is most likely? vaginal delivery with a significant
1. "The medication was added 10 postpartum hemorrhage?
minutes ago to prevent excess bleeding 1. Alteration in comfort related to
during her transfer." afterbirth pains.
2. "The medication was added 2. Risk for altered parenting related to
immediately after the baby's birth to grand multiparity.
promote placental delivery." 3. Fluid volume deficit related to blood
3. "The medication was added after the loss.
placenta was delivered because of its 4. Risk for sleep deprivation related to
rapid separation." mothering role.
4. "The medication was added while she
was pushing to speed up the baby's A mother, G6 P6006, is 15 minutes
birth." postpartum. Her baby weighed 4,595
grams at birth. For which of the following
complications should the nurse monitor
this client?
1. Seizures.
2. Hemorrhage.
3. Infection.
4. Thrombosis.
A breastfeeding woman has been A multiparous client whose fundus is
diagnosed with retained placental firm and midline at the umbilicus 8 hours
fragments 4 days postdelivery. Which of after a vaginal birth tells the nurse that
the following breastfeeding when she ambulated to the bathroom
complications would the nurse expect to after sleeping for 4 hours, her dark red
see? lochia seemed heavier. Which of the
1. Engorgement. following would the nurse include when
2. Mastitis. explaining to the client about the
3. Blocked milk duct. increased lochia on ambulation?
4. Low milk supply. 1.Her bleeding needs to be reported to
the physician immediately.
A woman who has given birth to a 2.The increased lochia occurs from
healthy neonate is being discharged. As lochia pooling in the vaginal vault.
part of discharge teaching, the nurse 3.The increase in lochia may be an early
should instruct the client to observe sign of postpartum hemorrhage.
vaginal discharge for postpartum 4.This increase in lochia usually
hemorrhage and notify the health care indicates retained placental fragments"
provider about:
1.Bleeding that becomes lighter each A primiparous client who was diagnosed
day with hydramnios and breech
2.Clots the size of golf balls presentation while in early labor is
3.Saturating a pad in an hour diagnosed with early postpartum
4.Lochia that lasts longer than 1 week hemorrhage at 1 hour after a cesarean
birth. The client asks, "Why am I
A client has experienced a postpartum bleeding so much?" The nurse responds
hemorrhage. The primary health care based on the understanding that the
provider verbally ordered carboprost most likely cause of uterine atony in this
tromethamine (Hemabate) 0.25 mg IM client is which of the following?
stat at the time of the hemorrhage and 1.Trauma during labor and birth.
this was given by the nurse. The primary 2.Moderate fundal massage after birth.
health care provider put a prescription 3.Lengthy and prolonged second stage
into the electronic medical record for of labor.
0.25 mg carboprost tromethamine IV 4.Overdistention of the uterus from
stat. When seeing the prescription, how hydramnios."
should the nurse administering the
carboprost tromethamine respond? A multiparous client visits the urgent
1.Ask the charge nurse to have a care center 5 days after a vaginal birth,
discussion with the primary health care experiencing persistent lochia rubra in a
provider about the prescription. moderate to heavy amount. The client
2.Initiate an incident report. asks the nurse, "Why am I continuing to
3.Call the primary health care provider, bleed like this?" The nurse should
discuss the prescription, and request instruct the client that this type of
revision if heard correctly. postpartum bleeding is usually caused
4.Wait until the primary health care by which of the following?
provider returns to the unit and discuss 1.Uterine atony.
the situation in person. 2.Cervical lacerations.
3.Vaginal lacerations.
4.Retained placental fragments.
A primiparous client develops uterine following clients should the nurse
atony and postpartum hemorrhage 1 assess first?
hour after a vaginal birth. The physician 1.A multiparous client at 48 hours
has prescribed IM prostaglandin-F2a. postpartum who is being discharged.
After administration of the medication, 2.A primiparous client at 2 hours
the nurse should observe the client for postpartum who gave vaginal birth to a
which of the following? term neonate.
1.Tachycardia. 3.A multiparous client at 24 hours
2.Hypotension. postpartum whose infant is in the
3.Constipation. special care nursery.
4.Abdominal distention 4.A primiparous client at 48 hours after
cesarean birth of a term neonate.
A nurse on the labor-and-birth unit
transfers a primiparous client and her
term neonate to the mother-baby unit 2 1. A gestational diabetic client, who
hours after the client gave vaginal birth delivered yesterday, is currently on the
to the neonate. Which of the following postpartum unit. Which of the following
information is a priority for the nurse to statements is appropriate for the nurse
report to the nurse receiving the client to make at this time?
on the mother-baby unit? 1. "Monitor your blood glucose five times
1.Firm fundus when gentle massage is a day until your 6-week check-up."
used. 2. "I will teach you how to inject insulin
2.Evidence of bonding well with the before you are discharged."
neonate. 3. "Daily exercise will help to prevent
3.Labor that lasted 12 hours with a you from becoming diabetic in the
1-hour second stage. future."
4.Temperature of 99°F (37.4°C) and 4. "Your baby should be assessed every
pulse rate of 80 bpm. 6 months for signs of juvenile diabetes."

The nurse delegates the care of a A client, who is 2 weeks postpartum,


multiparous client who gave birth to a calls her obstetrician's nurse and states
viable term neonate vaginally 30 hours that she has had a whitish discharge for
ago and is preparing to be discharged to 1 week but today she is, "Bleeding and
a licensed practical nurse (LPN). The saturating a pad about every '12 hour."
nurse should instruct the LPN to notify Which of the following is an appropriate
the nurse if the client exhibits which of response by the nurse?
the following? 1. "That is normal. You are starting to
1.Pulse rate of 100 bpm. menstruate again." 2. "You should stay
2.Oral temperature of 99°F (36.8°C). on complete bed rest until the bleeding
3.Excessive perspiration during the subsides."
assessment. 3. "Pushing during a bowel movement
4.Frequent voiding in large amounts. may have loosened your stitches."
4. "The physician should see you.
Please come in whenever you are
The nurse has been assigned to care for ready."
several postpartum clients and their
neonates on a birthing unit. Which of the The nurse is performing a postpartum
assessment on a client who delivered 4
hours ago. The nurse notes a firm A woman, 26 weeks' gestation, has just
uterus at the umbilicus with heavy delivered a fetal demise. Which of the
lochial flow. Which of the following following nursing actions is appropriate
nursing actions is appropriate? at this time?
1. Massage the uterus. 1. Remind the mother that she will be
2. Notify the obstetrician. able to have another baby in the future.
3. Administer an oxytocic as ordered. 2. Dress the baby in a tee shirt and
4. Assist the client to the bathroom. swaddle the baby in a receiving blanket.
3. Ask the woman if she would like the
6. A client has been receiving doctor to prescribe a sedative for her.
magnesium sulfate for severe 4. Remove the baby from the delivery
preeclampsia for 12 hours. Her reflexes room as soon as possible.
are 0 and her respiratory rate is 10.
Which of the following situations could A client, G1P0000, is PP1 from a normal
be a precipitating factor in these spontaneous delivery of a baby boy,
findings? Apgar 5/6. Because the client exhibited
1. Apical heart rate 104. addictive behaviors, a toxicology
2. Urinary output 240 cc/12 hr. assessment was performed; the results
3. Blood pressure 160/120. were positive for alcohol and cocaine.
4. Temperature 100°E Which of the following interventions is
appropriate for this postpartum client?
A client received general anesthesia 1. Strongly advise the client to
during her cesarean section 4 hours breastfeed her baby.
ago. Which of the following postpartum 2. Perform hourly incentive spirometer
nursing interventions is important for the respiratory assessments.
nurse to make? 3. Suggest that the nursery nurse feed
1. Place the client flat in bed. the baby in the nursery.
2. Assess for dependent edema. 4. Provide the client with supervised
3. Auscultate lung fields. instruction on baby care skills.
4. Check patellar reflexes.
A client is postpartum 24 hours from a
The nurse is caring for a postoperative spontaneous vaginal delivery with
cesarean client. The woman is obese rupture of membranes for 42 hours.
and is an insulin-dependent diabetic. Which of the following signs/symptoms
For which of the following complications should the nurse report to the client's
should the nurse carefully monitor this health care practitioner?
client? 1. Foul-smelling lochia.
1. Ineffective lactogenesis. 2. Engorged breasts.
2. Dysfunctional parenting. 3. Cracked nipples.
3. Wound dehiscence. 4. Cluster of hemorrhoids.
4. Projectile vomiting.
A client, 1 day postpartum (PP), is being 4. Give the parents a lock of the baby's
monitored carefully after a significant hair and a copy of the footprint sheet.
postpartum hemorrhage. Which of the
following should the nurse report to the The nurse is circulating on a cesarean
obstetrician? delivery of a G5P4004. All of the client's
1. Urine output 200 mL for last 8 hours. previous children were delivered via
2. Weight decrease of 2 pounds since cesarean section. The physician
delivery. declares after delivering the placenta
3. Drop in hematocrit of 2% since that it appears that the client has a
admission. placenta accreta. Which of the following
4. Pulse rate of 68 beats per minute. maternal complications would be
consistent with this diagnosis?
1. Blood loss of 2000 mL.
A nurse is working on the postpartum 2. Blood pressure of 160/110.
unit. Which of the following patients 3. Jaundice skin color.
should the nurse assess first? 4. Shortened prothrombin time
1. PP1 from vaginal delivery complains
of burning on urination. A client is 1-day post cesarean delivery
2. PP1 from forceps delivery with blood for eclampsia. The client is receiving 5%
loss of 500 mL at time of delivery. dextrose in 'h normal saline IV at 125
3. PP3 from vacuum delivery with cc/hr and magnesium sulfate IV via
hemoglobin of 7.2 g/dL. 4. P03 from infusion pump. Which of the following
cesarean delivery complains of firm and laboratory values should the nurse
painful breasts. report to the surgeon?
1. Serum magnesium 7 mg/dL.
2. Serum sodium 136 mg/dL.
A postpartum woman has been 3. Serum potassium 3.0 mg/dL.
diagnosed with postpartum psychosis. 4. Serum calcium 9 mg/dL.
Which of the following signs/symptoms
would the client exhibit?
1. Hallucinations. The home health nurse is visiting a
2. Polyphagia. client with HIV who is 6 weeks
3. Induced vomiting. postdelivery. Which of the following
4. Weepy sadness. findings would indicate that patient
teaching in the hospital was successful?
1. The client is breastfeeding her baby
The nurse is caring for a couple who are every two hours.
in the labor/delivery room immediately 2. The client is using a diaphragm for
after the delivery of a dead baby with family planning.
visible defects. Which of the following 3. The client is taking her temperature
actions by the nurse is appropriate? every morning.
1. Discourage the parents from naming 4. The client is seeking care for a recent
the baby. weight loss.
2. Advise the parents that the baby's
defects would be too upsetting for them
to see.
3. Transport the baby to the morgue as
soon as possible.
A breastfeeding woman has been A home care nurse is visiting a
diagnosed with retained placental breastfeeding client who is 2 weeks
fragments 4 days postdelivery. Which of postdelivery of a 7-lb baby girl over a
the following breastfeeding midline episiotomy. Which of the
complications would the nurse expect to following findings should take priority?
see? 1. Lochia is serosa.
1. Engorgement. 2. Client cries throughout the visit.
2. Mastitis. . 3. Nipples are cracked.
3. Blocked milk duct. 4. Client yells at the baby for crying.
4. Low milk supply.
A client who is post-cesarean section for
severe preeclampsia is receiving
A client is to receive a blood transfusion magnesium sulfate via IV pump and
after significant blood loss following a morphine sulfate via patient-controlled
placenta previa delivery. Which of the anesthesia (PCA) pump. The nurse
following actions by the nurse is critical enters the room on rounds and notes
prior to starting the infusion? Select all that the client is not breathing. Which of
that apply. the following actions should the nurse
1. Look up the client's blood type in the perform first?
chart. 1. Give two breaths.
2. Check the client's arm bracelet. 2. Discontinue medications.
3. Check the blood type on the infusion 3. Call a code.
bag. 4. Check carotid pulse.
4. Obtain an infusion bag of dextrose
and water. A nurse massages the atonic uterus of a
5. Document the time the infusion woman who delivered 1 hour earlier.
begins. The nurse identifies the nursing
diagnosis: Risk for injury related to
A nurse is caring for the following four uterine atony. Which of the following
laboring patients. Which client should outcomes indicates that the client's
the nurse be prepared to monitor closely condition has improved?
for signs of postpartum hemorrhage 1. Moderate lochia flow.
(PPH)? 2. Decreased pain level.
1. G1P0000, delivery at 29 weeks' 3. Stable blood pressure.
gestation. 4. Fundus above the umbilicus.
2. G2P1001, prolonged first stage of
labor. A postpartum woman has been
3. G2P0010, delivery by cesarean diagnosed with postpartum psychosis.
section. Which of the following is essential to be
4. G3P0200, delivery of 2200-gram included in the family teaching for this
neonate. client?
1. The woman should never be left
alone with her infant.
2. Symptoms rarely last more than one
week.
3. Clinical response to medications is
usually poor.
4. The woman must have her vitals Which of the following is a priority
assessed every two days. nursing diagnosis for a woman,
G10P6226, who is PP 1 from a
A couple, accompanied by their spontaneous vaginal delivery with a
5-year-old daughter, have been notified significant postpartum hemorrhage? 1.
that their 32-week-gestation fetus is Alteration is comfort related to afterbirth
dead. The father is yelling at the staff. pains.
The mother is crying uncontrollably. The 2. Risk for altered parenting related to
5-year-old is banging the head of her grand multiparity.
doll on the floor. Which of the following 3. Fluid volume deficit related to blood
nursing actions is appropriate at this loss.
time? 4. Risk for sleep deprivation related to
1. Tell the father that his behavior is mothering role.
inappropriate.
2. Sit with the family and quietly A woman has just had a macrosomic
communicate sorrow at their loss. baby after a 12-hour labor. For which of
3. Help the couple to understand that the following complications should the
their daughter is acting inappropriately. woman be carefully monitored?
4. Encourage the couple to send their 1. Uterine atony.
daughter to her grandparents. 2. Hypoprolactinemia.
3. Infection.
The blood glucose of a client with type 1 4. Mastitis.
diabetes 12 hours after delivery is 96
mg/dL. The client has received no
insulin since delivery. The drop in serum A client is on magnesium sulfate via IV
levels of which of the following pump for severe preeclampsia. Other
hormones of pregnancy is responsible than patellar reflex assessments, which
for the glucose level? of the following noninvasive
1. Estrogen. assessments should the nurse perform
2. Progesterone. to monitor the client for early signs of
3. Human placental lactogen (hPL). magnesium sulfate toxicity?
4. Human chorionic gonadotropin 1. Serial grip strengths.
(hCG). 2. Kernig assessments.
3. Pupillary responses.
4. Apical heart rate checks
A couple has delivered a 28-week fetal
demise. Which of the following nursing In discussing obstetric emergencies with
actions are appropriate to take? Select a new registered nurse on Labor and
all that apply. Delivery, you initiate a conversation
1. Swaddle the baby in a baby blanket. reviewing the common risk factors for
2. Discuss funeral options for the baby. postpartum hemorrhage. Which of the
3. Encourage the couple to try to get following is considered a risk factor?
pregnant again soon. a. Age > 25
4. Ask the couple whether or not they b. Induction or augmentation of labor
would like to hold the baby. c. Drug induced analgesia
5. Advise the couple that the baby's d. Fetal intrauterine growth restriction
death was probably for the best.
Which of the following is the best
strategy to prevent a postpartum
hemorrhage?
a. Active management of the third stage
of labor
b. Routine administration of a uterotonic
drug like oxytocin
c. Perform an operative vaginal delivery
d. Screen and treat for maternal anemia

Postpartum hemorrhage is a common


maternal morbidity in high resource
countries and is on the rise. Which one
of the following is the most common
cause of postpartum hemorrhage?
a. Retained placental tissue
b. Atonic uterus
c. Coagulopathy
d. Vaginal laceration

Which of the following is not a part of


the active management of the third
stage of labor?
a. Oxytocin soon after delivery
b. Continuous controlled cord traction
c. Transabdominal uterine massage
after placenta delivers
d. Delayed cord clamping

You are attending the delivery of a


36-year-old gravida 6 para 5 female at
term. Her pregnancy has been
uncomplicated, with the exception of
chronic hypertension treated with
Labetalol 100 mg po BID. After a normal
spontaneous vaginal delivery, you note
a large gush of blood, prompt delivery of
the placenta, and brisk vaginal bleeding.
In addition to uterine massage, which of
the following uterotonic agents should
not be used to treat postpartum
hemorrhage in this patient?
a. Misoprostol 800 mcg PR
b. Oxytocin 20 IU in 1000 mL normal
saline. Infuse 500 mL over 10 minutes
then 250 mL/hour
c. Carboprost 250 mcg IM
d. Methylergonovine 0.2 mg IM

You might also like